You are on page 1of 81

FUNDAMENTALS OF NURSING c.

Her training in Kaisserswerth hospital


d. her work during the Crimean war
I. PERSONAL AND PROFESSIONAL
GROWTH AND DEVELOPMENT 7. Which of the following is considered the
pioneer country in Modern Nursing?
A. HISTORICAL PERSPECTIVES IN a. England
NURSING b. United States
c. Denmark
Situation 1: The original focus of nursing was d. England
caring. It still is notwithstanding the present
professional status of nursing. 8. In which of the following periods of
1. Traditionally, nursing was concerned with: development of nursing did nurses learn their
a. Attending to the poor skills through formal curriculum program and
b. Working with the dregs of society clinical training?
c. Caring for the sick and infirmed a. Contemporary
d. Keeping people healthy and well b. Intuitive
c. Apprenticeship
2. These are historical events that most d. Educative
influenced the development of women, except:
a. Women suffrage movement 9. When was the “ spirit of nursing” thought to
b. Liberation of women have started?
c. Year of the women a. time when hospitals and schools of nursing
d. Better education for women were established
b. as early as civilization
3. Nursing has evolved from a subservient role c. Florence Nightigale time
to one that is: d. during establishment of university colleges
a. People-oriented and schools of nursing
b. Handmaiden of doctor
c. Coordinative role 10. Which of the following triads of school of
d. Self-regulatory nursing were first established in the
Philippines?
4. Who initiated the professional nursing a. St. Luke’s Hospital. UST Hospital, Manila
movement in the Philippines? Doctors Hospital
a. Julita V. Sotejo b. Philippine General Hospital, St. Paul
b. Anastacia G. Tupas Hospital, Mary Johnston Hospital
c. Mrs. Manule L. Quezon c. Philippine General Hospital, North General
d. St. Paul’s sisters Hospital, Baguio General Hospital
d. Iloilo Mission Hospital, St. Paul Hospital,
5. Your competence as a professional nurse Mary Johnston Hospital
can best be demonstrated by:
a. Performance in practice based on nursing 11. The founder of the Philippine Nurses
standards Association (PNA)?
b. Leading a healthy lifestyle a. Anastacia Giron-Tupas
c. Adherence to hospital policy b. Julita Sotejo
d. Strict observance of Code of Ethics for c. Mrs. Socorro Salamanca-Diaz
Nurses d. Miss Mary Coleman

Situation 2: To develop respect and love for Situation 3: Having finished with your Master’s
the profession, every nurse must have a good degree in Nursing. You were hired as a faculty
group of its historical developments including member and was assigned to teach the topic
the important personages and their History of Nursing.
contributions. 12. Historically, nursing function were initially
6. Which of the following achievements of provided by the Shamams and the Babylon. At
Florence Nightingale has the greatest impact which period in the nursing history was this
on the professionalization of nursing? seen?
a. establishment of St. Thomas Hospital School a. Apprenticeship Period
of Nursing b. Primitive Period
b. Being the leaser of a religious order devoted c. Colonization Era
to nursing d. Period of Christianization
3. One’s ideas, convictions and attitudes which
13. Florence Nightingale and her contemporary influence his level of health is referred to as
nurse learned nursing skills and techniques as his:
“trainees”. During which period in the nursing a. Health beliefs
history is this? b. Health habits
a. Apprenticeship Period c. Health practices
b. Primitive Period d. Health perceptions
c. Intuitive Period
d. Educative Period 4. The Health Belief Model states that one
taken health action if:
14. Hospice care is a recent development in a. Is consistent with his health vbeliefs
nursing care in the care of the terminally ill b. Get support from family
patients. In this situation, in which setting is c. Agrees with society’s norms
nursing care provided? d. Will prevent his getting sick
a. Sanitaria
b. Homes 5. Health actions that keeps one’s present level
c. Hospitals of health is referred to as:
d. Clinic a. Health enhancement
15. Which of the following is the most b. Health promotion
acceptable instruction in the training of nurses c. Death prevention
according to Florence Nightingale when she d. Health maintenance
established the St. Thomas Hospital School of
Nursing? Situation 2: As a registered nurse, you are
a. Indigenous nurses who are solely devoted to accountable to the public to provide safe,
teaching competent and effective health care.
b. Head nurses and supervisors of specialty 6. You can best contribute to better nursing
hospitals practice by:
c. Qualified nurses who are solely devoted to a. Observing standards of care
teaching b. Charting SOAPIE
d. Residents physician assigned in the ward c. Reading professional journals
d. Following medical protocols
16. Which of the following is the main function
of hospitals and other similar health 7. In professional caring, which of the following
institutions? nursing behaviors is essential?
a. Early diagnosis and treatment of disease a. Show concern for patients
b. Promotion of health and prevention of b. Report promptly for duty
disease c. Observe hospital rules
c. Research and health education d. Perform procedures correctly
d. Autopsy of patients to determine the cause
of death 8. You demonstrate the advocacy role when
you:
B. NURSING AS A PROFESSION a. Defend rights of patients
Situation 1: Nurses provide care to individuals b. Take action on behalf of the patient
in the various stages of health and illness c. Interpret patient needs to health team
1. Which of the following are internal factors d. Provide safe nursing practice
that influence health practice?
a. Family practices 9. Standard of nursing practice serve as basis
b. Norms of society for:
c. Religious beliefs a. Evaluating safe practice
d. Developmental stages b. Executing nursing techniques
c. Performing competent care
2. Health-illness continuum refers to which of d. Assuming appropriate nursing roles
the following statements. A person:
a. Can both be healthy or ill 10. Related learning experiences in health care
b. Is either healthy or sick settings intend nursing students to:
c. Can be at any level of health and illness a. Supplement nursing services of the hospital
d. Is either not healthy or not ill b. Use actual patients for laboratory practice
c. Apply in practice what was learned in class
d. Enhance clinical practice skills
18. The most important functions of the nurse
Situation 3: You are responsible for achieving is:
and maintaining professional and ethical a. Curative care
components. b. Prevention of diseases
11. You understand that professional nursing c. Rehabilitative care
actions are derived from the: d. Promotion of health
a. Actions you plan and implement
b. Utilization of the nursing process 19. In the advocacy role, the nurse:
c. Theory and skills in nursing a. Speaks on behalf of her patients
d. Making a nursing diagnosis b. Respects dignity of patients
c. Defends rights of her patients
12. When a nurse demonstrates competence, d. Safely practice nursing
she can:
a. Out a patient at ease 20. Based on RA 7164, which function is new
b. Perform nursing practice based on standards in the scope of nursing practice:
c. Do nursing procedures effectively a. Focuses on the family as the unit of service
d. Follow doctor’s order efficiently b. Attends to normal delivery
c. Administers IV injection
13. Which statement about value is correct: d. Provides primary health care services
a. Extrinsic value are associated with
maintenance of life Situation 5: You have chosen nursing as a
b. Most individuals possess many personal career. Understanding nursing as a profession
values will help protect the image of nursing in public.
c. Nurse values are essential for quality care 21. Nursing is best defined as a profession
d. Nurse values rarely influence one’s that:
perception of patients a. Is concerned with nursing diagnosis and
treatment
14. The personal conviction that something is b. Help people to do activities that contribute
absolutely right or wrong in all situation: to health recovery or peaceful death
a. Moral belief c. Care for people, sick or well, in any health
b. Personal value setting
c. Legal obligation d. Assist people towards self-care
d. Ethical issue
22. What nursing is and ought to be is
15. A basic structure under which competent embodied in:
café is objectively measured is: a. Code of Ethics
a. Standard b. Standard Office Nursing Practice
b. Law c. Mission Statement
c. Principle d. Philosophy of Nursing
d. Code
23. Which of the following statements is a
Situation 4: As a graduate of a BSN program, most important characteristic of professional
you are expected to demonstrate basic nursing nursing:
skills for safe practice. a. Is community-oriented
16. The most important characteristic of a b. Observes ethical practice
professional nurse is: c. Demonstrate self-control
a. Harmonious relationship with co-workers d. Provides a human resource
b. Critical thinking and decision-making
c. Competence and caring 24. Safe nursing practice requires:
d. Understanding human needs a. Professional competence
b. Completion of BSN
17. The best way to improve your competence c. Registration as a nurse
after initial registration is to: d. Membership in the PNA
a. Practice nursing regularly
b. Take professional adjustment courses 25. Nurses demonstrate professional service
c. Enroll for graduate studies and utilize critical thinking and decision-making
d. Participate in Continuing Professional through:
Education a. Practice based on scientific theories
b. Use of nursing process methodology
c. Views the patient as a human being
d. Participation in research 33. It is envisioned that nursing practice
Situation 6: A sense of history gives you better settings will be mainly:
understanding of nursing as a profession. a. Hospital-based
26. Which of the following is the most b. In any health setting
distinguishing characteristic of a profession? c. Home-based
a. Respects rights of co-workers d. Community-based
b. Orientation towards services
c. Attention to care needs of clients 34. Nursing will be more involved in the
d. Attitude of helpfulness following areas, except:
a. Population projects
27. To prepare for professional practice in the b. Health policy making
next century, you should: c. Strategic Planning
a. Review nursing in light of future needs d. Multi-disciplinary research
b. Update knowledge required of changes in
roles 35. Nursing education and training will produce
c. Anticipate shifts in health care delivery nurses whom will be:
systems a. Multi-skilled competent
d. Extend services to community-based b. Gerontologist
programs c. Family-nurse practitioner
d. Preventive-oriented
28. The most important specific characteristic
of a true professional nurse is:
a. Accountable for her actions C. THEORETICAL FOUNDATION OF
b. Observes professionalism NURSING APPLIED IN HEALTH CARE
c. Competence and ethical work behavior SITUATIONS
d. Sensitive to needs of others Situation 1: After passing the recent board
examination, Ms. Cervera applied for possible
29. What is the most important contribution or employment in various health institutions.
religion to the practice of nursing: While preparing for a possible interview, she
a. Respect for life thought of reviewing the various theories in
b. Humanistic view of man nursing.
c. Spiritual health of patients 1. According to Orem’s Self-Care theory, G2P1
d. High value on service of fellowmen mother, 7 months pregnant, who missed the
last scheduled visit with her obstetrician would
30. Thrust of our government for social require which type of nursing activities?
program is globalization. For nursing, this a. Supportive-educative
means: b. Wholly compensatory
a. Opportunity to work abroad c. Partially compensatory
b. Agreement on terms of nurses employment d. Universal intervention
c. Exchange in nursing knowledge
d. Borderless nursing practice 2. According to Maslow’s Hierarchy of Needs,
which of the following clients would most likely
Situation 7: A professional nurse is assumed to evidence a self-esteem need?
update herself about the profession to maintain a. A pre-eclamptic woman who is on a low-salt
competence practice. diet
31. In the coming millennium, this factor would b. A 60-year old man who has urinary
probably most influence nursing practice: incontinence
a. High technology c. A hypochondriac who constantly goes to a
b. Nursing education doctor for consultation
c. Social issues d. Teen-age boy who is on NPO prior to an
d. Economic issues operation

32. In the next century, the professional nurse 3. The physical, psychological and social
is expected to be: environment in the care of clients is a major
a. A primary health care expert consideration in whose theory of nursing?
b. Mostly working in tertiary settings a. Sister Callista Roy
c. Globally competitive b. Florence Nightingale
d. In community health c. Joyce Travelbee
d. Hildegard Peplau d. “Nursing process is defined as dynamic
interpersonal process between nurse, client
4. In this theory, Abdellah views client health and health care system”.
needs as nursing problems. She describes
these problems as covert or overt problems. 9. Which of the following statements best
An example of a nursing problem that can be supports the concepts that nursing is dynamic?
mostly manifested by covert signs and a. Every patient is a unique physical,
symptoms is the need to: emotional, social and spiritual being
a. maintain good hygiene and physical comfort b. It is important that the patient participate in
b. maintain oxygen supply the overall nursing care plan of the nurse.
c. accept positive and negative expression, c. Nursing practice is expanding in the light of
feelings and reactions the modern developments that take place
d. promote exercise, rest and sleep around her.
d. The health status of every patient is
5. Hildegard Peplau identified four sequential constantly changing and the nurse must be
phases in interpersonal relationships. Which cognizant and responsive to these changes.
phase is characterized by the nurse and
patient concurrently striving to identify the Situation 3: A sound scientific basis is essential
problem, both are beginning to become for quality professional practice.
comfortable with one another with the client 10. Like other professions, what is unique in
setting to the helping environment? Phase of: professional nursing?
a. Resolution a. Its compassionate and caring role
b. Identification b. Its caring role
c. Orientation c. A well defined body of knowledge and
d. Exploitation expertise
d. Its involvement in nursing research
Situation 2: One of the things that the nurse
should do to ensure quality nursing care is to 11. When you assist an individual to do what
formulate a nursing goal for clients based on he cannot normally do in illness, you are using
some theoretical nursing model that she can whose nursing theory?
adopt in planning nursing care. a. Henderson
6. One example of a nursing model is the self- b. Roy
deficit theory. In this model, nursing care c. Orem
becomes necessary when client is unable to d. Abdellah
fulfill biological, psychological, developmental
or social needs. The model was popularized by: 12. The degree to which a nurse behaves as a
a. Dorothea Orem professional is reflected in which of the
b. Hildegard Peplau following?
c. Faye Abdellah a. Accepts, promotes and maintains
d. Virginia Helderson interdependence of theory and practice
b. Works in coordination of other professional
7. The goal of nursing according to the nursing groups
theory of Florence Nightingale is to: c. Demonstrates the ideals of nursing
a. develop interaction between nurse and client d. Is a member of the accredited professional
b. care for and help clients retain total self care nursing organization
c. facilitate the body’s reparative processes by
manipulating client’s environment 13. Florence Nightingale views nursing as:
d. reduce stress so that client can move easily a. Assisting patients towards early recovery
through the recovery process b. Caring for both healthy and ill individuals
c. Utilizing the patient’s environment for health
8. In interacting with patients, a nurse can be d. Helping patients deal with health problems
governed by Travelbee’s nursing model. The
framework for practice in this model is that: 14. Professionalism is demonstrated in which
a. “Interpersonal process is viewed as human of the following behaviors:
to human relationship” a. Maintains interdependence of members of
b. “Client continuously changes and co-exists the health team
with environment” b. Observes moral and ethical values
c. Caring is central and unifying domain for c. Supporting, counseling and assisting other
nursing knowledge and practice”. nurses
d. Communicates and disseminates health d. being free of an acute or chronic disease
knowledge
21. In prioritizing nursing care services to
15. The nursing theorist who gave emphasis on patients in the ward a nurse must remember
adaptation? Maslow’s hierarchy of physiologic needs, the
a. Dorothea Orem greatest priority of which is:
b. Ida Jean Orlando a. oxygen
c. Sister Callista Roy b. love
d. Betty Neuman c. elimination
d. nutrition
16. The nursing theorist who did not believe in
the germ theory? 22. A nurse’s initial priority in creating a
a. Florence Nightingale therapeutic environment for a patient should
b. Imogene King be:
c. Virginia Henderson a. explaining all laboratory and diagnostic
d. Madeleine Leininger procedures to be done
b. providing safe nursing practice
17. The nursing theorist who identified the 14 c. accepting his individuality as a person
basic needs of human. d. orienting him to hospital procedures and
a. Florence Nightingale routines
b. Imogene King
c. Virginia Henderson 23. As a nurse, you assume different roles in
d. Madeleine Leininger the care of your patients. You demonstrate a
patient advocate role when you perform which
18. In the Health Center, the nurse assists a of the following?
couple who chooses ligation for family a. investigate why family members have not
planning. Based on the systems of Dorothea visited the patient
Orem, this is identified as: b. intervene when patients and family have
a. Wholly compensatory conflict
b. Partially compensatory c. interpret the needs of patients to others
c. Self-care theory d. intercede in behalf of the patient
d. Supportive education Situation 5: Lizbeth is reviewing her notes in
preparation for her upcoming exam.
Situation 4: Some of the important things that 24. In concepts of health and illness, who
you learned in school are the theories in postulated that health is the ability to maintain
nursing. As a nurse practitioner, it is expected the internal milieu?
that you integrate these theories in the care of a. Claude Bernart
your patients. b. WHO
c. Walter Cannon
19. The four main concepts common to nursing d. Nightingale
that appear in each of the current conceptual
models are: 25. In Lydia Hall theory, which of the following
1. person represents core?
2. nursing a. Nurturance and factors that are exclusive to
3. environment nursing
4. medicine b. Therapeutic use of self
5. health c. Focuses on nursing related to physician’s
6. psychology order
a. 1, 2, 3 and 5 d. All of the above
b. 1, 2, 3 and 6
c. 1, 2, 3 and 4 26. Founder of the Philippine Nurses
d. 3, 4, 5 and 6 Association?
a. Julita Sotejo
20. The term high-level wellness as described b. Anastacia Giron Tupaz
by H.L. Dunn, is best defined as: c. Eufemia Octaviano
a. surviving beyond the current life expectancy d. Mary Vita Jackson
b. functioning at one’s best biopsychosocial
level 27. Known as the Father of Sanitation?
c. fluctuating on a wellness-illness continuum a. Hippocrates
b. Moses b. Martha Rogers
c. St. Elizabeth of Hungary c. Sister Callista Roy
d. St. Catherine of Siena d. Lydia Hall

28. Presented adaptation model viewing a Situation 7: As a professional nurse, you are
person as a unified bipsychosocial/adaptation expected to come up with decisions that are
system in constant interaction with changing considered legal and ethical. Your action must
environment. The adaptive system inpit control be supported by utilization of different nursing
process output and feedback. theories in the patients.
a. Hildegard Peplau 34. Which of the following theories consider
b. Martha Rogers and utilize nature and environment in the
c. Sister Callista Roy healing process?
d. Lydia Hall a. Ida Jean Orlando
b. Florence Nightingale
Situation 6: Nursing practice is governed by c. Julita V. Sotejo
different theoretical framework postulated by d. Imogene King
known theorists.
29. According to self care and self deficit 35. Florence Nightingale was born in:
theory, when the nurse is expected to a. Germany
accomplish all the patient’s therapeutic self b. Britain
care or to compensate for patient’s inability to c. Italy
engage in self care or when the patient needs d. France
continuous guidance in self care, it is
categorized as: 36. Which is unlikely to Florence Nightingale?
a. Wholly compensatory a. Transcultural nursing
b. Partially compensatory b. Lady with a Lamp
c. Supportive-educative c. Notes in Nursing
d. Conservation of personal integrity d. St. Thomas School of Nursing

30. The four concepts common to nursing 37. What country did Florence Nightingale train
conceptual models: in nursing?
a. Person, nursing, environment, medicine a. Belgium
b. Person, health, nursing, support system b. US
c. Person, environment, health, nursing c. Germany
d. Person, environment, psychology, nursing d. England

31. Identified 14 basic needs and says that the 38. Which of the following is recognized for
unique function of the nurse is to assist the developing the concept of high level wellness?
client, sick or well in the performance of these a. Erickson
activities contributing to health, its recovery or b. Madow
peaceful death. c. Peplau
a. Faye Abdellah d. Dunn
b. Virginia Henderson
c. Ida Jean Orlando Situation 8: A significant milestone influencing
d. Imogene King the development of nursing concepts and
theories was the establishment of Journal of
32. Identified 21 nursing problem areas: Nursing Research. Several nursing theorist
a. Faye Abdellah have published the framework for practice
b. Virginia Henderson according to their respective nursing theory
c. Ida Jean Orlando
d. Imogene King 39. Which of the following best describes a
theory?
33. She introduced interpersonal model and a. It is a group of interrelated beliefs and
define nursing as an interpersonal process of attitudes
therapeutic interactions known as nurse-client b. It is well-defined testable description of a
relationship. She classified its phases as subject
orientation, identification, exploitation and c. It is a description of the relationship
resolution. between professional beliefs and attitudes
a. Hildegard Peplau
d. It is a graphic representation of relationship a. Establishing a meaningful nurse-patient
between different variables relationship
b. Formal education in a university setting
40. Which of the following nurse theorists has c. Assisting patient to cope with illness
stated that the goals of nursing is to maintain d. Manipulation of environment
and promote health, prevent illness and care
for and rehabilitate ill and disabled client 46. Whose theory in nursing are you using
through humanistic science and nursing. when you focus on assisting self-independence
a. Rogers of your patients?
b. Nightingale a. Florence Nightingale
c. Orem b. Levine’s theory
d. Abdellah c. Orem’s theory
d. Maslow’s theory
41. That “nursing care becomes necessary
when client is unable to fulfill biological, 47. The Bachelor of Science in Nursing
physiological development or social needs,” curriculum prepare you to be:
explains which of the following theories? a. Primary health care nurse
a. Adaptation Theory b. A generalist nurse
b. Self-care deficit theory c. Staff nurse in the hospital
c. Theory of the Unitary Man d. A community-oriented nurse
d. Transcultural Care Theory
48. Which of the following activities contribute
42. In her theory, Imogene King defined to the knowledge base of professional nursing?
nursing process as a dynamic, interpersonal a. Nursing research
process between nurse, client and health care b. Graduate studies
system. In this theory, which of the following c. Continuing education
nursing skills is most important to help client d. Seminar workshops
establish positive adaptation to environment?
a. Technical skills Situation 9: In rehabilitation, the professional
b. Assessment skills nurse has a major responsibility for enhancing
c. Environment management skills client’s capacity for self care.
d. Communication skills 49. This is the most important self-care
equipment:
43. Who among the following theorists has a. Maintenance of balance between activity and
identified twenty-one specific client needs or rest
problems in the areas of comfort, hygiene, and b. Maintenance o wholesome recreational
safety, physiological balance, psychological and activities
social factors and sociological and community c. Maintenance of educational work and
factors? activities
a. Dorothy Johnson d. Maintenance of sufficient food
b. Virginia Henderson
c. Sister Callista Roy 50. Based on Orem’s self care theory, care
d. Faye Abdellah during rehabilitation from illness or in advance
chronic illness is:
Situation 8: One of the characteristics of a a. Educative developmental
professional is that its practice is based on a b. Fully compensatory
body of knowledge. An understanding of the c. Partially compensatory
scientific basis of nursing practice will enable d. Compensatory
you to practice quality nursing care.
44. Which of the following expands your critical 51. To enable an individual to develop self-care
thinking and decision-making skills. skills, which teaching method would be most
a. Theories effective:
b. Nursing research a. Demonstration-return demonstration
c. Nursing process b. Role playing
d. Nursing standards c. Lecture-discussion
d. Group dynamics
45. Florence Nightingale considered the nurse
theorist viewed nursing as: 52. At the family level, the most important self
care ability to develop is:
a. Education of members for health b. View video tapes in clinical nursing
b. Health care of chronically dependent c. Teach a clinical nursing subject
members d. Actually work with patients
c. Family dynamics for health care
d. Compliance of ill members to medical 6. In one of the seminars on continuing
regimen professional education (CPE) , one of the
lectures emphasized is the following primary
53. At the community level, the major goal of characteristics of a profession:
self-care program is to: a. Can be used as a stepping stone to go
a. Give the community the sole control over abroad
their health b. Nursing action is best supported by natural
b. Empower people to take responsibility for instincts
own health c. It is governed by a code of ethics based on
c. Mobilize the community for self-help ethical principles
d. Organize the community for basic health d. Emphasis of nursing responsibilities is on
services dependent rather than independent functions

7. The objectives requiring nurses to attend


CPE seminars and workshops are to:
D. CONTINUING PROFESSIONAL a. Increase revenue of the government and
EDUCATION organizations providing CPE
b. Update one’s knowledge and skills relevant
Situation 1: As a professional nurse, you take to nursing
responsibility for personal and professional c. Renew old acquaintances and establish
growth. camaraderie among nurses
1. An important responsibility of a professional d. Update of professionals in the country.
nurse is:
a. Refine competence in safe practice
b. Safety of those she serves
c. Know trends in health and nursing
d. Accountability for her actions E. PROFESSIONAL ORGANIZATIONS IN
NURSING
2. For relicensure, you are required to
complete continuing education professional Situation 1: In the practice of her profession,
education equivalent to: nurses are expected to demonstrate desirable
a. 20 hours per year values and practices
b. at least 2 weeks seminar-workshop 1. One of the expectations is for nurses to join
c. a post basic study professional associations primarily because:
d. 50 hours for 3 years a. Promotes advancement and professional
growth of members
3. You are a live-in private nurse of a family b. Works for raising of funds for nurses’ benefit
with a chronically ill member. You can maintain c. Facilitates and establishes new
your image as a professional nurse by: acquaintances among nurses
a. Doing non-nursing activities d. Assists them in securing jobs abroad
b. Being present during physician’s visit
c. Maintaning good grooming at all times 2. What are considered as most essential to a
d. Socializing with family members good working relationship among members in
any health care setting?
4. The primary purpose for requiring a. Punctuality and efficiency
continuing education for relicensure is to: b. Discipline and loyalty
a. Ensure legal practice c. Personality and neatness
b. Ensure safety of public served d. Intelligence and capability
c. Compliance to Board of Nursing
requirements 3. Which of the following best describes the
d. Improve competency after registration action of a nurse who documents her nursing
diagnosis?
5. The best method of maintaining your clinical a. She documents it and charts it whenever
competence is to: necessary
a. Take advance nursing courses b. She can be accused of malpractice
c. She does it regularly as an important 2. The promotion and protection of health is a
responsibility major responsibility of the Department of
d. She starts it only when the patient is acutely Health, which can be the most successful when
ill there is an active partnership with and/or
involvement of:
4. Which of the following does not govern a. Department of Education and Local
nursing practice? Government
a. RA 7164 b. Professional Health Association
b. RA 9173 c. The people and every citizen
c. BON Resolution for Code of Ethics d. Non-Government organization
d. Board Resolution Scope of Nursing Practice
3. Based on the health belief model, a person
5. A nurse who is maintaining a private clinic in is more likely to take preventive health action
the community and renders service on if he:
maternal and child health among the a. Understands the nature of the disease
neighborhood for a fee is: b. Has access to health care facilities
a. A primary care nurse c. Believes he is susceptible to the disease
b. An independent nurse practitioner d. Believes in the competence of the nurse
c. A nurse midwife
d. A nurse specialist 4. The motivation of individuals to participate
in health promotion activities and take specific
6. As a nurse you should be an actively health action is most effective in which of the
involved member of this nurse’s organization following strategies:
including attending sponsored activities: a. Enactment of a law
a. ICN b. Use of social pressure
b. Alumni Association c. Experimental activities
c. ANA d. Health education
d. NLN
5. Which of the following is a health concern in
7. Which of the following is the non the primary level of prevention?
government agency which provides training a. Development of health habits and practices
programs to promote mental health? b. Poverty alleviation
a. Association of Mental Health Nursing c. Early and prompt treatment
b. PNA d. Case finding
c. Philippine Mental Health Association
d. ORNAP Situation 2: Health is dynamic state in which
the individual adapts to changes in internal and
external environment to maintain a state of
F. THE NURSE IN HEALTH CARE well-being in all dimensions.
1. Eleven key areas of 6. Which of the following is not considered an
responsibilities illness behavior?
2. Fields of nursing a. Interpreting a physical symptom of pain as
3. Roles and functions to its possible cause
b. Ignoring a physical symptom such as
Situation 1: An ounce of prevention is better headache
than a pound of cure. This dictum should make c. Applying for sick leave from work
preventive health services an attractive d. Going to the family physician for
alternative to the curative approach in health consultation
care particularly that more and more people
are under the poverty line and also because of 7. Which of the following are health preventive
the high cost of hospitalization and medical activities?
expenditures. a. Daily exercise
1. Prevention is an important responsibility of b. Eating well-balanced diet
the nurse in: c. Indulging in wholesome recreational
a. Hospital and infirmaries activities
b. All health care settings d. Giving immunization to infants
c. Health centers and industrial clinics
d. Community and school setting 8. The following are negative health behavior
that must be provided to prevent illness.
a. Regular client check-up 15. You are a factory nurse in a bottling
b. Self-medication industry. What priority health promotion
c. Immunization program can you undertake?
d. Adequate sleep pattern a. Physical fitness program
b. Accident prevention program
9. The following is an example of tertiary level c. Formulating work standards
of prevention d. Relaxation exercises
a. Teaching a stroke victim how to check bath
water to prevent burns 16. A nurse provided the patient and the family
b. Administering insulin injection to diabetic with information regarding the client’s care.
patients This constitutes:
c. Supervising Lamaze class to couples a. Patient advocate role
d. Inserting IUD to a mother with two children b. Surrounds role
c. Counselor role
10. The following is an example of secondary d. Patient role
level of prevention:
a. Giving of tepid sponge bath 17. Which of the following statements if made
b. Organizing summer tennis club and act class by the nurse would you consider him/her as a
for children prudent nurse?
c. Campaigning for immunizations a. Questions wrong order of the doctor
d. Doing passive and active exercises for an b. Follow the doctor’s order even if she thinks
orthopedic patient it is wrong
c. Very cautiously asks supervision all the time
Situation 3: Professional nursing practice d. Careful in giving medication in a central line
primarily aims to promote health and prevent
illness. Situation 4: Care of the elderly is included in
11. The most basic goal of health promotion is: the WHO Health for All. In the Philippines, the
a. Healthy elderly is properly cared for at home.
b. Absence of disease 18. Which Maslow’s hierarchy of needs do old
c. No illness people need?
d. Wellness a. Spiritual
b. Emotional
12. Health promotion essentially involves c. Economic
nursing activities that: d. Belongingness
a. Disseminates health knowledge, values and
healthy lives 19. Respiratory disease affect the elderly.
b. Encourage nurse community partnership in Which nursing function is best to assist your
health grandparents?
c. Helps people develop abilities to keep a. Guidance counseling
healthy b. Referral to specialist
d. Enhances people’s quality of life c. Home care management
d. Function specific to disease
13. This is the most basic nursing intervention
in health promotion. 20. Humanitarian issues affect care of elderly.
a. Health information, communication and Filipino values are changing. Which one would
education you maintain in the care of the elderly?
b. Screening and case finding activities a. Security and protection
c. Behavior modification for healthy living b. Educational and recreation
d. Community organization for health action c. Income of elderly
d. Labor and management
14. When you conduct health promotion
activity, an important aspect is: 21. What program would you make for the
a. Giving anticipatory guidance elderly?
b. Coordinating public health education a. Build home for the elderly
c. Assisting in immunization b. Information on physical and social process
d. Case finding c. Children to be responsible for parents
d. Involve elderly in volunteer work
22. In our country, senior citizens are given Situation 6: An understanding of the factors
the privilege in the purchase of their influencing the health care delivery system will
medicines. How much is the discount? enable nurses to adjust to change, create
a. 20% better ways of providing nursing care and
b. 15% develop new nursing roles.
c. 10% 28. Wellness clinics and health education and
d. 25% health education activities have been
integrated in government hospitals and render
appropriate services. Which of the following
Situation 5: The following questions pertain to purposes LEAST help clients in cases of these
the nurse duties and responsibilities. health promotion activities?
23. Concerned with the care of school children, a. maintain maximum functions
the nurse does the following except, b. identify disease symptoms
a. Provides for a safe environment c. reduce the costs of health care
b. Detects, diagnose and treats children with d. promote health habits
health problems
c. Identifies and refers children with eye 29. With regards to illness prevention activities
problems as part of nursing care, which of the following
d . Conduct health education classes will help clients most?
a. promote habits related to health care
24. The school nurse contributes to the health b. manage stress
care and education of the school population c. maintain maximum body function
through which of the following: d. reduce risk factor
a. Health assessment measures
b. Managing school health services 30. By experience, which of the following are
c. Health counseling and coordination with the the MOST common services overlooked by
community nurses and other members of the hospital
d. All of these team in the care of their clients?
a. Health promotion
25. In the provision of care to a family, the b. Diagnosis and treatment
nurse uses the following as a tool in c. Rehabilitation of Patients
determining the family’s strength and specific d. Illness prevention
difficulties.
a. Family coping estimate 31. Which of the following health care agencies
b. Family folder is usually family-centered relatively recent in
c. Family health record popularity and oftentimes focuses on
d. Family health care plan maintenance of comfort and satisfactory
lifestyle of clients in the terminal phase of
26. An important reason why erasures in the illness?
patient’s charts should be avoided is: a. Support Group
a. Chart would look dirty and reflects the nurse b. Community health center
personality c. Non-government organization
b. Recordings may be illegible d. Hospice
c. Chart is a legal document
d. Chartings are essential for endorsement and Situation 7: In taking caré of clients, the nurse
follow-up care of patient assumes responsibility for one’s own values.
Ethnicity, ethical and legal components have
27. The nurse documents all nursing care affected the practice of nursing particularly in
provided to the patient or family for the transforming one’s roles and functions.
following reason. The document: 32. Which of the following statements about
a. Is a tool for the assignment of the health promotion and assisting the clients to
patient’s / family’s condition attain a higher level of health is correct?
b. Describes the nursing problem of the patient a. These are reserved for advance practitioners
or family b. They are an integral part of the application
c. Provides a record of the medications and of the nursing process
procedures done c. These are roles and functions purely for the
d. All these reasons public health agencies to perform
d. They are unrealistic given the large client
population
33. The Philippines is a country where patients 39. Which statement on the Health Belief
come from various ethnic groups. Which of the Model (HBM) is true:
following must the nurse do to effectively take a. HBM identifies vulnerability of individual to
care of their clients from varied geographical become ill
setting? b. HBM identifies an individual’s misconception
a. Minimize transcultural communications on health
b. Work with one ethnic folk healer c. HBM is based on health practices
c. Determine patient’s personal beliefs and d. HBM recommends what health action to take
values
d. Study the ethnic culture of each of the client 40. Based on HBM, a factor most likely to
influence initial action of Mrs.Tulosan to quit
34. Which of the following is the MOST smoking is:
important factor in providing nursing care to a. The support given by the nurse
clients coming from a specific ethnic group? b. Knowledge of smoking-related diseases
a. Environmental control c. Her perceived benefit of the health action
b. Time reorientation d. The family’s approval of her health action
c. Biological intervention
d. Communication 41. This action strategy will most likely
influence the smoking behavior of Miss
35. Which of the following roles match the Tulosan: a. Positive reinforcement from peers
work related activity of the nurses whereby b. Declaring her firm “no smoking” area
he/she functions as clinician, educator, c. An administrative policy to prohibit smoking
manager, consultant and research within a d. A “no smoking” sign
specific practice?
a. Nurse practitioner Situation 9: Nurse Lilian is assigned in the
b. Nurse anesthetist Rehabilitation Unit of HSOD, a tertiary hospital.
c. Clinical specialist 42. Lilian should know that rehabilitation is a
d. Nurse educator health oriented process that assists clients,
sick or well to:
36. Which of the following roles or functions is a. Live with permanent disabilities
fulfilled by the nurse as he/she answers the b. Adjust to his environment and disabilities by
client’s questions about the discharge learning to use resources
medication and ways to hasten to recovery c. Achieve an acceptable quality of life
after abdominal surgery? d. Achieve her greatest possible level of
a. Decision-making physical, mental, social and emotional
b. Teacher functioning
c. Caregiver
d. Manager 43. Achievement of self-care independence is
manifested by which of the following:
Situation 8: Miss Tulosan, a young promising a. Client uses assistive device with assistance
executive, came to the clinic for chronic cough b. Client reports dissatisfaction in her action
and progressive weight loss. There were no c. Client remains withdrawn
abnormal findings noted after a physical d. Client dresses self at a maximal level of
examination. independence

37. You advised Miss Tulosan to quit smoking. 44. In assessing possible footdrop with a client
This is based on the knowledge that: in leg cast, the nurse should let:
a. Smoking is associated with lung cancer a. The client move his toes and foot
b. Nicotine is a throat irritant b. The client massage his foot
c. Smoking is an addictive habit c. The client perform a kicking motion
d. Smoking as relaxing is a myth d. The client flex his knee

38. The first step to take in attempting to 45. You are caring/ assigned to a patient in
change health behavior is: Buck’s extension traction after fracture of the
a. Specific measures to take left tibia. Which of these questions is best to
b. Desire to change ask in assessing for complications?
c. Commitment to change a. Can you feel my pen on your toes?
d. Awareness of need to change b. Can you attend to yourself?
c. Can you feel the weather? 51. Which of the following characterizes health
d. Can you move your left buttock? promotive behavior for Aling Maria
a. Taking medication obtained over the counter
46. A client after an acute attack of MI for a physical symptom
recovers without complication. She is now on b. Having fresh vegetables and fruits regularly
physical conditioning. Lilian should first: in her diet
a. Assist the client in walking inside the room c. Consulting a physician because of a
b. Assist in dangling his client’s legs perceived headache
c. Allow the client to perform active range of d. Withdrawal from work activities of
motion fatigability
d. Check tolerance to activity

Situation: Miss Cruz is a staff nurse in the II. SAFE AND QUALITY CARE
emergency hospital of a remote town in the
south. As a nurse, she takes care of patients A. THE NURSING PROCESS
with varied cases and different ages.
Situation 1: You are a head nurse in a unit
47. One of the two patients she has met is where a project on Quality Assurance (QA) is
Aling Maria, 60 years old, who for two years going to be piloted. As the headnurse, you
have been taking medications and following a expect your staff nurses to demonstrate high
low salt diet to control her blood pressure. level skills in determining nursing diagnosis so
She says, these make her feel so much better. that appropriate nursing care plan is designed
Which of the four models of health best for your patients.
explains Aling Maria’s behavior?
a. agent-host 1. One of the patients in the ward was Rhea,
b. health-belief model 26 years old, who was admitted for excision of
c. health-illness continuum a small cyst in the left breast. Upon further
d. high-level illness interview, you learned from the staff nurse
that Rhea does not know that regular breast
48. In one of her health tasks, Miss Cruz self-examination is important. Which nursing
gathered all four of her patients with similar diagnosis of the said nurse would be most
conditions and conducted health education on appropriate for this situation:
diet and activities for hypertensive patient. a. Non-compliance with cancer-prevention
This kind of nursing activity is an example of activities
what level of prevention? b. Knowledge deficit relate to anxiety about
a. Tertiary cancer
b. Primary c. Altered health maintenance related to lack of
c. Quarterly information about breast self-examination
d. Secondary d. Knowledge deficit related to inability to
perform breast self examination
49. Aling Maria’s age and her lack of regular
exercise increased her vulnerably to 2. June, another patient in the ward, is a post-
hypertension. These and other variables that abdominal patient. She will be discharged in 2-
increase her vulnerability to hypertension is: 3 days from now. Which behavior objectives
a. life-style determinant would be most accurate?
b. Negative health behavior a. Client will demonstrate abdominal dressing
c. Illness behavior change
d. Risk factor b. Client will change abdominal dressing using
clean technique before hospital discharge
50. One program that Aling Maria could be c. Client will know the correct technique for
encouraged to actively involve with is to abdominal dressing change prior to hospital
improve her present level of illness and discharge
decrease the risk of hypertension is: d. Client will describe how to change the
a. weight reduction abdominal dressing.
b. smoking cessation
c. mental hygiene 3. Miss Golez, another patient, has been
d. physical fitness confined to bed for several days. She needs to
learn to perform back-strengthening exercise.
The best teaching method that you will suggest 10. Nursing process is a problem solving
to your staff nurse for this patient is: technique that utilizes which of the following
a. group discussion sequential order?
b. formal lecture a. Planning, assessment, implementation and
c. individual study evaluation
d. demonstration b. Assessment, implementation, planning and
evaluation
4. You will tell your staff nurse that the most c. Problem identification, planning,
important factors to consider when developing implementation and evaluation
a teaching plan for a child is: d. Planning, implementation, assessment and
a. the chronological age of the child evaluation
b. the information to be taught
c. the amount of time available 11. Which of the following is included in a
d. the developmental level of the child psychosocial assessment of patients?
a. Doing the examination using the
Situation 2: Nursing as a profession benefits cephalocaudal approach
from the use of nursing diagnosis process in a b. Gathering the information on the personal,
professional clinical practice. social and environmental history of the patient
5. The most important values of the use of c. Getting the general data and chief
nursing diagnosis in care planning is: complaints of her patients
a. Highlights your critical thinking and d. Obtaining the past medical history of her
decision-making patient
b. Focuses on the cause of the illness
c. Helps to define what is health 12. Which of the following is the main purpose
d. Identifies what the nurse can prescribe of doing psychosocial and physical
6. Which statement best describe the purpose assessment?
of the nursing diagnoses? a. Formulate a nursing diagnosis for the plan of
a. Identification of problem areas care
b. Specification of patient’s health care needs b. Identify past potential and probable health
c. Organization of assessment data problems of the patient
d. Preparation of a clinical abstract c. Ascertain the reason why the patient is
brought to the hospital
7. The nursing diagnosis of your patients d. Know the patient and his family at the time
consists of statements of: of admission
a. Health problems
b. Medical impression 13. Which of the following information about
c. Response to illness patient care is provided by evaluation as part
d. Alteration of health of the nursing process?
a. The patient has agreed with the process
8. Nursing diagnosis as differentiated from b. Nursing assessment was properly done
medical diagnosis has its emphasis on: c. Nursing goal is met through the use of
a. Patient’s response to illness certain nursing interventions
b. Cause of illness d. The competence of the nurse is determined
c. Specific disease process in managing patient care
d. Pathology
14. The general appearance and emotional
9. These statements about nursing diagnosis disposition of a client is best observed initially
are true, except: Nursing diagnosis during which of the following situations?
a. States etiology of the problem a. Taking of vital signs
b. Is disease-oriented b. Interview
c. Is guided by independent nursing action c. Implementation of initial care
d. Is complimentary to medical diagnosis d. Actual physical examination

Situation 3: The scope of nursing practice Situation 4: The final activity in the process of
provides that the nurse render nursing care to diagnosing is the formulation of the nursing
the client through the application of nursing diagnosis. It is the problem the nurse chooses
process. to address within the scope of the legal nursing
practice.
15. It refers to a problem that exists at the 22. Which of the following is the process
present moment element as framework of nursing care?
a. Actual nursing diagnosis a. Outcome changes
b. Risk nursing diagnosis b. Determining nursing goals
c. Possible nursing diagnosis c. Organizational structure
d. Collaborative problems d. Nursing objectives

16. It is clinical judgment that an individual, 23. Which of the following is the outcome
family or community is more vulnerable to element of nursing care?
develop the problem than are others in the a. Success of nursing action
same similar situation: b. Budget and equipment
a. Actual nursing diagnosis c. Coordination of services
b. Risk nursing diagnosis d. Skill level of client
c. Possible nursing diagnosis
d. Collaborative problems 24. What is the measure of nursing
effectiveness if quality assurance if
17. This is a tentative nursing diagnosis which recommended?
may be compared to a physician who lists a. Standard of practice
several rules out medical diagnosis in a b. Change in behavior of the client
patient’s admission assessment. The nurse c. High cost of health care regimen
assures continued collection of relevant data. d. Nurse-patient ratio
a. Actual nursing diagnosis
b. Risk nursing diagnosis Situation 6: The application of the nursing
c. Possible nursing diagnosis process is a rational method of planning and
d. Collaborative problems providing nursing care. As a basic tool in
professional nursing practice, its utilization
18. Which of the following is the correct way in ensures competent and safe practice.
writing an actual nursing diagnosis: 25. When promoting health of the individual,
a. Problem + risk factors family and community, the first step you
b. Problem + risk factors + signs and should take is:
symptoms a. Do health status assessment
c. Problem + etiology + signs and symptoms b. Conduct a population census
d. Problem + signs and symptoms + etiology c. Establish a registry of families
d. Organize the community for health action
19. What particular nursing diagnosis focuses
on well patients 26. When assessing the family health status,
a. Actual nursing diagnosis one most important data to collect is the:
b. Risk nursing diagnosis a. Roles of the family members
c. Potential nursing diagnosis b. Level of immunization of the children
d. Wellness nursing diagnosis c. Hospitalization experience of the family
d. Health resources of the family
Situation 5: Evaluation of care is an integral
part in the application of Nursing Process. 27. This is a statement of nursing diagnosis
related to health promotion:
20. What are the changes in the client’s health a. Health-seeking behaviors related to
status which is the result of nursing ignorance
intervention? b. Potential coping deficit because of poverty
a. Healthy lifestyle c. Misconception about health promotion
b. Desired outcome attributed to culture
c. Modified behavior d. Malnutrition due to poor eating habits
d. Compliance with regimen
28. In health promotion, which of the following
21. Which of the following is the structural nursing interventions should you give priority:
element from which nursing care is delivered? a. Adult education
a. Philosophy of care b. Health education
b. Health team c. Primary health care
c. Care regimen d. Program planning
d. Modification of symptoms
29. When implementing a health promotion
plan, it is most important to develop the 35. A nurse who is taking a patient with severe
individual’s dehydration due to loose bowel movement
a. Self-respect decided to first provide the patient with liquids
b. Self-responsibility and juices and give health instruction on
c. Self-worth sanitation and hygiene. The nurse in doing
d. Self-image these activities is performing what element in
the nursing process?
30. Nursing process is a problem solving a. Implementation
methodology of the nursing profession. It is b. Evaluation
the manner of: c. Planning
a. establishing a trusting and productive d. Assessment
relationship
b. determining the outcome or result of action 36. The comprehension and interpretation of
taken patient’s data to identify possible nursing
c. relating, assessing, planning and problems is processed through:
implementing and evaluating a. Assessment
d. Both a and b b. Evaluation
c. Analysis
31. Implementation of plan action is done d. Nursing Diagnosis
through
a. Independent and interdependent measure 37. In taking care of the patient for cardiac
b. Dependent and independent measure catheterization, the nurse noted that the
c. Dependent and interdependent measure patient manifested fear related to cardiac
d. none of the above catheterization and its outcome. This
statement is an example of:
32. Which of the following characteristics of the a. Nursing diagnosis
client goal in the plan of care is correct? b. Implementation
a. Nurse-focused, flexible, measurable and c. Assessment
realistic d. Evaluation
b. Client-focused, flexible, realistic and
measurable Situation: As a community health nurse, you
c. Nurse-focused, time-limited, realistic and are planning, carrying out and evaluating care
measurable of patients, particularly those who cannot be
d. Client-focused, time-limited, realistic and attended by the Rural Health Midwife.
measurable 1. In order to provide individualized care to
your patients, you must develop for each
patient a:
Situation 7: During the orientation program for a. List of factors affecting patient’s condition
new staff nurses, one of the first things b. Patient’s needs
emphasized is the application of nursing c. Doctor’s order
process in the management and care of thin d. Family’s instruction
patients.
2. The provision of immediate care needed by
33. The establishment of a data base for a the patient is your concern. Hence, you
newly-admitted patient forms which phase of establish priorities according to:
the nursing process? a. List of factors affecting patient’s condition
a. Assessment b. Patient’s needs
b. Implementation c. Doctor’s order
c. Planning d. Family’s instruction
d. Nursing Diagnosis
3. For the nursing care plan to be effective,
34. Assessment phase of the nursing process is this must be prepared with and accepted by
completed: the:
a. At the beginning of every shift a. Rural Health Physician
b. In the terminating phase of nurse-patient b. Other members of the health team
relationship c. Patient
c. In the initial nurse-patient relationship d. Midwife assigned to the area where patient
d. At the beginning and end of every shift resides
3. Mr. Ocampo appears quite thin. What data
4. Having in mind Maslow’s hierarchy of needs, would you initially need to determine his level
which of the following problems should be of nutrition?
given first priority? a. Nutrition notes by dietitician
a. Patient has difficulty of urination b. A sample of his daily diet
b. Patient has poor and demeaning concept of c. Food he ate yesterday
self d. His height and weight
c. Patient expresses need to get a job
d. Patient feels unsafe and insecure for being 4. Your admission data will include social data
left alone about your patient. These are examples of
social data, except:
5. In order to evaluate the effectiveness of a. Perception of illness
your care plan and have a basis for change or b. Religious practices
modification, you should: c. Family home situation
a. Have a well-documented assessment d. Patient’s lifestyle
b. Have a specific and measurable outcome
criteria 5. Which of the following about nursing
c. Establish a nursing diagnosis diagnosis is true?
d. Have a list of nursing interventions based on a. Assessment ends with a diagnosis
needs b. Assessment data is recorded in SOAP
c. Nursing audit is part of assessment
d. Assessment is a continuous process

B. BASIC NURSING SKILLS 2. VITAL SIGNS


Situation: Mr. Jocson, 65 years old, complained
1. ADMISSION AND DISCHARGE of feeling extremely cold on a hot summer day.
1. You understand that elderlies are at risk of
1. The nurse is performing an admission hypothermia because of:
assessment on a client with exacerbation of a. Lack of exercise
asthma. Which of the following times is ideal to b. Inadequate food intake
design discharge planning? c. Reduced sex hormones
a. At the time of admission d. Reduced heat regulation efficiency
b. The day before discharge
c. After acute episode is resolved 2. To take his temperature accurately, let the
d. When the discharge order is written thermometer stay:
a. In the axilla for no time limit
Situation: As an RN, you assume responsibility b. In the rectum, mouth or axilla at least one
for systematically assessing needs of your or two minutes
clients/patients. You admitted Mr. Ocampo to c. In the rectum for at least five minutes
the ward. d. In the mouth or at least one minute

1. In your initial contact with Mr. Ocampo, 3. The next day, he felt extremely warm. You
which of the following greetings will more likely put on the electric fan to increase heat loss
put him at ease? through:
a. “Good morning. I am Leah, your new a. Conduction
nurse”. b. Convection
b. “How are you today? I am your new nurse.” c. Radiation
c. “Are you Mr. Ocampo, Bed 20”. d. Vaporization
d. “I have been assigned to take care of you”.
4. Mr. Jocson is experiencing a fever of 39
2. In your initial and ongoing assessment, you degrees Celsius. In Farenheit, this is equivalent
utilize the: to:
a. Problem-oriented medical records a. 99.8’F
b. Kardex recording system b. 100.20 ‘F
c. Problem-solving approach to care c. 102.2 ‘F
d. Standards of nursing care assessment d. 104.15 ‘F
5. Two days later, he had fever of 39.5 d. inspection, percussion, palpation and
degrees Celsius. The best nursing measure you auscultation
can immediately give:
a. Increase intake of fluid Situation 2: Mrs. Dela Cruz, 35 years old,
b. Administer antipyretic came to a family planning clinic for
c. Alcohol rub consultation. One of the things that a nurse
d. Tepid sponge bath usually does to assist a patient decide an
appropriate contraceptive method is to perform
3. PHYSICAL EXAMINATION AND HEALTH a physical assessment.
ASSESSMENT
6. The purpose of having another female staff
Situation 1: In doing physical examination, the in the same examination room when a male
patient usually is asked to assume different nurse is assigned to a patient is to:
position depending upon the parts of the body a. Ensure that he nurse performs the physical
to be examined. assessment in an ethical manner
b. Observe the patient’s emotional responses
1. In examining the client’s head, neck and and inform the nurse about it
breath sound, it is best that the nurse position c. Validate or confirm the observation of the
her client to: nurse
a. Lithotomy d. Respond to the patient’s queries so as not to
b. Sim’s position interrupt the nurse
c. Dorsal recumbent
d. Upright position 7. In doing the physical assessment, the nurse
observes the following sequences:
2. The best position for externally examining a. Toe to head
the cardiovascular system uterus and other b. Postero-lateral
organs in the abdominal regions of the client c. Head to toe
is: d. Antero-lateral
a. Supine position
b. Dorsal recumbent 8. To have a comprehensive past medical
c. Lithotomy history, the nurse asks the patient of any
d. Sim’s position significant illnesses that occurred in the past.
The assessment process is called:
3. The best measure to psychologically prepare a. Psychosocial
the client for physical examination is: b. Head to toe
a. explain the procedure and allow her to c. Systems Review
verbalize her discomfort and doubts d. Cephalocaudal
b. tell the clients as to what she can do to
assist in the examination 9. Before measuring the client’s height and
c. avoid unnecessary noise as it will interfere weight, the nurse should:
the examination a. Calibrate the weighing scale by setting the
d. provide screen for privacy weight at zero
b. Ask the patient her height and weight
4. In examining the rectum of a client, the best c. Ask if there has been a change in her height
position is: and weight
a. Sim’s position d. Instruct the patient to step right away on
b. Dorsal recumbent the weighing scale
c. Lithotomy
d. Prone 10. The component that should receive the
highest priority before a physical examination
5. In the physical examination of the patient is the:
with abdominal pain, the following sequence of a. Psychological preparation of the client
technique is recommended: b. Preparation of the environment
a. inspection, auscultation, percussion and c. Physical preparation of the client
palpation d. Preparation of the equipment
b. inspection, palpation, percussion and
auscultation 11. Which of the following is the correct
c. inspection, percussion, auscultation and sequence in abdominal assessment?
palpation
a. Inspection, auscultation, percussion, Situation 4: Nurse Susie is the headnurse in
palpation the Pediatric Unit. She considers history
b. Inspection, palpation, percussion, taking, among others, very vital to the care of
auscultation patients.
c. Auscultation, percussion, palpation, 18. To elicit data gathering about a child,
inspection which of the following should the nurse
d. Inspection, percussion, auscultation, perform first?
palpation a. Assess what a family understands about
their child’s health
12. To ensure pulmonary function, which of the b. Formulate an individual plan of care
following will the nurse do? c. Establish a relationship with the child and
a. Auscultation the family
b. Percussion d. Correct any misinformation the family may
c. I nspection have
d. Palpation
19. Jerry, a 6 month old infant is admitted to
Situation 3: Man is a biopsychosocial being. the unit. The staff nurse takes the history.
Understanding this concept will enable you ton Among others, history-taking in infants focuses
identify the nursing needs of your patient. specifically on:
a. Family background
13. The needs of your patient are those that he b. Stress, prenatal and postnatal history
must have to survive while problems are c. Physical stress
those: d. Parental and sibling relationships
a. Encountered during illness
b. Conditions that can be solved by the nurse 20. Gina was admitted for bodily bruises. As a
c. Barriers to health 2-year old child, the nurse took the standard
d. Situations requiring assistance history taking procedures with specific focus
on:
14. This is an example of a covert need: a. Physical and emotional status of the child
a. Desire for attention b. Religious affiliations and practices
b. Respiratory distress c. Socio-economic status
c. Expressed concern of patient d. Home environment, safety issues
d. Impending shock
21. For school age children, which of the
15. It is important to consider your following aspects in history taking should the
client/patient within his environment in order nurse particularly focus on:
to: a. Church, house, geographical location
a. Know his strengths and weaknesses b. Child’s physical and emotional make-up
b. Correct undesirable health practices c. School, friends, reaction to previous
c. Improve his well-being hospitalizations
d. Identify appropriate nursing intentions d. Financial status of the family

16. When assessing the needs/problems of the 22. Nurse Susie noted a rise in admission
patient, which of the following is paramount. among adolescent group. She cautioned her
His: staff to give special attention in history-taking
a. Financial capacity on:
b. Support system a. Alcohol, drug, friends, sexual history
c. Coping ability b. Food and food stuffs
d. Health resources c. Socio-cultural backgrounds
d. Academic performance
17. The patient views illness as a punishment
form God. How would you respond: Situation 5: Mr. Roy, a 48 years old executive
a. Allow patient to express her hostile feelings was admitted for diagnostic studies. He weighs
b. Encourage patient to express his concept of 210 lbs, leads a sedentary life and is a chain
illness smoker. You studied the assessment of the
c. Explain that her view is incorrect patient.
d. Ignore patient’s view and change the topic. 23. The purpose of your initial physical
assessment is to:
a. Effectively plan nursing care
b. Prepare the patient for diagnostic tests 3. Which of the following statements is a
c. Provide baseline data for further assessment priority health promotion goal for Aling Puring?
d. Encourage nurses to expand their role a. To have regular physical examination
b. To limit activities that tire her
24. In performing physical assessment of the c. To plan her diet
abdomen, the proper sequence is: d. To reduce stress
a. Inspection, palpation, auscultation,
percussion 4. Aling Puring’s blood pressure registered
b. Inspection, auscultation, palpation, 180/110 mmHg. Which of the following factors
percussion would you consider at most risk to her?
c. Palpation, auscultation, inspection, a. Sleeping habits
percussion b. Past illnesses
d. Auscultation, inspection, palpation, c. Daily activities of living
percussion d. Dietary patterns

25. When taking an apical pulse rate, you 5. This is an information which will give you
should place the stethoscope: additional data on Aling Puring’s condition?
a. Between the third and 4th ribs to the left of a. Her health beliefs and practices
the sternum b. Illnesses in the family
b. On the fifth intercostals space along the left c. Demands on her work
clavicular line d. Health resources of her family
c. Between the 6th and 7th ribs
d. Just to the left with the medium part of the Situation: Risk factors can be present in the
sternum following interrelated categories like genetic
and physiological factors, age, physical
26. This is an accurate description of a lymph appearance. Identifying risk factors is
node assessment: important for health and illness prevention.
a. Enlarge
b. Easily palpable 1. Which of the following risk factors is
c. Mobile associated with genetic and physiological
d. Tender nodes factors? a. Poor living condition
b. Poor personal hygiene
27. What confirmation is obtained through c. Obesity and renal disease
percussion of abdominal organs: d. Drug abuse
a. Location, size and density
b. Location and density 2. Which of the following environmental
c. Size and density condition is considered a health risk?
d. Size and location a. Poor personal hygiene
b. Complications of pregnancy
Situation: Aling Puring, 65 years old, a sari- c. Family history of diabetes
sari owner, came to the outpatient clinic d. Chemical or toxic wastes
complaining of easy fatigability. She is very
much worried about her health. You made a 3. Which of the following are internal variables
risk appraisal. that influence health beliefs and practices?
1. Risk appraisal will be more effective if you a. Cultural background and nationality
demonstrate: b. Socio-economic factors and intellectual
a. An attitude of concern background
b. Acceptance of the client c. Spiritual factors and developmental stage
c. A helping attitude
d. Client trust in you 4. When a client relies on a health professional
for relief of his symptoms and accepts care, he
2. The objective of a risk appraisal is to is in which of the following stage of illness?
identify: a. Dependency role
a. Undesirable health practices b. Health care professionals contact
b. Health deficits c. Assumption of the risk role
c. Health hazards d. Recovery and rehabilitation
d. Danger signs of a disease
5. Any variable increasing the vulnerability of
an individual or a group to an illness or Situation 2: Nurses are prone to medication
accident is considered which of the following? errors, this they are advised to be cautious in
a. Negative health behavior their drug administration especially in
b. Risk factor parenteral medicine.
c. Lifestyle determinant 7. In cleansing the site for injection, the nurse
d. Illness behavior should:
a. Swab is used to wipe injection site in a
circular motion moving in an inward direction.
4. ADMINISTRATION OF MEDICATIONS b. Swab is used to wipe injection site on both
sides
Situation 1: You know that age affects drug c. Swab is moved in a circular motion in an
action. Hence, care should be observed in outward direction
administering medications to any age group. d. Any of the above
1. The doctor ordered antibiotics. Why are
antibiotics generally taken after meal? To: 8. The nurse should remember that massaging
a. Delay rate of absorption the injection site is contraindicated.
b. Enhance action of drugs a. After an intradermal injection
c. Prevent irritation of gastric mucosa b. Adminitration of insulin
d. Prevent side effects c. IM injection using a Z-track technique
d. All of the above
2. Central nervous system depressant has
more toxic effects when given to younger 9. The most common site for SQ injection is:
patients primarily due to increased drug: a. Outer thigh
a. Absorption b. Upper arm
b. Excretion c. Abdomen
c. Distribution d. Middle portion of the back
d. Action
10. The amount of medication that could be
3. How do you prevent introduction of an given through a deltoid muscle is:
intramuscular injection into the venous a. 0.5 ml or less
system? b. 1.0 ml or less
a. Inject medication slowly c. 1.5 ml or less
b. Use a Z tract method of injection d. 2.0 ml or less
c. Aspirate drug after insertion of needle
d. Inject in an area with fewer blood vessels 11. The needle appropriate to a z-track
procedure is:
4. A patient said he follows his routine in a. 1-2 inch needle (2.5-5 cm)
taking medication. Which is your best initial b. 2-3 inches needle (5-7.5 cm)
response? c. 0.5-1 inch needle (1.3-2.5 cm)
a. There is a hospital policy to follow d. 3-4 inches needle (7.5-10 cm)
b. Let me refer you to your doctor
c. Ask what those routines are Situation 3: Lyka, a BSN student, was assigned
d. That’s alright for me in the medical ward. She is to administer
medication under the supervision of her clinical
5. When a patient refuses to take his instructor.
medication, what is your initial action: 12. When administering drugs, the nurse
a. Ask the reason why compares the label of the drug container with
b. Give him time to change his mind the medicine card.
c. Your doctor may order your discharge I. Before removing the container from
d. Convince him that medicines are for his the drawer or shelf
welfare II. As the amount of drug ordered is
removed from the container
6. Drug absorption in IM injection is decreased III. Before returning the container to
due to: the storage
a. Fat deposits IV. Before directly administering the
b. Decreased blood supply to the muscle drug
c. Muscles are full of connective tissue
d. Increased blood flow a. I and II only
b. I, II, III and IV Situation 1: In nursing practice, you are
c. I, II and III directly involved in providing a safe
d. I, II and IV environment.
1. A biologically safe environment is free from:
13. What is the best way of identifying the a. Living sources of infection
right client? b. Harmful microorganism
a. Check the medicine card against the client’s c. Dirty treatment instruments
identification bracelet? d. Corrosive agents to body tissue
b. Nurse speaks the name of the client
c. Nurses ask the client to state full name 2. What is the main goal of hospital against
d. Nurses consult the physician infection?
a. Interrupt bacterial growth
14. If the doctor’s order 8h, what does it b. Avoidance of cross infection
mean? c. Break the chain of infection
a. The medication should be given during the d. Strict observance of asepsis
waking hours
b. The medication should be given round the 3. This measure will prevent the spread of
clock infection in hospital wards:
c. Both a and b a. Increase knowledge of staff on risk factors
d. None of the above b. Strict observance of asepsis
c. Disinfection of known contaminated
15. When in doubt about drug names, what is instruments
the BEST nursing action? d. Prompt report of signs of infection
a. Read the patient’s chart
b. Ask the attending physician 4. The most common infectious agent in the
c. Ask another nurse patient’s unit is:
a. Nosocomial infection
16. The correct technique in Z-tract injection of b. Escherichia coli
iron is: c. Virulent microorganisms
a. Pull the skin at site of injection down d. Iatrogenic agents
b. Inject deep IM
c. Massage the skin 5. Your patient complained for severe pain at
d. Pull the skin tight the inflamed wound and asked you why. Your
correct response is:
17. A nurse supervisor observed the newly a. Injury to local nerve endings
hired nurse in the injection of a parenteral b. Increase permeability of injured capillaries
medication of the buttocks of the patient. The c. Pain stimulant at inflamed site
newly hired nurse needs no further teaching if d. Pressure of accumulating fluids at inflamed
she: site
a. Injects at the inner upper quadrant
b. Draws an imaginary line at the injection site 6. Which of the following interventions would
c. Pinches the skin before injecting be appropriate before touching Mrs. B’s
d. Inject medication at 45 degrees dressing?
a. Obtain a physician’s order to remove
18. After an injection on buttocks of the bandage
patient, the patient complained of a paralysis- b. Ensure that povidone-iodine solution is
like feeling on his leg. The patient asks you if available for cleaning the wound
this could be caused by injection. What would c. Check the patient’s peripheral pulses
be your response? d. Put on sterile gloves
a. Say Yes
b. Say No 7. All the following statements are true about
c. Say perhaps donning sterile gloves except:
d. Say it needs assessment a. The first glove should be picked up by
grasping the inside of the cuff
b. The second glove should be picked up by
5. ASEPSIS AND INFECTION CONTROL inserting the gloved fingers under the cuff and
pulling the glove.
c. The glove should be adjusted aiding the
gloved fingers under the sterile cuff and pulling 15. What is the term which means there is an
the glove under the wrist. increase susceptibility to special substances
d. The inside of the glove is considered sterile. especially protein. When protein is introduced
to the body, hypersensitivity is alarming:
8. Effective handwashing requires the use of a. Anaphylaxis
a. Soap or detergent to promote emulsification b. Neurogenis
b. Hot water to destroy bacteria c. Serum sickness
c. A disinfectant to increase surface tension d. Antitoxins
d. All of the above
16. Before introduction of any foreign
9. Which of the following procedures always substance to the body, it is the duty of the
require surgical asepsis? nurse to prepare a syringe with adrenalin or
a. Vaginal installation of conjugated estrogen aminophylline to counteract allergies or
b. Urinary catheterization anaphylactic shock. What do you call this
c. Nasogastric tube insertion procedure?
d. Colostomy irrigation a. Horse-serum
b. Antidote
10. The nurse is aware that transmission of c. Sensitivity
microorganisms can be primarily prevented by: d. Dermatologic
a. Handwashing
b. Sterilization Situation 3: The nurse plays an important role
c. Disinfection in the control of preventable diseases.
d. Fumigation 17. The control of any preventable disease
depends to a large extent on the following,
11. You are caring for a client in isolation. After except:
nursing care is done, what will you remove first a. Assignment of the epidemiologist
after handling the patient? b. Nature of the disease itself
a. Mask c. Application of effective control measures
b. Gown d. Knowledge of appropriate technology
c. Gloves
d. Bonnet 18. The first line of defense of the body against
disease is:
Situation 2: Immunity maybe natural or a. An intact skin
acquired but there is not absolute immunity. b. Personal hygiene
12. What kind of immunity is a state of c. Complete immunization
resistance to infection inherent in the body not d. Physical stamina
brought about through a previous attack of the
disease? 19. When breaking the chain of infection,
a. Passive which of the following is true:
b. Natural a. A virulent agent can resist disinfection
c. Anaphylactic b. An appropriate portal of entry will not cause
d. Active infection
c. Mode of entry of harmful organisms is direct
13. This kind of immunity is acquired resulting entry
from previous attack of the disease or repeated d. Every link in the chain of infection can be
exposure to doses of an organism? interrupted
a. Anaphylactic
b. Active 20. Why do newborn babies have a limited
c. Passive ability to produce antibodies against infection?
d. Natural a. Their immune system is still undeveloped
b. All babies have antibodies from their
14. What kind of immunity is dependent on the mothers
presence of the blood of immune substances c. Babies are less exposed to harmful
not produced by the person’s own body cells? microorganisms
a. Passive d. Skin of babies are quite intact
b. Active
c. Natural 21. Health education is a vital component in
d. Prophylactic the control of diseases. Its major aim is to:
a. Modify risk behavior c. Makes the microorganisms less
b. Helps individual protect himself contaminated
c. Reduce risk of exposure d. Rapidly kills viruses causing AIDS or
d. Avoid unnecessary stress Hepatitis B

Situation 4: You have just attended training on


family planning. One of the important things
you have learned as a family planning service 6. FIRST AID MEASURES
provider is infection prevention.
22. To achieve high-level disinfections by
boiling, which of the following should NOT be
practiced? 7. WOUND CARE
a. Completely submerge in clean water Situation 1: Mr. Alfred, a public school teacher
b. Start timing when the stove is turned on is walking towards school when a stray bullet
c. Start timing when boiling start hit him. He is rushed to the Provincial Hospital
d. Boil the instrument for at least 20 minutes where an exploratory laparotomy is ordered.
while keeping the lid covered 1. In changing the wound dressing a day after
the operation, the nurse can BEST prevent
23. Immediately after pelvic examination of a spread of infection by discarding the soiled
client, you planned to sterilize the gloves and dressing in which of the following?
the speculum for next day’s use. Your first step a. The garbage can in the corridor
should be which one of the following? b. The garbage can in Mr. Alfred’s room
a. Clean them with soap and water to remove c. In an improvised paper bag then discard in
visible dirt the yellow marked garbage can
b. Set them aside and boil them in the morning d. In a plastic bag then discard in the green
for 30 minutes marked garbage can
c. Boil them immediately after use for 30
minutes 2. Which of the following is the nurse’s MAIN
d. Soak them in a 0.5% chlorine bleach priority when changing the dressing?
solution for 10 minutes before cleaning a. Wear sterile gloves when removing the
dressing
24. In which of the following situations is b. With the same sterile gloves in removing,
aseptic techniques NOT necessary? apply the dressing
a. When there is a gross examination of blood c. Wash hands after the dressing change
and other secretions d. Wash hands before and after the dressing
b. When there is contact with a client who has change
communicable disease
c. When there is contact with any client with or 3. When charting the nurses notes after a
without communicable diseases dressing change, which of the following is NOT
d. When there is contact with the belongings of included?
a client who has communicable disease a. Amount, odor and appearance of drainage
b. Quantity of cotton balls used in changing the
25. A client asks you what initially is the wound
simplest and most important procedure being c. Appearance of the skin around the wound
done in a clinic so she is not infected with d. Time dressing change is done
other client’s diseases. Which one of the
following is your answer? 4. Which of the following is the appropriate
a. Periodic fumigation of the health center action when assisting Mr. Alfred with the
b. Sterilization of all instruments postoperative coughing and deep breathing
c. Handwashing before and after examination exercises?
of every client a. Place the patient in dorsal recumbent
position during coughing and deep breathing
26. Decontamination of surgical instruments by exercises
soaking in 0.5% chlorine bleach for minutes b. Administer prescribed analgesics before the
results to which of the following? coughing and deep breathing exercises
a. Enables the health workers to reuse the c. Instruct Mr. Alfred to cough every 3 deep
instrument immediately breathing
b. Completely kill all microorganisms d. Administer prescribed analgesics after the
coughing and deep breathing exercises
5. Which of the following is BEST effective skin 1. As part of the physical per-operative
disinfection before a surgical procedure? preparation, the doctor ordered: Enema until
a. Shaving the surgical site in the day before return flow is clear. The safest solution to use
the surgery for repeated enemas would be:
b. Apply on antiseptic solution on the surgical a. Sterile water
site in the morning before the surgery b. Tap water
c. Have the patient take a bath with an c. Soap sud
antiseptic soap in the evening before and d. saline
morning of surgery
d. Have the patient take a bath in the morning 2. What height would be appropriate for
of the surgery elevation of the enema bag for a cleansing
enema?
Situation 2: You took over the nursing care of a. slightly below the hips
a patient with an infected wound. b.12-18 inches above the hips
6. When body calls are responsible for tissue c. less than 12 inches above the hips
repair: d. 18-24 inches above the hips
a. Macrophages
b. Neutrophils 3. What is the best position of the patient
c. Platelets during an enema?
d. Epithelial cells a. Lithotomy
b. Right lateral Sim’s position
7. The best solution to use in cleaning c. Dorsal recumbent
decubitus ulcer is: d. Left lateral Sim’s position
a. Isotonic saline
b. Sterile water 4. After 2 days postoperatively, Sam complains
c. Betadine of abdominal pain and shortness of breath. The
d. Hydrogen peroxide nurse observes abdominal distention. The most
likely cause of Sam’s current condition is:
8. Which of the following statements on wound a. Flatulence
healing is not true: b. Constipation
a. Wounds heal faster in one patient c. Slowed esophageal emptying
b. Stress interferes with wound healing d. Increased peristalsis
c. Wound healing is delayed in malnourished
patients 5. After 5 minutes while giving enema, Sam
d. Ischemia slows down wound healing complains of abdominal cramping. The most
appropriate action to take is:
9. When dressing contaminated wounds, you a. Clamp the tubing temporarily
must keep in mind the following, except: b. Encourage the client to change position
a. Apply cold around wound area after dressing c. Remove the rectal tube
b. Disinfect all instruments used d. Take no action as this is expected
c. Use a non-irritating disinfectant solution
d. Isolate patient from those with clean
wounds Situation 2: Normita is a staff nurse of the OR
6. As a scrub nurse, your duties before the
10. The development of an infection depends surgeon arrive in the filed area are the
primarily on the relationship between: following, except:
a. Harmful and harmless microorganism a. Complete the scrub
b. Numerous body defenses b. Drape and set a small table for the patient’s
c. Body resistance to disease and good skin preparation
nutrition c. Gown and glove
d. Agent, host and environment d. Open and prepare a disposable or
prepackaged prep tray

8. PERIOPERATIVE CARE 7. In preparation for operation, which of these


Situation 1: Sam, 25-year-old was admitted in should not be included in the Mayo table?
the surgical ward after diagnosed with acute a. Towel forceps
appendicitis. He is scheduled for a STAT b. Smooth tissue forceps
operation. c. Retractors
d. Toothed tissue forceps d. In the same syringe drawing up atropine
and Demerol
8. Lea is for perineal repair. The rationale for
wrapping her legs in the OR with an elastic 15. If Mr. Domingo were given spinal
bandage is: anesthesia. You would expect him to have
a. To prevent emboli formation hypotension because there is:
b. To avoid chilly sensation a. Decreased strength of cardiac contraction
c. To protect her legs from injury b. Dilatation of blood vessels
d. To prevent mesh c. Loss of blood
d. Unstable blood pressure
9. When raising the legs on the stirrups, you
should:
a. Raise legs simultaneously 29. After an intravenous pyelography, the
b. Raise legs simultaneously with the help of patient was not able to void. Based on
another person Maslow’s Hierarchy of needs, this is:
c. Raise legs carefully one after the other a. Love and security
d. Raise legs carefully with the help of another b. Physiologic need
person c. Self-actualization
d. Self-esteem
10. Joe is for nephrectomy. You would position
him in the OR table in: 30. A post-appendectomy patient was
a. Right lateral endorsed to the recovery room. As a nurse in
b. Modified recumbent the recovery room what will you include in
c. Jack Knife your nursing care plan?
d. Reverse Trendelenburg a. Orient the patient as to the time, place and
person
Situation 3: Mrs. Domingo, a newly operated b. Ask the patient where he is
patient was just wheeled in from the operating c. Let the patient sleep and rest
room still in an unconscious state. d. Monitor vital signs every 2 hours
11. During the immediate postoperative
period, you should give high priority to: 31. After a modified radical mastectomy, the
a. Observing for hemorrhage nurse would correctly:
b. Checking vital signs every 15 minutes a. Raise the arm of the affected side with a
c. Monitor patent airway pillow under
d. Recording intake and output b. Begin active exercise on the affected side
c. Instruct the patient not to move the affected
12. You refer postoperative patient under arm for at least 24 hours
general anesthesia to the doctor when he has: d. Give analgesics as necessary
a. Cold clammy skin and filiform pulse
b. Snoring respiration and rapid pulse
c. Accidental removal of the airway Situation: John had a lobectomy for cancer of
d. A drop in blood pressure and rapid pulse the left lobe of the lungs. He is 18 hours post-
operative when you start caring for him.
13. How do you move the penlight when 1. Which of the following positions would be
testing the papillary reflex: indicated for John?
a. All direction a. Turn to operative side
b. From side to side b. Flat on his back
c. Up and down c. Semi-Fowler’s position when turning to
d. IN a circular either side
d. Turn to operative side
14. How do you administer a postoperative
order of Demerol 50 mg and Atropine gr. 2. John is complaining of severe pain on
1/150 intramuscularly? inspiration. Which one of the following drugs
a. In separate syringes without saline solution would probably be prescribed to relieve pain?
b. On the same syringe drawing up Demerol a. Empirin compound
then atropine b. Codeine sulfate
c. In separate syringes both diluted in saline c. Morphine sulfate
solution d. Demerol HCL
3. Which one of the following nursing c. Pulmonary venules
interventions can least control post-operative d. Pulmonary artery
pain for John?
a. Administering pain reliever for at least every 3. One physiological factor influencing tissue
4 hours for the first 24 hours oxygen among premature infants is:
b. Keeping bed linens free of wrinkling a. Teething process and increase nasal
c. Position client for at least every four hours congestion
at a time b. Surfactant deficiency
d. Keeping the bladder empty c. Airway obstruction from foreign object
d. Decreased lung elasticity
4. After 24 hours, you planned to assist John
to ambulate. It has been four hours since his 4. The mechanism of oxygenation is affected in
last pain medication and he refuses to move patients with cardiovascular accident because
considering his condition. Which one of the of:
following is preferred nursing action? a. Decreased cardiac output
a. Give the medication and complete bed rest b. Withdraw metabolic rate
from nursing activities before ambulating him. c. Altered central nervous system function
b. Give John the medication and assist him to d. Decreased oxygen capacity of the blood
ambulate immediately after
c. Give the pain medication and wait for 30 5. An abdominal condition in which the person
minutes to ambulate him must sit, stand or use multiple pillows to
d. Straighten his legs to reduce strain breath when lying down is:
a. Orthopnea
5. Following oral Demerol, John vomits and b. Dyspnea
states that he feels like vomiting again. Which c. Eupnea
of the following nursing interventions would d. Apnea
you not continue or complete?
a. Administer Demerol intramuscularly (IM) to 6. The best position for dyspneic client is:
relieve pain a. Orthopneic position
b. Splint the incisional area b. Semi-Fowler’s
c. Offer carbonated uncola beverages if not on c. Fowler’s
NPO temporarily d. Supine
d. Have client take slow deep breaths
38. In assisting a client in the insertion of a
9. POSTMORTEM CARE nasal cannula, the following are appropriate
nursing interventions:
a. Do not allow the patient to use razor
b. Measure from the tip of the nose to earlobe
and to the xiphoid process
c. Apply a mineral oil to the tip of the cannula
to aid in insertion
d. Position the client from semi-Fowler’s
C. MEASURES TO MEET PHYSIOLOGICAL position
NEEDS
2. NUTRITION
1. OXYGENATION Situation 1: In order to measure nutritional
Situation 1: Oxygen is one of the vital needs of needs of the patients, nurses must have a very
man. good background of the digestive and
1. Which of the following factors is required for metabolic processes of the body.
oxygen transport and delivery? 1. The major portion of digestion occurs in
a. Digestion and skin excretion which of the following parts?
b. Fibrillation and tachycardia a. Large intestine
c. Diffusion and ventilation b. Mouth
d. Inhalation and exhalation c. Stomach
d. Small intestine
2. The exchange of gases in pulmonary
circulation results in the: 2. Which of the following primary nutrient is
a. Pulmonary capillary bed absorbed in the large intestine?
b. Pulmonary veins a. Bile
b. Water
c. Iron 10. What mineral works with Vitamin E to
d. Vitamins protect the body compounds form oxidation?
a. Sodium
3. Which of the following metabolic processes b. Selenium
is responsible for the breakdown of chemical c. Zinc
substances into simpler substances? d. Manganese
a. Anabolism
b. Digestion Situation 3: Osteoporosis is considered a major
c. Catabolism helath and economic problem. One in every
d. Diffusion four woman over age 50 is believed to have
this disease. As nurses we neeed to educate
4. A fully breast-fed infant is one who is given: people about it since according to the 1994
a. Breast milk plus 2 tablespoon of calamansi survey, only 4 out of 10 Filipinos are aware of
juice daily after one month the disease. Among this 4 misconception are
b. Breast milk plus sterile water in between common.
feeding
c. Breast milk only 11. What is the main reason why Asian women
d. Breast milk plus 4 ounces of cerelac daily are more prone on developing osteoporosis
after the third month than African-American women?
a. There are more post menopausal aged Asian
5. At what age are solid food best introduced in women
a fully breast-fed infant? b. Bones of Asian women are 30-40% less
a. 9 months dense than that of African-American women
b. 6 months c. African-American women eat more
c. 3 months compared to Asian women
d. 12 months d. Asian women have lesser bone mass
compared to African-American women
Situation 2: The 2000 Nutritional Guidelines is
formulated to improve the nutritional stayus of 12. There are factors that predispose an
the Filipinos. The following questions are individual for osteoporosis. What factor is least
concerned with nutrition. associated with the disease?
a. decrease level of estrogen
6. Which one is not a function of calcium? b. calcium deficiency
a. Bone and teeth mineralization c. exposure to sunlight
b. Absorption of iron and formation of d. lack of exercise
hemoglobin
c. Blood clotting 13. When is the peak bone mass or PBM
d. Muscle relaxation and contraction achieved?
a. 0-10 years old
7. Xerophthalmia is characterized by b. 10-20 years old
a. Tunnel vision c. 20-40 years old
b. Floaters d. 40-60 years old
c. Night blindness
d. Window vision 14. Women over 50 years old are very much at
risk for fractures. Where is the most common
8. A deficiency in protein leads to site of fractures secondary to osteoporosis?
a. Kwashiorkor a. femur
b. Rickets b. forearm
c. Beri-beri c. vertebra
d. Hemorrhage d. hips

9. Nutrients are classified according to their Situation 4: Mrs. M has been given a copy of
structures. Which among these is not a her diet. He consulted you about his prescribed
micronutrient? diet.
a. Vitamin A 15. You discuss the food allowed on 500 mg
b. Iron low sodium diet. Which of the following is
c. Iodine allowed on a 500mg diet?
d. Amino acid
a. A ham and swiss cheese on sandwich on 23. Mr. K is placed in a sodium free diet
whole wheat bread because of his hypertension. An example of
b. Mashed potatoes and broiled chicken sodium-free meal is:
c. A tossed salad without vinegar and olives a. Sodium-free soup, 3 oz of baked chicken,
d. Chicken bouillon plain potatoes, plain string beans, a salad with
oil and vinegar , a roll with margarine, one
16. Mrs. M asks you about niacin. The major small apple and coffee
food source of niacin (nicotinic acid) is b. Split pea soup, a ham sandwich, potato
a. meat chips, 4 oz of milk, gelatin and decaffeinated
b. oats coffee
c. rice c. Bouillon, 3 oz of turkey, sweet potatoes,
d. corn beans, two slices of white bread with
margarine and custard
17. Which of the following diseases results d. Cheese omelet, orange juice, sodium free
from niacin deficiency? roll with margarine, gelatin and tea
a. diabetes
b. beriberi 24. Which of the following fluids is
c. pellagra contraindicated before sleep?
d. megaloblastic anemia a. Hot chocolate
b. Coffee or tea
18. He also ask about Vitamin E. The richest c. Cola and root beer
source of Vitamin E is: d. All of the above
a. vegetable oil
b. milk Situation 6: Mrs. Opal Quinto, 40 years old, is
c. fish brought to the hospital with complaints of
d. cereals nausea, vomiting and severe epigastric pain.
Initial care includes insertion of a nasogastric
19. The most common vitamin deficiency seen tube. (NGT)
in alcoholics is:
a. thiamine 25. Which of the following steps should the
b. riboflavin nurse take first when preparing to insert a
c. pyridoxine nasagastric tube?
d. panthothenic acid a. Wash hands
b. Apply sterile gloves
20. Which of the following vitamins is known c. Apply a mask and gown
for its blood clotting effect? d. Open all necessary kits and tubing
a. Vitamin D (calciferol)
b. Vitamin E (tocopherol) 26. The nurse on duty is inserting a
c. Vitamin C (ascorbic acid) nasogastric tube, Mr. Recto begins to gag.
d. Vitamin K Which actions should the nurse take?
a. Remove the inserted tube and notify the
Situation 5: Mr. K, a patient in MS ward, wants physician
to increase his Vitamin C intake. He also ask b. Stop the insertion, allow the client to rest,
you several questions about nutrition. then continue inserting the tube
21. Which of the following foods is not well- c. Encourage the client to take deep breaths
known source of Vitamin C? through the mouth while the tube is being
a. tomatoes inserted
b. grapeftuit d. Document in the chart the insertion, method
c. green and yellow vegetables used to check tube placement and clients
d. oranges
27. Which of the following steps, if taken by
22. Vitamin C deficiency is characterized by the the nurse after insertion of nasogastric tube,
following except could harm Mr. Recto?
a. redness and edema of the gums a. Affix the nasogastric tube to the nose with
b. pinpoint peripheral hemorrhages tape
c. pernicious anemia b. Check tube placement by aspirating
d. bones that fracture easily stomach contents using a piston syringe
c. Check tube placement by installing 100 ml 5. Mrs. Caro reports a loss of gas in her
of water into the tube to check for stomach colostomy bag. Which of the following is
filling correct?
d. Document in the chart, the insertion, a. Burp the bag
method used to check tube placement and b. Eat less beans
client response to the procedure. c. Replace the bag
d. Put a tiny hole in the top of the bag
28. Which of the following drug forms can be
administered through a nasogastric tube? 6. Mrs. Caro asks a nurse how to avoid leakage
a. enteric coated from the colostomy bag. Which of the following
b. oral instructions is correct?
c. parenteral a. Limit fluid intake
d. sublingual b. Eat more fruits and vegetables
c. Empty the bag when it is half full
Situation 7: Mrs. Caro, 72 years old, is d. Tape the end of the bag to the surrounding
admitted to the hospital with an intestinal skin
obstruction due to acute diverticulitis. She
undergoes emergency surgery and a double
barrel colostomy.
29. Four days after the surgery, Mrs. Caro is Situation: Mr. Yoyo, 75 years old, has
having trouble adjusting to it. Which of the swallowing problems due to a recurrent cancer
following is the most common? in his throat. A gastrostomy operation is
a. Anxiety performed.
b. Low self esteem
c. Alteration in comfort 1. Gastrostomy is performed by:
d. Alteration in body image a. Relieve abdominal distention
b. Avoid nasogastric tube
2. A nurse approaches Mrs. Caro for a routine c. Administer food through stomach
assessment and finds her tearful. Which of the d. Improve nutrition
following actions is appropriate?
a. State she’ll come back another time 2. What is the advantage of gastrostomy from
b. Ask the client if she’s having pain or nasogastric tube feeding:
discomfort a. Easier to perform the procedure
c. Tell the client she needs to perform b. Cardioesophageal sphincter remains intact
assessment c. No regurgitation of food
d. Sit down with the client and ask if she’d like d. More acceptable to the patients
to talk about anything
3. Your most important responsibility is to:
3. After a review of colostomy care, Mrs. Caro a. Observe any signs of infection
says she doesn’t know if she’ll be able to care b. Make patient comfortable
for her self at home without help. Which of the c. Ensure proper food and fluid intake
following nursing interventions is most d. Regularly check status of the tube
appropriate?
a. Review care with the client again 4. The best position to assume during
b. Provide written instructions for the client gastrostomy feeding is:
c. Ask the client if there is anyone who can a. Supine position
help b. As ordered by the doctor
d. Arrange for home health care to visit the c. Sideways
client d. In back rest

4. Mrs. Caro is experiencing mild diarrhea 5. The first sign that gastrostomy feeding is
through his colostomy. Which of the following working when patient is:
is correct? a. Has fecal movement
a. Eat prunes b. Is comfortable
b. Drink apple juice c. No obstruction in tube
c. Increase lettuce intake d. Abdomen is not distended
d. Increase intake of bananas
Situation: Nutritional requirements for older
adults are planned to counteract age-related 1. A child whose diet is always deficient in
changes. green and yellow vegetables, fats and oils is
1. Why should caloric needs of adults be prone to have:
reduced? a. Protein-energy malnutrition
a. Because of decreased body metabolism b. Conjunctivitis
b. Because of inability to digest food c. Vitamin A deficiency
c. Because of lack of appetite d. Vitamin D deficiency
d. Because of loss of weight
2. Your healthy elderly patient does not have 2. Which of these groups are vulnerable to
good appetite. In your assessment, which is anemia?
not to be expected? a. Adolescents who are underweight
a. Feeling of sadness b. All these groups
b. Denture of problems c. Pregnant women who have several
c. Change in taste and smell pregnancies
d. Unusual loss of weight d. Children who have not been given
supplementary foods
3. Which of the following most likely to
maintain adequate intake of food? 3. In order to improve resistance to infection,
a. Some exercises which of these target groups is given first
b. Taking appetite stimulants priority for supplementation:
c. Well balanced meals a. All 1-4 year old children
d. Frequent small amounts of food intake b. 104 year old moderately underweight
children
4. Laboratory results of blood test showed high c. Pregnant women
cholesterol. The following foods are substitute, d. Lactating women
except:
a. High protein diet 4. Emma, 2 years old, had measles last April
b. Chicken with sauce 3, weeks after her sister Elsa got sick of the
c. High fiber diet same disease. Their brother Leo was exposed
d. Lean meat to measles but did not get sick. What type of
immunity did Emma and Elsa get?
5. You include milk in the diet of the elderly a. Artificially acquired active
because it promotes defecation due to the: b. Naturally acquired passive
a. Direct stimulation of colonic musculature c. Artificially acquired passive
b. Irritating effect of fiber on the bowel wall d. Naturally acquired active
c. Action of milk diet on large intestines
d. Tendency of smooth muscle to contract 5. In view of Leo’s repeated exposure to his
when stretched sisters with measles, he developed which type
of immunity?
7. Which form of medication can be given to a a. Artificially acquired active
client with a nasogastric tube? b. Naturally acquired passive
a. Enteric coated tablet c. Naturally acquired active
b. Oral medications d. Artificially acquired passive
c. Parenteral medications
d. Medications in capsule form Situation: Mrs. Villa, 42 years old, visited the
health center complaining easy fatigability,
13. The proper position to facilitate insertion of irregular meal time and dizziness. She said she
NGT is: has been so busy that she did not find time to
a. Fowler’s consult regarding her ailments.
b. Supine 1. You noted that your patient often leaves
c. Side-lying food untouched or barely eat meals served.
d. Lateral The first step you will take is:
a. Check food preference
b. Determine cause of bad eating habits
Situation: You scheduled a staff development c. Take a nutrition history
session on the Department of Health programs d. Suggest to choose appropriate food
with midwives. The following were
issues/questions discussed.
2. Your patient expressed that she has no b. Many pharmaceuticals supply sample of
appetite for eating. For optimum nutrition, you these
would: c. Because these prevent conditions like caries
a. Feed her to conserve her energy and bone breakage
b. Let her verbalize why she has no appetite d. They act like natural vaccines
c. Ensure that she observes good eating habits
d. Let her participate in the feeding activity 2. If a child has dental caries, he will be prone
to have:
3. What assessment information would be a. Flu
most helpful to help you establish a nursing b. Baldness
diagnosis: c. Fractures
a. Eating habits d. Wounds
b. Nutritional history
c. Occupation 3. For Vitamin C to be easily absorbed by the
d. Dietary pattern child’s gut, what form of presentation is
recommended by the nurse?
4. Her leg cramps may be a probable a. Tablets
symptoms of: b. Liquids
a. Hypocalcemia c. Chewables
b. Hypokalemia d. Injectables
c. Anemi
d. High blood pressure 4. In an array of food spread out on a table, a
child by nature will have servings of?
5. Iron medications were prescribed for the a. Only pick-a-pick choices
patient. This is to: b. Necessary nutrients
a. Cure patient’s anemia c. Nonsense food groups
b. Support red blood cell formation d. Sweets and junk foods
c. Increase RBC count
d. Counteract effects of menopausal Situation: The goal of the 2000 Nutritional
osteoporosis Guidelines for the Philippines is the
improvement of the nutritional status of the
Philippines through desirable dietary practices
23. You are caring for a client with NGT and and healthy lifestyle?
the parent complained of abdominal distention. 1. What are the chemical substitutes that keep
What will be your appropriate nursing the body healthy?
intervention? a. Nutrients
a. Reposition the client b. Macronutrients
b. Flush the tube with normal saline solution c. Micronutrients
c. Clamp the tube d. Vitamins
d. Check the gastric contents if the previous
feeding was absorbed 2. Which of the vitamins below help in the
formation of new cells, maintain nerve cells
39. To ensure that NGT is in place, what is the and assists in the metabolism of fatty acids
best way to determine its placement? and amino acids?
a. Instill 100 ml of saline a. Vitamin B12
b. Aspirate gastric content with bulb syringe b. Vitamin A
c. Place NGT tip in water and observe for c. Macronutrients
bubbling d. Vitamin D
d. Auscultate for breath sounds
3. Which mineral maintains normal fluid and
Situation: Children need adequate intake of electrolytes in the body?
Vitamin C and calcium. The nurse must a. Calcium
emphasize this in the health teaching sessions b. Flouride
to mothers. c. Chloride
1. Why should Vitamin C and calcium rich d. Magnesium
foods occupy a greater part of the child’s daily
menu? 4. Water helps regulate body temperature,
a. Readily available and affordable over the transport nutrients and help in the excretion of
counter
waste products. What is the volume of water 1. To verify the placement of the tube position
required daily? in the stomach:
a. 3-6 glasses a. Suggest a fluoroscopy
b. 4-6 glasses b. Let patient press stomach
c. 5-8 glasses c. Mark tube at the point of exit from the nose
d. 6-8 glasses d. Measure length from nose to approximate
site of stomach
5. Which nursing procedure is vital in relation
to water requirements, which should not be 2. The first consideration in selection of enteral
delegated to non-nursing personnel? formula is:
a. Health teaching about water sanitation a. Dietitian’s recommendation
b. Formula computation of water requirement b. Patient’s nutritional needs
c. Water and electrolyte balancing c. Advice of attending physician
d. Mesurement of intake and output d. Digestibility of food

Situation: Marino, a 35 years old drug 3. This is a measure that can satisfy Noel’s
representative came to the outpatient nutrition desire to taste and chew:
clinic due to a progressive loss of weight. a. Give a bit of the formula before tube feeding
1. A risk for poor nutritional status is one b. Allow to chew food and spit them out
whose weight is: c. Let the patient smell food formula before
a. 5% of normal weight tube feeding
b. an unstable daily weight d. Put formula dish with insight for sometime
c. 5% less of normal weight before feeding
d. unintentional weight loss of 10%
4. The most important goal in tube feeding is:
2. Which information would be most helpful a. To maximize comfort of patient
when assessing whether Mario is eating b. To keep good nutrition during illness
adequately? c. To monitor for tube feeding complications
a. Knowledge of nutrition d. To increase patient mobility
b. 72 hours diet recall
c. What he takes for snacks 5. This is a possible complication of feeding
d. Nutritional history tube insertion.
a. Stomach ulcer
3. What is the body mass index of Marino if his b. Infection
height s 1.7 meters and weighs 69 kilograms? c. Electrolyte imbalance
a. 24 d. Intestinal distension
b. 18
c. 20
d. 22
3. ACTIVITY, REST AND SLEEP
4. You can best assume your role in nutrition Situation: Stress is the single most important
care by: contributing factor to illness. It is related to
a. Preparing therapeutic meals many of man’s illnesses and interventions of
b. Choosing appropriate foods for patient these illnesses are based on the body’s
c. Giving nutritional counseling reaction to stress.
d. Helping patient adjust to changes in diet 1. Which of the following theories explain that
stress is demonstrated by a specific
5. Physiological alteration of nutrition less than psychological reaction without consideration of
body requirements is related to which of the cognitive influences in the person?
following causes: a. Stimulus-based model
a. Too much physical exercises b. Adaptation model
b. Poor eating habits c. Response-based model
c. Knowledge deficit d. Transaction-based model
d. Inability to absorb nutrients
2. Which of the following patient’s behavior is a
Situation: Noel, a 35 years old head injury spiritual indication of stress?
patient with feeding impairment is for a. Restlessness and difficulty of sleeping
nasogastric tube insertion. b. Questions about the Meaning of life
c. Inability to resolve conflict
d. Feelings of inadequacies
3. Which of the following goals of nursing 5. Teaching relaxation techniques begin with:
intervention is most appropriate when applying a. Explanation of the technique
the adaptation model of stress? b. The value of relaxation
a. To reduce anxiety c. Physiological changes occurring
b. To maintain fluid balance d. The need to relax
c. To improve or stabilize blood pressure
d. To promote reality orientation
Situation: Mrs. Sol consulted in the health
4. Which of the following body systems is center because she is experiencing daily
primarily involved in General Adaptation headache for two weeks. She is afraid she
Syndrome as a response to stress? cannot finish her work on time. Her headache
a. Neurological and endocrine system is temporarily relieved by paracetamol. Mrs.
b. Endocrine and respiratory system Sol speaks in a low hesitant voice.
c. Central nervous system and cardiovascular
system 1. As per your assessment, Mrs. Sol has been
d. Musculoskeletal and immunological system under stress. Which of the following are the
signs of stress?
5. The 3 stages of General Adaptation a. Teeth grinding or worn teeth
Syndrome are alarm, resistance and b. All these signs of stress
exhaustion. Which of the following response c. Cold hands, headache and tense muscles
are best attributed to the stage of exhaustion? d. Excessive sweating and hypertension
a. Vital signs returning to normal
b. Release epinephrine, norepinephrine 2. Stress can be managed through:
c. Diminished physiological regulation a. Biofeedback
d. Increased blood glucose level b. Relaxation exercise
c. Counseling
Situation: Andrea, 35 years old and married, d. Any of these
augments the family income by managing a 3. For Mrs. Solo, your first nursing intervention
sari-sari store. She worried about her cardiac would be to:
condition. a. Keep her extremities warm
1. What characteristic of a stressor most b. Help her identify source of her stress and to
influence a person’s response to stress? smile
a. Nature c. Demonstrate deep breathing exercises
b. Duration d. Help her set priorities
c. Modifiability
d. Intensity 4. You need some information from Mrs. Sol in
order to help her. You should be able to ask
2. These are physiological response to stress, questions effectively by:
except: a. Asking questions answerable by yes or no
a. Elevation of blood pressure b. Starting questions with a why
b. Increased salivation c. Asking the same question in the same
c. Diaphoresis manner if Mrs. Sol failed to understand
d. Dilation of pupils d. Asking only one question at a time

3. Which part of the nervous system is 5. Listening actively is another way of helping
activated when the body tries to adopt to Mrs. Sol. You can do this by:
stressors: a. Not accepting all that she says
a. Sympathetic nervous system b. Thinking of what to say next as she is
b. Cerebellar stem talking to you
c. Pyramidal tracts c. Giving her time to think and ask questions
d. Medulla oblongata d. Not repeating any statement that you have
heard from her.
4. Why does illness decrease one’s capacity to
adapt to stress? Situation: Sleep is essential for health. It is a
a. It reduces one’s coping ability basic human need affecting all ages.
b. Illness costs a fortune 1. Which of the following is the BEST
c. It is a threat to self-independence intervention to promote safe and restful sleep
d. Illness limits one’s physical activity for an infant?
a. Provide a soft pillow and light blanket c. Some exercise before sleeping
b. Position crib near open window for fresh air d. Taking PRN medications
c. Keep the room softly lit
d. Maintain the room temperature at 70’F 5. Hypnotics are administered with care due to
its:
2. Which intervention would be expected to a. Danger of overdose
best promote safe and restful sleep in a b. Side effects
confused client? c. Addictive potential
a. Allow him/her to watch his/her favorite d. Potential cumulative effect
telenovela
b. Position the bed side rail up 4. FLUID AND ELECTROLYTE BALANCE
c. Keep the room brightly lit Situation: Mrs. Alcantara, a 56 years old
d. Provide sedative at bedtime cardiovascular patient, has edema of the lower
extremities. You suspect some electrolyte
3. Bedtime rituals are particularly important to imbalance.
which of the following groups?
a. Adolescents 1. These are signs of fluid volume excess,
b. School age except:
c. Toddler a. Weight gain
d. Pre-schooler b. Oliguria
c. Neck vein distention
4. Which of the following is a bedtime ritual d. Edema
appropriate for an adult?
a. Use bedroom as work area for unfinished 2. Mrs. Alcantara frequently suffers from
office business diarrhea. Which of the following is indicative of
b. Physical and mental stimulation before dehydration?
bedtime a. Poor skin turgor
c. Exercise at bedtime b. Increase leukocyte count
d. Relaxation techniques such as medications c. Decrease urinary output
or praying at bedtime d. Unstable weight

Situation: In your night rounds, you observed 3. Which is the main excretory organ for
that Mrs. Sayson kept tossing in bed. regulation of fluid electrolyte balance:
1. This is typical altered facial expression of a a. Kidneys
patient unable to sleep: b. Gastrointestinal tract
a. Irritable c. Lungs
b. Restlessness d. Skin
c. Blank look
d. Yawning 4. Who are at highest risk of fluid-electrolyte
imbalance?
2. Which of the following statements on sleep a. Infants
is true: b. Diabetics
a. Bedtime rituals should be avoided c. The elderly
b. Sleep patterns are unique to each individual d. Patients with fluid restrictions
to each individual
c. Patients are restlessness due to illness 5. Which is the best way to assess degree of
d. Naps should be encouraged to make up for edema?
sleep a. Skin indentation when pressed by finger
3. This is the accepted active theory of the b. Comparing present with previous weight
physiology of sleep: c. Pinching a fold of skin
a. Emotional rest d. Measuring intake and output
b. Physical inactivity
c. A nervous center that causes sleep Situation: The doctors wrote in your patient’s
d. Brain fatigue chart this order: “For blood transfusion”.

4. Which is the most practical nursing action to 1. Your most important responsibility in blood
induce sleep? transfusion is to:
a. Some boring activity before retiring a. Take the vital signs before infusion
b. Creating a restful environment b. Determine any known allergy
c. Ensure the correct blood typing and cross- d. Refer him to the doctor
matching
d. Obtain consent of patient 2. An appropriate nursing diagnosis would be:
a. Fluid volume deficit
2. Which of the following blood type is a b. Alteration in fluid-electrolyte balance
universal donor? c. Alteration in skin integrity
a. Type O d. Alteration in nutrition
b. Type B
c. Type AB 3. A few hours later, Gino started vomiting.
d. Type A Your immediate action would be:
a. Refer child to a doctor
3. You were busy when the doctor’s order was b. Let child take ice chips
written. To promptly give the blood c. Give tea for temporary relief
transfusion, you delegate this order to the d. Start IV infusion
midwife. The patient suffered from adverse
reaction and died. You are: 4. The physician ordered IV fluid. You should
a. Liable because of negligence observe the rate of flow in IV infusion to:
b. Not liable because you examined the a. Prevent increased fluid output
doctor’s orders b. Prevent cardiac embarrassment
c. Not liable because the doctor is around c. Avoid IV infiltration
d. Liable because you endorsed it d. Avoid over replacement of fluid

4. Which of the following conditions result 5. Patients with high fever need replacement of
when the wrong blood type is administered to fluids lost since:
a patient? a. Insensible water loss is increased
a. Blood clotting b. There is severe diaphoresis
b. Hemolysis c. Sweat production is increased
c. Internal bleeding d. Dehydration sets in
d. Agglutination reaction
Situation: Any patient with signs of fluid-
5. The best solution to flush IV tubing after electrolyte imbalance should be on intake and
blood transfusion is: output measurement.
a. Ringer’s lactate 1. A principle to remember in accurately
b. Normal saline solution measuring intake and output is:
c. Distilled water a. All fluid intake becomes urine output
d. Dextrose in water b. Input and output cannot be measured
accurately
12. Which of the following should the nurse c. As age increases, proportion of body water
NOT do before blood transfusion? decreases
a. Check identification tag d. Collection of 24 hours intake and output is
b. Check proper cross-match report more accurate
c. Document the blood transfusion
d. Prepare a gauge 23 needle 2. Taking of fluid intake and output is:
15. When administering blood transfusion the a. Initiated by nurse based on her assessment
IV needle gauge preferred to be used is: b. Requires physician’s orders
a. G18 c. Delegated to a watcher
b. G21 d. Can be a nursing order
c. G23
d. G25 3. Which of the following should not be
recorded as fluid intake:
Situation: Gino, 5 years old, was brought by a. Porridge
his mother to the health center with chief b. Cream soup
complaints of fever and watery stools for two c. Ice cream
days now. You noted the child’s sunken d. Gelatin
eyeballs and poor skin turgor.
1. Your initial responsibility is to: 4. Maintenance of fluid and electrolyte balance
a. Take a health history is more critical in children than in adults
b. Perform stool examination because of:
c. Start oral fluid replacement a. Daily fluid requirement is greater
b. Renal function Situation: Mr. Gonzales, 43 years old, is being
c. Cellular metabolism is less stable treated for acute renal failure. He has a foley
d. Proportion of water in body is less catheter in place.
1. Which of the following actions are correct
5. This is an attempt of the body to respond to when collecting a urine specimen from an
dehydration to maintain homeostasis: indwelling urinary catheter?
a. Kidneys retain more salts and water a. Collect urine form a drainage collecting
b. Decrease water absorption in alimentary bag
tract b. Disconnect the catheter from the
c. Less water passes to plasma drainage tubing to collect urine
d. Blood pressure lowers c. Remove the indwelling catheter and
insert a sterile straight catheter to
Situation: Mr. Pedro was admitted to the coolect urine.
emergency room for profuse bleeding from d. Insert a sterile needle with syringe
gaping wound in his right thigh sustained in a through a tubing drainage port cleaned
vehicular accident. with alcohol to collect specimen
1. The patient showed signs of impending
shock which include: 2. A client recovering from surgery tells a
a. Flaccid muscle tone nurse “I feel I have to urinate more often than
b. Restlessness and disorientation usual, and it burns when I urinate”. The nurse
c. Decrease pulse and respiratory rate should plan to obtain a urinary specimen for
d. fever and sweating which of the following factors?
a. culture
2. The physician on duty prescribed IV fluids. b. glucose
The reason for this order is to: c. ketones
a. improve blood pressure d. specific gravity
b. maintain vascular tone
c. improve tissue perfusion 3. A nurse is removing an indwelling urinary
d. improve kidney function catheter. Which of the following is appropriate?
a. wear sterile gloves
3. The least indicator of shock is: b. cut the lumen of the balloon
a. Skin color c. document the time of removal
b. Level of consciousness d. position of the client on the left side
c. Urinary output
d. Blood pressure
6. BOWEL ELIMINATION
4. To hasten wound healing, what would be the Situation: Healthy habits and regular bowel
appropriate diet for Mang Pedro : elimination are some of the factors that are
a. Medium protein, high vitamin diet often associated with health status of an
b. High protein, high vitamin diet individual.
c. High carbohydrate, high protein and fat diet 1. To regulate bowel elimination, it is best that
d. High protein, low carbohydrate diet the client regularly contains foods that are high
in roughage. Which types of foods contains the
5. To control bleeding, the best position for least fiber or roughage?
Mang Pedro is: a. Cooked fruits
a. Trendelenburg position b. Instant noodles
b. Sideways with injured thigh up c. Green vegetables
c. Fowler’s position d. Whole grains or cereals
d. Elevation of injured thigh
2. Bowel elimination is best facilitated by which
32. Home made Oresol is composed of: of the following:
a. Water and salt a. Increasing fluid intake ton 1400 to 2000 ml
b. Electrolytes daily
c. Water and little sugar b. Drinking milk
d. Electrolytes and sugar c. Drinking hot beverages and fruit juices
d. Eating green vegetables
5. URINARY ELIMINATION
3. Mechanical and chemical digestion begins in
the:
a. Stomach Situation: The home care nurse visited your
b. Small Intestine patient, Mang Lorenzo, and noted that he had
c. Large Intestine no bowel movement for five days now.
d. Mouth 1. Which of the following questions can elicit
information on Mang Lorenzo’s coping ability to
4. Most nutrients and electrolytes are absorbed deal with his problem defecating?
in the: a. “Do you have problems defecating?”
a. Stomach b. “ What have you done to relieve your
b. Small intestine constipation?”
c. Large intestine c. “Have you consulted a doctor for your
d. Colon problem?”
d. “Did you take any constipation pill?”
5. Which of the following will least likely
promote regular bowel habits to hospitalized 2. Mang Lorenzo was advised to take at least
patients? 3000 milliliters of waters a day. This amount is
a. Prohibit ambulation and exercise while in the approximately equivalent to how many glasses
hospital even if they are not contraindicated of water>
b. Take time for defecation a. 8
c. Make certain that treatment routine do not b. 10
interfere with the client’s schedule c. 14
d. Provide privacy c. 12

Situation: Von, 55 years old, is confined with a 3. Mang Lorenzo complained of a gassy
diagnosis of carcinoma of the colon. abdomen. His attending physician ordered
1. Which of these is not a sign and symptom of rectal tube insertion. You would insert the
carcinoma of the colon? tube:
a. Diarrhea a. 6-8 inches
b. Fatigue b. 2-4 inches
c. Anorexia c. 4-6 inches
d. Melena d. 8-10 inches

2. Diagnostic examination to establish 4. The rectal tube should remain in the colon
diagnosis of cancer of the colon: at least 30 minutes and reinserted every 2-3
a. Proctoscopy hours to prevent:
b. Anoscopy a. Undue irritation of rectal lining
c. Cholangiography b. Discomfort of patient
d. Sigmoidoscopy c. Puncturing of colon
d. Clogging tube of feces
3. How should Von be positioned for this
diagnostic examination? 5. Repeated inhibition of the urge to defecate
a. Sim’s results in:
b. Semi-Fowlers a. Slowing of peristaltic movement
c. Knee-chest b. Loss of sensitivity to defecate
d. Trendelenburg c. Rigidity of the anal sphincter
d. Formation of impacted feces
4. How far do you insert the catheter during
colostomy irrigation? Situation: Mrs. Dolores Diaz, 36, chose to have
a. 2-4 inches elective hemorrhoidectomy on doctor’s advice:
b. 4-6 inches 1. A cleansing enema is ordered pre-
c. 1-2 inches operatively, what is the proper patient position
d. 6-8 inches for this:
a. Prone
5. The size of the catheter the nurse should b. Left lateral
prepare for colostomy irrigation is from: c. Knee chest
a. 14-16 d. Right lateral
b. 18-20
c. 16-18 2. In the administration of pre-operative
d. 12-14 injections, which of the following available
needles should be used for an IM injection, 3. One of your clients has just undergone an
because the patient is 5 ft 6” and 115 lbs? ear surgery. Which of the following would be
a. G-20, 1 inch long inappropriate in planning for his care?
b. G-22, 1 ½ inches long a. Administration of anti-emetics and
c. G-19, 1 ¼ inches long analgesics as ordered
d. G-26, 1 inch long b. Daily irrigation of the ear canal
c. Walking with assistance at least 24 hours
3. In the early post-operative period, what is after the operation
the ideal position to put the patient in: d. Teaching the patient to avoid sneezing,
a. Prone coughing and nose blowing
b. High Fowlers
c. Trendelenburg 4. In which of the following conditions would
d. Supine an irrigation of the ear canal be an appropriate
intervention?
4. Mrs. Diaz is to continue her sitz bath at a. Foreign body in the ear canal
home. When is the best, safest time to do this: b. Serious otitis media
a. At bedtime c. Impacted cerumen
b. After a bowel movement d. Tympanic membrane perforation
c. First thing in the morning
d. As needed for discomfort 5. Children who have undetected hearing loss
are likely to exhibit which of the following:
5. Metamucil is to be taken BID to prevent a. indifference and lack of interest in the
constipation. How primarily does this drug environment
produce the desired effect? b. hyperactivity
a. Stimulating peristalsis thru contact irritation c. an increased interest in reading
b. Controlling irritating flatus d. hand gestures while speaking
c. Softening the stool
d. Increasing the bulk of stool Situation: It is the duty of every health
practitioner either at home, community or
hospital to always ensure that patients are
protected from health hazards.
1. The safety of any individual is usually
7. SAFETY, COMFORT AND HYGIENE threatened by:
Situation: Hearing impairment appears to be a. unrefrigerated fresh fruits and vegetab;es
common among elderly patients. But it also b. individuals incineration of garbage
occurs among children. c. lack of adequate water supply
1. To assess the degree of hearing impairment d. atmosphere carbon dioxide.
of a 70-year old client, which communication
approach would you initiate? 2. The most common major risk to safety of
a. Use verbal communication and observe the unconscious patients in the hospital are:
response a. carbon dioxide poisoning
b. Write messages to client in writing b. any poisoning
c. Ask a family member about the client’s c. falls from bed
health problem d. insect bites
d. Post a sign “Patient is deaf”
3. Which of the following takes the highest
2. While you are making your routine rounds, priority when fire occurs in a health care
you were told that there is a client in the ICU facility?
who is in the respirator and who lip-reads. To a. report the location of the fire
establish relationship with him, communication b. contain the fire
is best accomplished by: c. protect the patients from injury
a. speaking slowly but aloud d. protect yourself from injury
b. writing messages
c. gesturing while speaking 4. The best initial action after an accidental
d. using simple “charades” approach or poisoning is to induce vomiting by:
strategy a. asking the patient to bend forward until the
head reaches the level below the waistline
b. giving glass of milk
c. by inserting a tongue depressor in the Situation: Josefina who is assigned to a
mouth medical-surgical ward reviewed the principles
d. giving syrup of Ipecac of patient comfort and safety. The following
question relate ton what Josefina has been
5. Which of the following will least likely taught.
prevent falls among adult patients? 1. Which of the following has the greatest
a. Have a call light and promptly answer call influence on personal hygiene?
light a. Culture, knowledge and social influences
b. Keep client’s personal items away from bed b. Occupation, physical environment and
c. Identify clients at risk for falls genetics
d. Use a night life among adult patients in the c. Self-awareness, time of day and personal
hospital available for assistance
d. Socioeconomic status, health and age
Situation: A safe environment is much to be
desired in many of our home and 2. The most important intervention to correct
establishments like hospitals and schools. skin dryness.
1. Which of the following describe unsafe a. Avoid bathing the patient until the condition
hospital practice? is remedied and notify the physician
a. Physical hazards are reduced by putting up b. Ask the physician to refer the patient to a
appropriate and visible warnings dermatologist and suggest that the patient
b. Medical and surgical instruments are wear home-launder sleepwear
decontaminated before washing c. Consult the dietitian about increasing the
c. Used needles are thrown into a garbage can patient’s fat intake and take necessary
together with other hospital wastes measures to prevent infection
d. Transmission of pathogens is reduced by d. Encourage the patient to increase his fluid
mopping the floor with Lysol. intake, use non-irritating soap when bathing
the patient and apply lotion to the involved
2. Which of the following will least likely areas
endanger individuals’ safety in the community?
a. development stages 3. When bathing patient’s extremities, Josefina
b. safety awareness should use long firm strokes from distal to
c. normal sensory functioning proximal areas. This technique
d. lifestyle habits a. Provides an opportunity for skin assessment
b. Avoids undue strain on the nurse
3. Which of the following nursing intervention c. Increase venous blood return
best promotes safety in a hospitalized toddler? d. cause vasoconstriction and increase
a. Cover electrical outlets with protective circulation
covers
b. Provide matchbox case for group play Situation: The nurse in the care of her
c. Provide spoon and fork at meal time patients, often overlooks oral hygiene.
d. Put up bedside rail on one side to facilitate 1. Which of the following observation is most
reducing of nursing procedure. important to determine before providing oral
hygiene to the unconscious patient?
4. The greatest risk of death from home a. Presence of a functioning gag reflex
accidents occur in children: b. Presence of dental caries or halitosis
a. between 9-12 years old c. Color, texture or bleeding of gums
b. between 12-16 years old d. Dryness or discoloration of the tongue
c. less than 5 years old
d. between 5-8 years old 2. Miss Agnes is a 23-year old unconscious
patient because of a head injury. When
5. The majority of fatalities among children 6- providing mouth care, the nurse should:
12 years old are associated with: a. Position the patient on semi-fowler’s position
a. accidents to prevent aspiration of secretion
b. infectious diseases b. Position the patient on her back and use a
c. accidental poisoning padded tongue depressor to keep the teeth
d. congenital diseases separated
c. Position the patient in her crib and have
suction equipment available to remove
secretions
d. Position the patient in a Trendelenburg b. Pulse and temperature every 30 minutes
position to prevent aspiration of secretion and for one (1) hour after the procedure
c. Pulse and temperature immediately before
3. In caring for a patient with dentures, the and every 16 minutes during the procedure
nurse: d. Pulse and temperature immediately before
a. Rinses dentures in cold water to promote and after the procedure
client comfort
b. Uses hot water to ensure thorough cleaning 4. Which of the following is the correct
of denture temperature of the water for tepid sponge
c. Assists the client in brushing gums, palate bath?
and tongue a. 38’C (warm water)
d. Stores the denture in a clean, dry container b. 37’C (tap water)
c. 35’C (iced water)
4. Which of the following oral hygiene products d. 37.5’C (lukewarm water)
would place the patient at risk for oral
problems? 5. Which of the following is NOT a guideline
a. Lemon-glycerin sponges when providing a client with any type of bath?
b. Hydrogen peroxide with water a. Provide dependency needs of client
c. Flouride toothpaste b. Maintain safety
d. Potassium Permanganate c. Promote client independence
d. Provide privacy
5. Clients receiving medication therapy is at
risk for oral problems because radiation car 4. Which solution is used for rinsing the mouth
cause: of children?
a. nausea and taste changes a. NSS
b. patient is unable to attend to personal b. Hydrogen peroxide
hygiene needs c. Na Flouride 0.2%
c. dysphagia and oral infection d. Water
d. dryness and soreness of the mouth
Situation: Practically everyone has had
Situation: For quality care, nurses should experienced sensory deprivation his life time.
provide not only the therapeutic bath but This condition is especially experienced by
facilitate daily cleansing bath as well. hospitalized patients who find themselves
separated from their families and friends,
1. Which of the following is NOT a purpose of a unable to perform their regular functions and
cleansing bath and skin care? sometimes, because of illness, have difficulty
a. To reduce local inflammation in their sensory functions.
b. To promote comfort
c. To provide exercise 1. Which of the following client groups have
d. To stimulate circulation the LEAST risk for sensory alteration during
hospitalization?
2. Which of the following positions of the client a. Elderly clients
and the hospital bed is MOST appropriate in b. Clients in ICU
doing a bed bath for him? c. Ambulatory patients
a. Move the client away from you and adjust d. Bedridden patients
the bed in high position
b. Move the client towards you and adjust the 2. Because of existing multiple sensory stimuli,
bed in low position patients with sensory overload also experience
c. Move the client towards you and adjust the sensory alterations. Which of the following
bed in high position nursing interventions would be appropriate for
d. Move the client away from you and adjust clients with sensory overload such as those
the bed in low position with hyperesthesia?
a. Firm pressure when touching body parts
3. Your patient is running a 38.5 temperature. b. Frequent back rubs or body massage
Before you administer a tepid sponge bath, c. Minimal use of direct touch
which of the following should you assess? d. Vigorous hair brushing or massage of scalp
a. Pulse and temperature before the procedure
and one hour after the procedure 3. Clients who are recovering from a general
anesthesia oftentimes manifest altered level of
consciousness. Which of the following nursing 4. Injuries most common during early school
interventions will be of LEAST help to post- age are all of these, except:
anesthesia patients? a. Falls
a. Encourage relatives to talk and touch as b. Drowning
though client is conscious c. Motor vehicle accident
b. Reorient client to time and place d. Poisoning
c. Don’t waste time by explaining to clients
routine procedures like bath, feeding and 5. As part of her school health program, the
others nurse anticipates health teachings on
d. Introduce self before starting an activity precautionary measures and safety education
with client appropriate to their typical behavior. For
adolescent group, the focus is on:
4. Which of the following assessment tool is 1. Enforcing safety rules in diving,
used in identifying visual sensory deficit? swimming and other play
a. Tuning fork 2. Avoiding smoldering fires and
b. Snellen chart matches, bottles and can that may explode
c. Otoscope 3. Pointing out serious effects of
d. Audiometer fireworks and other hazardous materials
4. Storing dangerous tools and
5. Which of the following goals is LEAST equipment in locked cupboard
appropriate for clients with sensory a. 1, 2, 3 and 4
alterations? b. 1, 2 and 3
a. Establish a safe environment c. 1, 3 and 4
b. Achieve self-care d. 1, 2 and 4
c. Provide well-balanced diet
d. Create meaningful sensory stimuli Situation: Mang Beloy, 26 years old laborer,
fell from a 4-storey building construction. He
Situation: This pertains to alerting and was rushed to the emergency room and
instructing parents aimed at injury prevention. admitted. He became unconscious after two
The Parent Teachers Association agreed to hold hours.
health-teaching classes for parents of school 1. The priority nursing goal of patient with
age children. sensory alteration is:
1. During the health teaching classes for a. Maintaining adequate communication
parents, the nurse stresses the fundamental b. Ensuring proper nutrition
prerequisite of injury prevention for school age c. Providing a safe environment
children, which are: d. Promoting social interaction
a. Diligence, hard work and practice
b. Safe home environment and regular 2. In the case of Mang Beloy, you will consider
supervision the following goals. The one to receive first
c. Place for developing and accomplishing goal priority is to:
d. Education, discipline and obedience a. Monitor neurological status including vital
signs
2. Child health supervision is designed to b. Maintain his fluid-electrolyte balance
provide all of the following except c. Maintain an open airway
a. Detecting the presence of deformity or d. Control his pain and restlessness
disease
b. Helping in interpreting nutritional 3. An unconscious patient is unable to:
requirements and protecting against diseases a. Move spontaneously
c. Giving appropriate counseling b. React to painful stimuli
d. Providing monetary assistance for needy c. Hear voices
families d. Control elimination

3. A powerful physical antidote that absorbs 4. You should refrain discussing an


most poison is: unconscious patient’s condition at his bedside
a. Apomorphine because:
b. Activated charcoal a. Critical condition of an unconscious patient
c. Syrup of Ipecac is confidential information
d. Narcan b. Level of awareness of patient is high
c.. It is unethical to discuss critical condition of c. The skin is pink and warm to touch
patient d. You know that Mrs. Tano has not been
d. The hearing sense is not entirely lost when eating well and she is at risk for pressure sores
unconscious
4. You have the most important reason for
5. The most sensitive indicator of an bedsores as a threat to Mrs. Tano’s skin
unconscious patient with head injury is the: integrity is:
a. Muscle twitching a. Moisture from perspiration can produce
b. Pupillary changes laceration of the skin
c. Blood pressure and pulse b. Excessive pressure collapse blood vessels
d. Motor functions and impedes blood flow eventually causing
tissue damage
8. MOBILITY AND IMMOBILITY c. Invasion of microorganisms create infection
in the skin which has been subjected to
Situation: Mrs. Tano, 60 years old, was continuous pressure over time
admitted because of cardiovascular accident d. Friction allows pulling in tissues thus
(CVA) resulting to paralysis of the left side of stretching and injuring tissue and blood vessels
the body.
5. Your priority intervention with Mrs. Tano
1. You planned to start passive range of when you discover she is developing pressure
motion exercises today. Which one of the sores would be to:
following should you avoid in doing passive a. Rub affected skin vigorously to stimulate
range of motion exercises? circulation
a. The extremity should be supported b. Relieve pressure to prevent interference
particularly the bones below and above the with blood flow
exercising joint. c. Cleanse the area well with alcohol to prevent
b. During the passive exercises, the client infection
should be lying in supine position. d. Increase fluids to prevent electrolyte
c. Each joint should be moved slowly through imbalance
its full range.
d. If a joint appears to be stiff, the exercise Situation: Aling Iska, a 45-year old paralytic
should be continued slightly beyond the point patient is bedridden
of pain. 1. What would be your primary goal to
maintain skin integrity of Aling Iska?
2. One day, while making the usual rounds of a. Keep patient dry
your clients in the ward, you realize that Mrs. b. Avoid infection
Tano has been lying in the same position for c. Prevention of bed sore
more than two hours. You are very much d. Regular turning from side to side
concerned because:
a. Mrs. Tano is vulnerable to the development 2. The immediate cause of bed sore is:
of pressure sores during the time period a. Unhygienic procedures
b. The husband has been complaining about b. Skin break
her wife’s care and you are worried that the c. Sustained pressure
husband might report you to the supervisor d. Poor blood circulation
c. Mrs. Tano is vulnerable to the development
of contractures in that time perid 3. Why is the coccygeal prone to bed sore
d. You have repeatedly instructed the formation?
attendant to turn Mrs. Tano every two hours. a. There is less blood circulation
b. It is a bony prominence
3. You went to see Mrs. Tano to assess the c. It is constantly in contact with linen
integrity of her skin because of your concern d. It is near the perineum
that she has not been turned. When you check
her bony prominence on one side, you noticed 4. You regularly turn the patient from side to
that there is beginning pressure sores side mainly to:
formation because: a. Make patient comfortable
a. You know that the attendant did not turn b. Put patient in proper position
Mrs. Tano to the sides c. Maintain proper body alignment
b. The skin is red to purple gray and hot to d. Promote blood circulation
touch
5. Which of the following is the first line of b. Skin is blanched
defense of the body? c. Papules appear
a. Hygienic habits d. Skin is taut
b. Clean environment
c. Good nutrition 2. How often should the patient be
d. An unbroken skin repositioned:
a. Every 4 to 6 hours
Situation: This situation is about nursing b. Every shift
responsibilities in ambulation across groups. c. Every 2 to 4 hours
1. In caring for patients who have undergone d. As often as necessary
abdominal operative procedure and have been
given general anesthesia, how soon should you 3. Which of the following techniques of moving
encourage ambulation? would most likely cause bedsore formation?
a. If patient is afebrile a. Moving patient to a chair with support
b. The day after surgery b. Rolling patient to his side with a drawsheet
c. Upon awakening from anesthesia c. Sliding patient on the sheet to move him up
d. When catheter is removed in bed
d. Lifting patient from bed to stretcher
2. How do you start ambulating your patient
who have hip braces? 4. When transferring the patient from bed to
a. Standing up with full back support stretcher, where should the stretcher be placed
b. Wheelchair first then walk with active in relation to the bed:
assistance a. At a 45 degree angle near the middle of the
c. Side lying with one foot dangled bed
d. Start by dangling both feet with back b. Parallel to the side of the bed
support c. At a right at the foot of the bed
d. Diagonally at the foot of the bed
3. When a child starts to crawl and stand up,
how do you prepare for ambulation? 5. What is the diet of a patient with developing
a. Surround with hammock belt support and decubitus ulcer:
walk a. High vitamin intake
b. Hold both hands and step according to b. High protein intake
child’s capability c. High caloric intake
c. Hold hands tightly and let child follow your d. Low fat diet
small steps
d. Put child in a walker with a security belt Situation: Good body mechanics is safe use of
the body to produce motion and maintain
4. What about older persons who need to walk balance during activity.
a little, how do you ambulate them? 1. The major purpose of body mechanics is to:
a. Support elbow and place other hand on a. Prevent injury or falls
shoulder of patient b. Improve body image
b. Put hand on waist and walk c. Facilitate efficient use of appropriate
c. Hold the elbow while walking muscles
d. Hold hands tightly d. Maintain body balance and posture gait

5. How do you ambulate a deaf patient? 2. Which of the following basic elements are
a. Hold one hand and walk according to one’s involved in body mechanics:
pace a. Posture balance and body movement
b. Walk in front guiding the way b. Range of action movement and exercise
c. Walk side by side c. Exercise, nutrition and posture
d. Hold waist and walk slowly d. Activities of daily living and exercise

Situation: You were newly assigned to Mang 3. What condition would be an outcome of a
Cocoy, an unconscious patient with right-sided client’s potential immobility problem?
paralysis. You noted reddening of the skin over a. Has no feelings of depression
the coccygeal area. b. Gains more knowledge about body
1. What is the first sign that decubitus ulcer alignment
might be developing: c. Identifies factors contributing in mobility
a. Rash appearance d. Joins groups with similar problems as client
4. These are factors that influence early Situation: One of the important roles of a
ambulation of immobilized patients, except: nurse in the care of her patients is that of
a. Will to live being a health educator. As in any teaching-
b. Good body image learning situation, the nurse must be able to
c. Minimize more immobility problem apply the various teaching-learning principles
d. Better lifestyle and must always demonstrate the positive
attitude of an educator.
5. Body mechanics is of value to the nurse as 1. Which of the following factors LEAST
it: influence the learning readiness of an adult
a. Decrease risk of injury learner?
b. Promote active exercise a. The individual’s level or stage of
c. Reduce extra body fats development
d. Increases energy conservation b. Ability to concentrate on information to be
learned
III. HEALTH EDUCATION c. The individual’s psychosocial adaptation to
his illness
A. TEACHING AND LEARNING PRINCIPLES d. Internal impulses that drive the person to
IN THE CARE OF CLIENTS take action
Situation: Your teaching role is most essential
if you expect people to keep healthy and well. 2. Which of the following is the most
1. The ultimate aim of teaching clients/patients important condition for diabetic patients to
is to bring about: learn how to control their diet?
a. Increase knowledge of a disease a. Use of pamphlets and other materials during
b. Develop self care skills health instruction
c. Awareness of health practices b. Motivation to be symptom-free
d. Change in health behavior c. Ability of the patient to understand teaching
instruction
2. When teaching an elderly, what is the most d. Language used by the nurse
important ability that you should assess?
a. Sensory alterations 3. An important skill that a primigravida has to
b. Level of developmental task acquire is the ability to bathe her newborn
c. Able to express feelings baby and to clean her breast if she decides to
d. Level of health knowledge breastfeed her baby. Which of the learning
domain will you classify the above goals?
3. Your teaching will be more effective if you a. Psychomotor
start: b. Cognitive
a. With a problem area of concern to the c. Affective
patient d. Attitudinal
b. When the doctor has discharge orders
c. When the patient is ready to learn 4. When you prepare your teaching plan for a
d. As early as the first nurse-patient contact group of hypertensive patients, you first
formulate your learning objectives. Which of
4. What is the first thing you would do when the following steps in the nursing process
planning a teaching session: correspond to the writing of learning
a. Set teaching objectives objectives?
b. Interview target audience to get their a. Planning
interest b. Implementing
c. Identify learning needs c. Evaluation
d. Identify learning resources d. Assessment

5. Giving health information by itself is unlikely 5. Rose, 50 years old, a newly diagnosed
to result in better health of your patient diabetic patient, must learn self injection of
unless: insulin. Which of the following physical
a. Patient takes health actions attributes is NOT related to her ability to self
b. Patient shows willingness to comply inject insulin?
c. Your message is clear a. Strength
d. Information has been shared with other b. Coordination
patients c. Body built
d. Sensory Acuity 5. In giving health teaching, it is important for
the nurse to be aware of which of the following
34. Which of the following is correct in factors regarding client’s learning needs?
teaching the elderly a new learned material? a. They change as you modify the teaching
a. Give in a 15 minute session technique
b. Distribute reading materials b. They are the same for everyone
c. Allow more time for learning c. They change throughout the life cycle
d. Give in a hurried pace to force them to pay d. They change daily
attention
Situation: Miss Bartolome is assigned to Mang
Situation: Health education is an important in Lando, a 62-year old newly diagnosed diabetic
the care of patients. As such, the nurse must patient. She is beginning to write objectives for
be cognizant of the principles involved in the her teaching plan?
teaching-learning process as applied to client 1. Which of the following objectives is written
teaching. in behavioral terms?
1. A learning objective by a nurse reads “ a. Mang Lando’s wife needs to understand
Clients will change her dressing using clean b. Mang Lando’s daughter should learn about
technique before his discharge from the diabetes mellitus within a week
hospital.” Wound dressing using clean c. Mang Lando’s sister will be able to
technique is classified under which component determine his insulin requirements based on
of learning objective? blood glucose levels from the glucometer in 2
a. Purpose days
b. Behavior d. Mang Lando will know about diabetes
c. Condition related to foot care and techniques and
d. Content equipment necessary to carry out

2. In teaching a post-oophorectomy patient on 2. Which of the following is the BEST rationale


how to apply the dressing at home, which of for written objectives?
the following physical attributes would not a. Ensure communication among staff
relate to his ability to perform psychomotor members
task? b. Ensure learning on the part of the nurse
a. Privacy c. Facilitate observation of performance on the
b. Age nurse
c. Strength d. Document the quality of care
d. Coordination
3. Which of the following behavior BEST
3. A clinical example of psychomotor learning contributes to the learning of Mang Lando
is when a client does whish of the following? regarding his disease condition?
a. Chooses the type of exercise after delivery a. Detailed lengthy explanations
b. State an acceptance for the need to b. Loosely structured teaching session
amputate his leg c. Frequent use of the technical terms
c. Explains the purpose and action of a blood d. Drawing him into discussion about diabetes
transfusion
d. Changes the dressing aseptically in a leg 4. Which of the following behaviors of Mang
ulcer Lando is LEAST indicative of his readiness to
learn?
4. Nurses are expected to provide the needed a. Inquiring about the date of discharge
health teachings to all her patients as provided b. Reading brochures and pamphlets about
in which of the following? diabetes
a. The Constitution of the Philippines as c. Talking with the nurse in charge and his
contained in the functions of the DOH doctors
b. The By-Laws of the Philippine Nurses
Association
c. The Code of Ethics for Nurses as B. HEALTH EDUCATION IN ALL LEVELS OF
promulgated by the BON CARE
d. Nursing Act of 2000 Situation: Andres, 26 years old, a tricycle
driver and married, consulted the outpatient
clinic for hoarse voice on and off for almost
two years now. Health history reveals that he
has been smoking since his teenage years. 2. Discharge teaching instructions are very
important to all clients. Which of the following
1. Illness prevention activities are designed to instructions should you emphasize to prevent
help clients: recurrence in a patient treated for
a. Manage stressful events pyelonephritis?
b. Identify danger signals of disease a. Monitoring urine output
c. Reduce risk factors of specific diseases b. Maintenance of perineal hygiene
d. Change habits affecting health c. Limiting fluid intake to 1 liter per day
d. Discontinuance of antibiotic therapy when
2. To be able to modify the smoking behavior the client feels better
of Andres, it is most important to collect which
of the following data: 3. On discharge, the nurse teaches the client
a. His health beliefs and practices diagnosed to have Meniere’s syndrome to care
b. His family health lifestyle for herself at home by which of the following?
c. Coping ability of his family a. Avoiding quick movement of the head
d. Available health services in his community b. Forcing fluids to decrease
hemoconcentration
3. Which of the following conditions will most c. Reducing loud and excessive noise in the
likely convince Andres to quit smoking: environment
a. A strong motivation to quit smoking d. Increasing activity such as walking rapidly
b. Giving some incentive if he quits smoking to help maintain mobility
c. Knowledge on ill-effect of smoking
d. Social support for the desired behavior 4. A client who has Meniere’s syndrome is to
be discharged. Which of the following should
4. In attempting to change undesirable health the nurse teach about diet at home?
behavior, it is most important that the decision a. Increase intake of milk
is: b. Eat three large meals a day
a. Made by the nurse together with the client c. Eat whatever is tolerated as long as they
b. Left to the client himself don’t cause some distress
c. Refer to the decision makers in the family d. Avoid caffeine and alcohol
d. Done by the nurse who is an authority on
health 5. A client diagnosed with rheumatoid arthritis
is taught by the nurse to do range of motion
5. Smoking cessation programs is more likely (ROM) exercises as a preparation for
to be successful if you first focus intervention discharge. In implementing a plan of care for
on the behavior that is: the patients, which of the following should a
a. A hazard of public health nurse do?
b. Of most concern to client a. Allow long rest periods between exercises
c. Unacceptable to friends b. Point out small accomplishments made by
d. Prohibited by authorities the client
c. Praise the client only when she makes
Situation: Patient education is one of the most significant accomplishment
important responsibilities of the nurse. To be d. Do simple things for the client so she knows
effective, it is necessary for the nurse to the nurse is supportive of her.
render appropriate health teaching according
to the specific health problems of her clients. Situation: Celiovit is a 28 years old father
whose only son, Nilo, contacts his sore eyes
1. In educating clients regarding testicular from playing dirty water in Biniray river bend.
prostate cancer, which of the following 1. What will the nurse teach Celiovit about sore
statements is correct? eyes from the statements below?
a. “Excess alcohol intake of more than three a. Offer white chicken to remove redness of
bottles per day can cause prostate cancer”. sore eyes
b. “Men who fathered children are less prone b. Sore eyes are curable and are self limiting
to prostate cancer.” c. Sore eyes are caused by peeping in the hole
c. “Testicular cancer is the leading cause of of others
death among men in middle-age group.” d. Fresh urine of babies are cure for sore eyes
d. “Testicular cancer is a disease of the
elderly.”
2. In rural areas, the old people believe that d. Important and accepted
when they cannot see clearly, an evil charm
has fallen on them. You can tell them this: 4. When conducting teaching sessions with
a. Perhaps you have cataract – this can be older adults, the best method to use is:
remedied a. Give them reading materials
b. Diabetes make eyesight blurred and you b. Allow them to socialize
may be having this c. Keep session short enough
c. Maybe you have glaucoma – you need d. Serve snack in between sessions
treatment right away
d. This is just passing, you will see later on 5. Which of the following remarks will most
likely create a positive attitude of your patient
3. How important is health teaching in rural on rehabilitative nursing:
areas as you have experienced during your a. Hospitalization is only temporary
community exposure-immersion? b. Rehabilitation is positive health
a. 75% c. Life goes on even after hospitalization
b. 50% d. While there is life, there is hope
c. 100%
d. 25% C. DISCHARGE PLANNING
Situation: Aling Juana is for discharge. During
4. Why is health teaching an important facet of her hospital confinement, you noted her lack of
the nursing profession? Because health knowledge about her illness. You made a
teaching is: discharge teaching plan.
a. Done when there is nothing else to do
b. One way of keeping promotional health 1. The choice of discharge teaching topics will
c. Portraying the teaching role of nursing depend on:
d. Part of total nursing care a. Continuing needs of patients
b. Patient diagnosis
5. Cleanliness is best done through: c. Instructions as ordered by doctor
a. Community work d. Treatment regimen
b. Role modeling
c. Competitive activities 2. You can say that your patient has learned
d. Health teaching when:
a. He says so
Situation: Aling Clara, 60 years old, is a retiree b. Promises to follow advice
from a government office. She has spent thirty c. Asks further questions
years working in the Department of Health. d. Takes the desired action
1. In assessing the health status of Aling Clara,
the most important information to obtain for a 3. When giving counsel to a patient, he is most
teaching plan is her: likely to need your advice if you:
a. Health beliefs and practices a. Assure him of benefits
b. Retirement plan b. Let him make the decision
c. Modification in activities c. Threaten him for non-compliance
d. Physiological changes d. Give him time to reflect in your advice

2. In establishing goals of nursing care for 4. You can best model your teacher role
Aling Clara, you will give priority to: through:
a. Selecting a variety of recreational activities a. Instructing your patients after care
b. Submitting for physical examination b. Giving incidental teachings
regularly c. Providing health posters
c. Maintaining independence as much as d. Practicing healthy habits
possible
d. Referring to social services when necessary 5. One of the following important nursing
activities in promoting health is:
3. The most important attitude to demonstrate a. Case finding
when doing rehabilitative nursing is to let the b. Conducting patient instruction
patient feel: c. Assisting in immunization
a. Support from family d. Giving anticipatory guidance
b. Trusted and respected
c. Less of a dependent
Situation: Mr. Tinio, a cardiovascular patient
has been hospitalized for 4 months and his 3. On discharge, a good regimen for Mrs. Prado
doctor has ordered for discharge. is:
1. Mr. Tinio appears anxious about numerous a. Spend most of the time reading
aspects of his home care. The best response b. Watch horror movies in TV
you would give is: c. Active exercise
a. Explore with him his fears and allow him to d. Social recreation
verbalize feelings.
b. Advice him to take all his medications 4. In long term disability, it is especially
regularly important that the nurse at all times:m
c. Explain why he has to have certain activity a. Hasten self-care skills
limitation b. Treat patient as a human being
d. Let him express his feelings to reduce his c. Show general concern
anxiety d. Keep busy to avoid boredom

2. An immediate need for Mr. Tinio is: 5. The most important health team member in
a. Reduce anxiety home care is:
b. More independent in our look a. The Physician
c. Security and comfort b. The Public Health Nurse
d. Conserve anxiety c. The Social Worker
d. The Patient and his Family
3. Cardiac rehabilitation programs in the acute
stage aids the person in:
a. Reaching an activity level required for self-
care IV. ETHICO-MORAL RESPONSIBILITY
b. Preventing further complications A. BIOETHICAL PRINCIPLES
c. Plan his activity of living 1. Beneficence
d. Acceptance of his condition 2. Non-malefescence
3. Justice
4. Cardiac rehabilitation goals in long term 4. Autonomy
restore individual to optimum health and: 5. Stewardship
a. Prevent heart complaints 6. Truth-telling
b. Slow down the progress of disease Situation: Nurses oftentimes encounter various
c. Another myocardial infraction attack ethical dilemmas in the care of her patient in
d. Avoid exposure to infection special areas of the hospital like the ICU and
the operating room.
5. The greatest effect on his home recovery
will be his: 1. One time a nurse in the ICU was told by the
a. Expectations to go back to work hospital administrator to omit blood
b. Dietary needs to be well transfusion ordered by the doctor because the
c. Family’s emotional support patient is going to die eventually anyw. What
d. Understanding of the cause of his illness must the nurse do?
a. Follow the doctor’s order and be silent about
Situation: Mrs. Prado, a 70 years old retiree it
expressed concern about her coming b. Resign from the job is pressured by the
discharge. administration
1. What would be the most effective c. Report the incident to the doctor and risk
intervention you would do first? her job depending upon the doctor’s decision
a. Ask what concerns her d. Obey the order of the hospital administrator
b. Teach family about home care
c. Request attending physician to reassure her 2. A patient with terminal cancer has reached
d. Refer her to social worker the stage of acceptance. She told the nurse to
discontinue all efforts to prolong his life. Which
2. Your home care teaching will more likely be of the following should the nurse do?
effective if you: a. Seek further advise from other nurses
a. Teach behavior modification before taking any further drastic decision
b. Include only in family teaching b. Disregard patient’s request and continue all
c. Leave teaching to midwife efforts to maintain patient’s life
d. Know patient home care needs
c. Interact with the patient and continue the 21.The belief that euthanasia is absolutely
care and support for the patient wrong in any scenario, it is a/an:
d. Tell the patient that it would be a sin a. Moral belief
because life is holy and dignified. b. Personal belief
c. Ethical issue
3. The dying patient was visited by his spiritual d. Ethical principle
adviser. The private nurse was with the patient
all the time during the chaplain visit. When an 40. A major principle says that do no harm:
elder brother came to visit, he asked the nurse a. Beneficence
what the sick brother and the chaplain were b. Justice
talking about. What would be the right thing c. Autonomy
for the nurse to do? d. Non-maleficence
a. Ask the relative to look for another nurse
who may tell him even if it is privilege 42. In the hospital where there is only one
communication pacemaker, which of the following client should
b. She should not reveal anything to the visitor be your priority to use the pacemaker?
because it is a privilege communication a. A 46-year-old woman cousin of a
c. Tell the elder brother that relatives should congressman
keep out of the spiritual affairs of the patient b. A 65-year-old diabetic client
d. She should reveal some information that c. A 10-year-old boy who supports the
concern the elder brother of the patient grandmother
d. A 26-year old man with 5 children.
4. A new OR nurse finds out that she is to
assist in bilateral salphingectomy operation. Situation: In rendering care to a handicapped
She has heard that this might involve a child, the nurse has many responsibilities. Most
question of morality. The head nurse tells her of the entire nurse should be ethical
that she has nothing to worry about. How
would you respond to the situation? 1. What principal bioethical principle should a
a. Examine herself for the meantime and wait nurse carry when caring for a handicapped
until she becomes a midwife child?
b. Assist when the indication for the operation a. Justice
is medically certain b. Beneficence
c. Seek further advise from the spiritual c. Respect
adviser d. Non-malificence
d. Report the whole matter to the head nurse
2. Regarding idiosyncracies of a handicapped
5. Which of the following should be considered child, how should a nurse deal with the
by the nurse before getting the client/patient tantrums?
to sign the operative consent? a. Respect
a. evaluate the client’s level of consciousness b. Beneficence
and intellectual capability c. Justice
b. sedate the client to make sure she is relaxed d. Non-malificence
enough to sign the consent without mild
anxiety 3. In apportioning candies and other nice
c. ascertain that the surgeon has prepared the goodies among children, what bioethical
client by explaining the proposed operative principle should a nurse observe?
procedure a. Justice
d. withhold all medications eight hours prior to b. Beneficence
obtaining the consent c. Non-malifescence
d. Respect

3. Principles behind prioritization of nursing 4. What is the nursing ethical guiding principle
care is: during invasive procedures being done to
a. Veracity children?
b. Beneficence a. Respect
c. Non-maleficence b. Beneficence
d. Fidelity c. Justice
d. Non-malifescence
5. A handicapped child like any normal child a. Rules that protect public interest and
should be loaded with: welfare
a. Toys b. Principles of conduct on becoming a nurse
b. Food c. Right ways of giving nursing care
c. Love d. Society’s norm in the practice of nursing
d. Care
3. When you start the practice of professional
Situation: Bioethics in caring for children nursing, you assume “ipso facto” obligations to
capitalizes on some important principles. You uphold the noble traditions of the profession.
are expected to know these by heart because You are expected to:
nurses care for children as well as for adults. a. Earn a living
1. What is the principle to hold on to when b. Safeguard public health
caring for children who have been abandoned c. Abide by the law
when abused? d. Fulfill your civic duties
a. Respect
b. Constant assurance 4. The scope of nursing practice based on RA
c. Autonomy 7164 includes the following responsibilities,
d. Allocation of resources except:
a. Interpreting results of sensitivity tests
2. The principle that tells children are to be b. Initiating Code Blue
treated accordingly as a person is this one: c. Suturing lacerations
a. Beneficence d. Care of women during pregnancy, labor and
b. Respect delivery
c. Justice
d. Autonomy 5. To develop independence if families in
health, which of the following is the best
3. When there is only one pacemaker and approach?
there are many patients needing it, the a. Family education for life
bioethical principle to be used is: b. Family empowerment
a. Beneficence c. Family health work
b. Respect d. Family health organization
c. Justice
d. Autonomy
7. CONFIDENTIALITY
4. When the nurse triage patient to render care 8. PRIVACY
accordingly, you are using this particular 9. INFORMED CONSENT
principle: Situation: Mrs. Diaz is assigned in the female
a. Respect surgical ward. While on duty, an 18-year old
b. Autonomy patient, married, was admitted for caesarean
c. Beneficence section. The informed consent operation has to
d. Allocation of resources be obtained.

5. In choosing sharp intramuscular needles for 1. The person legally responsible for taking the
injections, you are practicing this principle: informed consent is:
a. Beneficence a. The OR nurse who is going to assist during
b. Respect the operation
c. Justice b. The doctor assigned with the team.
d. Non-maleficence c. The doctor who is going to operate on the
patient
Situation: Ethical and legal responsibilities are d. The ward nurse where the patient stayed
part of your nursing practice. before the operation
1. Which of the following statements is true?
a. Ethics prescribe what are right and wrong 2. Mrs. Diaz has to remember the following
conduct about informed consent:
b. Ethics and morals are synonymous a. Because the patient is a minor, the parents
c. Morals as human conduct is also a legal right should be asked to sign the consent
d. An unethical act is immoral b. The informed consent should be signed
either by the patient or the husband
2. The Code of Nurses best describes:
c. The nurse has the main responsibility to c. Keep in the unit cabinet under lock and key
obtain the informed consent d. Discuss with her the hospital policy
d. When the consent form is signed by the
patient herself, the nurse should stand as a 3. The physician has an order to limit visitors
witness for your patient. One morning a relative
insisted to visit him. Your best response is to:
3. The medical intern who assisted in the a. Call the security guard
operations gave post-operative orders. In this b. Show him the “No Visitors” sign
case, Mrs. Diaz should: c. Explain the need for the patient to rest
a. validate the order from the surgeon and d. Allow a brief visit
request him to countersign
b. follow the order as long as they are within 4. It is a routine practice in the hospital to
the scope of nursing practice follow doctor’s rounds. One morning, a patient
c. clarify from the medical intern those that are with a dyspnea was for admission. The best
ambiguous action to take is:
d. refuse to follow the order because it is not a. Attend to the patient immediately
legal b. Inform the physician there is a patient for
admission
4. After one year, Mrs. Diaz was rotated to the c. Make the patient wait for a while
delivery room. As DR nurse, the obstetrician d. Ask a student nurse to admit the patient
ordered her to give spinal anesthesia because
the anesthesiologist was late. The nurse 5. These includes your responsibilities in family
should: nursing care, except:
a. give anesthesia if the supervising nurse a. Family planning counseling
approved it b. Monitoring child development
b. give the anesthesia if the obstetrician c. Screening for immunologic problems
supervises her d. Crisis intervention
c. give the anesthesia if the doctor writes the
order
d. not give anesthesia B. PATIENT’S BILL OF RIGHTS
5. Which of the following is NOT a legally
5. Which of the following should a nurse binding document but nonetheless very
remember in cases when doctor’s request important in the care of all patients in any
them to give anesthesia? setting?
a. The nurse has the right to refuse it, if the a. Bill of rights and provided in the Philippine
doctor’s order is unlawful constitution
b. Doctor’s orders should always be written b. Scope of nursing practice as defined on RA
c. The nurse may render medical procedure if 7164
the doctor supervises her c. Board of nursing resolution adopting the
d. The staff nurse can be accused of Code of Ethics
insubordination if she does not follow all d. Patient’s Bill of Rights
doctor’s order
Situation: In most instances, a Filipino does
Situation: Clients and patient in health care not know their rights as patients. To be a
have both ethical and legal rights. patient advocate, the nurses must have a good
1. When a patient wants to know the prognosis understanding of the Patient’s Bill of Rights.
of his illness, the best action to take is: 1. By listening carefully to the clients and their
a. Refer to your senior nurse relatives and explaining about what will
b. Advise patient to ask his doctor happen, why and when, the nurse is observing
c. Give a direct answer which of the following rights?
d. Discuss it with her the hospital policy a. Right to refuse treatment to the extent
permitted by law
2. Your patient resented your telling her to b. Right to expect reasonable continuity of care
take home her expensive belongings for c. Right to considerate and respectful care
safekeeping. What is the best action to take? d. Right to know what hospital rules and
a. Suggest that these be kept in her locked regulations apply to his conduct as a patient
table drawer
b. Bring the matter to the attention of the head 2. As the nurse in the prenatal clinic, you are
nurse able to protect the patient’s right to every
consideration of his privacy by observing which c. Penal code
of the following? d. Person’s Bill of Rights
a. In doing physical examination, the nurse
provides by applying the needed legging or 2. Mentally ill persons have specific rights
drape and a room or enclosed area based on:
b. The nurse accepts the patient’s desire not to a. Freedom of choice and self-determination
take experimental drugs and/or regimen b. Freedom from unreasonable search and
c. The nurse encourages patient to participate seizure
in planning her care c. Freedom of speech
d. The nurse does not impose any treatment if d. Right of privacy
the client refuses to it
3. Albert can be advised that his brother can
3. Which of the following is an emerging role of only be treated involuntarily if:
the nurse in her mission to provide quality care a. Nobody can take care of him at home
in any setting regardless of the type of a client b. He has suicidal tendencies
and meet her goal to contribute new c. He is dangerous to others
knowledge and technology in nursing? d. All of these
a. Nurse educator
b. Nurse executive 4. An important nursing concern for patients
c. Nurse manager involuntarily committed is:
d. Nurse researcher a. Developing a nursing care plan
b. Implementing medical orders
4. As member of the country, the Philippines is c. Basic nursing assessment documentation
committed to the worldwide movement of d. Adherence to legal constraints
“Achieving Health for All”, as patient’s
advocate, nurses must involve herself in 5. Which of the following patient’s rights can
activities that will focus on which of the be suspended for a valid reason?
following? a. Right to privacy
a. Lobby for increase budgetary allocation for b. Right to treatment with the least restriction
specialized hospitals and tertiary levels of care c. Right to freedom from physical restraint and
b. Delegate more nursing functions to support seclusion
and rehabilitate rather than promotive and d. Right to confidentiality of records
preventive aspects of care
c. Focus more of her services on curative and C. CODE OF ETHICS IN NURSING
rehabilitative rather than promotive and Situation: As a nurse, you are aware that your
preventive aspects of care actions have ethico-legal implications. Because
d. Emphasize on self-reliance and participation of this, you must always be alert of those
by the individual and community member of situations that demand an appropriate
health matters professional conduct.

5. Nowadays, funding for personal health care 1. Mr. Lacson is in the terminal stage of lung
comes from a variety of sources which the cancer. He tells you that he has a living will.
nurse must be knowledgeable about in order to You have known that the client’s living will
protect the rights of their clients to health care involves which one of the following?
services. Which of the following provides a. A document signed by the client’s family for
private health care services? the provider to withhold medical treatment
a. Employment compensation / health when death is eminent
insurance b. The medical treatment a client chooses to
b. Health maintenance organizations withhold if he is rendered unable to make
c. Preferred provider organizations decisions
d. Medicare / Philhealth c. It is confidential and not part of the client’s
medical record.
Situation: Albert’s brother is deciding whether d. Provides that the client’s wishes need not be
to commit him to a mental institution or not, followed if life can be prolonged
Albert refuses to cooperate.
1. The fundamental right of every individual 2. You understand that as a nurse, maintaining
emanates from: privacy and confidentiality is an important
a. Constitution of the Philippines aspect of which of the following?
b. Laws of the land a. Hospital policy
b. Nurse code of ethics 3. A patient, G8P5, refused to be injected with
c. What all client expects her 3rd dose of Depoprovera. The nurse insisted
d. It is an accepted practice inspite of the patient’s refusal and forcibly
injected the contraceptive. She can be sued for
3. Health care issues often become an ethical which of the following?
dilemma because of which of the following? a. Misrepresentation
a. Decision must be made quickly often under b. Assault
a stressful condition. c. Malpractice
b. Decision must be made based on individual d. Negligence
systems
c. The choices involved do not appear to be 4. A patient has been in the ICU for 2 weeks.
clearly right or wrong The relatives have been consented to a “Do not
d. A client’s legal rights co-exist with a health resuscitate” order. When the patient develops
profession a cardiac arrest, the nurse will carry out which
of the following actions?
4. A basic structure against which competent a. Only medication will be given
care is objectively measured is which of the b. All ordinary measures will be stopped
following? c. Basic and advance life support will not be
a. Principle given
b. Standard d. mechanical ventilation and NGT will be
c. Law stopped
d. Codes
5. When a patient falls from bed, which of the
5. Which of the following principles underlying following is your most immediate action?
ethical code of professional nurses is a. Report to the head nurse and call someone
considered when one tells the truth about to help
medication errors committed? b. Determine any injury or harm
a. Privacy c. Refer to the resident on duty
b. Veracity d. Put back patient to bed
c. Malificence
d. Beneficence Situation: Professional nursing has Code of
Ethics that spells out right conduct in nurses.
Situation: One important fact that will guide Each nurse should have a system of values
the nurse in the practice of the profession is that demonstrate ethical behaviors in practice.
her knowledge of the nursing law. 1. Which of the following values is paramount
1. The nurse practice act of 1991 regulates the in the ethical practice of nursing:
practice of nursing in the Philippines. Which of a. The welfare of the patients is ultimate
the following statements about this Act is NOT b. Every nursing activity has an ethical aspect
true? c. Nursing is a safe practice
a. This act delineates the practice of nursing d. The purpose of nursing is to promote well-
and midwifery being
b. It was enacted in November 1991
c. The primary purpose is to protect the public 2. In your nursing practice, the desire to
d. This Act defines the practice of nursing in choose good actions is mainly based on:
the Philippines a. PNA Code of Ethics for Nurses
b. International Council for Nurses Standards
2. When a nurse starts working in a hospital c. Board of Nursing Standards of Conduct
but without a written contract, which of the d. Nursing Education Ethical Standards
following is expected of her?
a. She is not bound to perform according to 3. This is the best course of action when faced
the standards of nursing practice with ethical difficulties:
b. Provides nursing care within the acceptable a. Discuss with a lawyer
standards of nursing practice b. Assess the ethical problem
c. She is not obliged to provide professional c. Reflect on your conscience
service d. Refer to a spiritual counselor
d. The employer does not hold the nurse
responsible for her action 4. In the final decision, your ethical actions are
most influenced by your:
a. Ethical knowledge
b. Conscience Situation: The Regional Training Nurse must be
c. Norms of society qualified before assuming the position.
d. Religious beliefs 1. What is the basic qualification of the public
health nurse at the Regional Office?
5. Based on the RA 7164, this is a ground for a. Three years abroad
suspension of registration certificate: b. BSN, RN with master’s degree
a. Libel c. Six years as CI
b. Negligence d. Relative of the governor
c. Sloppy work
d. Dishonesty 2. What is the qualification of the public health
nurse instructor?
a. recommended by the mayor
V. LEGAL RESPONSIBILITY b. BSN, RN with masters degree
c. experience as CI
d. experience in PHN
A. LEGAL ASPECTS IN THE PRACTICE OF
NURSING 3. Functions and responsibilities of the regional
Situation: Miss Javier is being interviewed by training nurse begins with the use of the
the Chief Nurse in relation to her application in nursing process. Which process does she need
tertiary hospital as staff nurse. to design a training program for nurses and
1. When asked whether nursing is a profession midwives?
or not, she quoted one authority two holds as a. assessment
the view that a profession is characterized by b. evaluation
the following: c.. intervention
a. It is highly skilled and purely technical d. planning
b. It demands specialized preparation and
training 4. Even if the nurse is already in the highest
c. It is determined by the compensation of its position, there is need to keep abreast. Which
members area of CHN should a nurse train or retrain?
d. It is a developing science a. community organizing
b. matrix of hospital administration
2. The chief nurse explained to Miss Javier that c. model of health care
the philosophy of nursing in the tertiary d. use of high tech machines
hospital is consistent with the generally
accepted definition of nursing which is: 5. Which one should a PHN retain as nursing
a. Caring for the sick only because the “well” value in her community health practice?
parents are taken care of by primary hospitals a. Spiritual
b. Putting the patient in the best condition for b. Good Will
the nature to act c. Human Relations
c. Caring for the sick and well d. Academic
d. Allowing the patient to determine the
medical and nursing plan of action Situation: PHC utilizes linkages all over the
world.
3. The chief nurse further explained to Miss 1. Who receives referral from intermediate
Javier that of the many functions of the nurse, level of care?
the only one dependent or interdependent a. chief nurse
function is: b. first line hospital personnel
a. Application and execution of doctor’s orders c. cardiologist
b. Application and execution of nursing d. medical director
procedures and techniques
c. Observation of signs and symptoms 2. When first line hospital workers are done
d. Supervision of patient and those with severe dehydration, patient goes home to
participating in his care your level of care. In what way could you
provide support?
a. Linkage with public works
b. Bring patient for check-up
B. THE PHILIPPINE NURSING LAW OF c. Refer clients with same problems
2002 (RA 9173) d. Give IVs in case of repeat dehydration
3. The dual role of PHC workers area a
community health care developer and 4. The following are qualifications of chief
community development. How could you link nurse in city health departments and health
and provide care at the same time? offices:
a. Health direction for better tomorrow I. Master’s degree in Public Health,
b. Health care available even with high cost Major in Community Health Nursing and
c. Health care procedure with supplies Administration
d. Health care accessibility with low cost II. Doctorate degree in Public Health
III. 3 years in supervisor position
4. PHC is a system which emphasizes well IV. 2 yaers in the Asst. Chief Nurse
being of all segments. What is the result of this position
effort? a. I and II
a. Progressive well-being b. II and IV
b. Progressive nation-building c. I, II and III
c. Economic development and technology d. I, II, III and IV
d. Economic progress and self-reliance
5. The following are the functions of the
5. PHC activities are fully integrated with other Assistant Chief Nurse:
sectors. Which sector is always involved? I. Assesses needs for community
a. Industries health services as part of total health program
b. Politician II. Evaluates the quality and effectivity
c. Congress of nursing services in relation to the total
d. Public works health program
III. Coordinates community health
Situation: Mrs. Fuentes was promoted as a services with other health and socio-economic
nurse supervisor progress
1. The following are the qualifications of Nurse IV. Supervises, coordinates and
Supervisor at the provincial level except: evaluate the performance of the District Nurse
a. BSN, RN Supervisors and assist them in the evaluation
b. At least 3 years experience of community health nurses
c. Masters in Public Health a. I and III
d. At least 5 years experience in Public Health b. II and IV
c. I, II and III
2. The following are the functions of Provincial d. IV only
Nurse Supervisors, except: Situation: The Nurse Practice Act of 2002 or RA
a. Interprets policies, guidelines and SOP to 9173 was passed and signed into law in
nursing and midwifery staff of the province October 2002.
b. Assesses training needs and plans for staff 1. Among the important provisions was the
development program for nursing and composition of the Board of Nursing. Which of
midwifery staff the following is provided for in RA 9173?
c. Participates in planning, developing and a. The Board of Nursing shall be composed of
evaluating OJT for nurses and midwives seven members
conducted by the department b. Have at least 10 years of continuous
d. Collects, consolidates, analyzes and practice of the profession
interprets health record and reports and makes c. The qualifying age should not be more than
recommendations as needed 70 years of age
d. The academe qualification of the members
3. The following are requirements and shall be a holder of Master’s degree in Nursing
qualifications of regional nurse supervisor: or related fields
I. BSN, RN
II. With at least 5 years experience in 2. Which of the following is an additional
CHN function of the Board, which is a departure of
III. With MAN major in CHN RA 7164?
IV. With 2 years experience in a. Promulgate decision for the improvement of
supervisory position nursing
a. I and III b. Describe the subjects in the licensure
b. II and IV examination
c. I, II and III c. Recommends the opening and closure of
d. I, II, III and IV Colleges of Nursing
d. Issue, suspend, revoke certificate of a. Psychology
registration b. Nursing only
3. Which of the following activities is c. Public health
considered to be expanded scope of nursing d. Nursing or relate field
practice?
a. Conducting health education classes 4. Nurses in the Nursing Service must also be
b. Teaching in Colleges of Nursing qualified. The nursing service director must
c. Counseling and doing research have:
d. Advance nursing practice a. Master’s degree in any field of nursing
b. Master’s degree in health sciences
4. The following nursing positions require that c. Master’s degree in Nursing major in
the nurse must be a holder of Master’s degree Administration/Management
of Nursing: d. Master’s degree in Public Administration
I. Dean of the College of Nursing
II. Chief Nurse of a Hospital 5. To be qualified, the nurse supervisors in the
III. Instructor in a College of Nursing hospital must at least have:
IV. Supervisor of a general hospital a. Master’s of Public Health
a. I and II b. Nine units in Master’s degree in Nursing
b. I and III c. Nine units in Management and
c. II and IV Administration course at graduate level
d. II and III d. Master’s of Arts in Nursing

5. Who among the following is not qualified to Situation: The Bachelor of Science in Nursing
practice nursing in the Philippines under the (BSN) program prepares you to be competent
special or temporary permit? in first level of nursing positions.
a. An American registered nurse who is part of
the medical team invited by the Basilan 1. As required by RA 7164, students admitted
governor to perform cleft palate operation in to a nursing school belong to what percent of
the province for free their high school graduating class.
b. A Filipino-American nurse who is part of the a. 60
medical team of a brain surgeon who is in the b. 50
country upon invitation to operate on a well- c. 40
known oligarch d. 70
c. A licensed nurse from Canada who is in a
medical mission to work as a volunteer nurse 2. As provided by RA 7164, the authorization
in the ER of PGH for one month to open a nursing school shall be based on:
d. A certified nurse who will work as a guest a. Only the CHED
lecturer in the training of organ transplantation b. Permit to operate issued by the Commission
care. on Higher Education (CHED)
c. Professional Regulation Commission and the
Situation: The Nursing Act of 2002 or RA 9173 CHED
has a provision that the nurse must continue d. Joint recommendation of Board of Nursing
learning. and CHED
1. Which of the following is formal continuing
education? 3. The Bachelor of Science in Nursing
a. Tele conference curriculum is:
b. Enrollment in graduate school a. Competency-based and community-oriented
c. Attendance in conference b. A step-ladder curriculum
d. Refresher course c. An alternative health science program
d. Focused on related learning experiences
2. Which of the following is the required
qualification of the Dean? 4. As a BSN graduate, you accepted a teaching
a. Nine units in Nursing Administration position in a nursing school. You can be liable
b. Master’s degree in related fields for illegal practice because:
c. Master’s degree in nursing a. You are not qualified based on RA 7164
d. Doctoral degree b. You are not a nursing specialist
c. You may commit an error in teaching
3. The faculty of a College of Nursing must d. You are not competent to teach
have a Master’s degree which is:
5. Based on the Implementing Rules and Situation: RA 7164 provides that all successful
Regulations of RA 7164, nursing faculty who examinees in the nursing licensure
are not master’s in nursing graduate should examination should be certificate of
finish the required degree in school year: registration holders.
a. 1998-99 1. Registration after passing the nursing
b. 1996-97 licensure examination is given by the
c. 1999-2000 government as:
d. 1997-98 a. A privilege
b. A right
Situation: Mr. Cruz, 60 years old, is scheduled c. An award
for laparotomy the next day. The nurse in the d. A recognition
afternoon shift advised him he would be on
NPO after midnight. On waking up the next 2. Your certificate of registration can be
morning, he saw his breakfast and ate it. His revoked or suspended for which of the
operation was cancelled. following acts:
1. In this particular situation, who is to be a. Gross incompetence
blamed: b. Illegal practice
a. The patient himself c. Practicing without a license
b. The nurse in the afternoon shift d. Unauthorized absence from work
c. The attendant for bringing in his breakfast
d. The nurse in the night shift 3. You submitted together with your nursing
licensure examination application form a fake
2. Mr. Cruz’s operation was rescheduled, IV Related Nursing Experience document. You are
infusion was ordered and you started it. Based liable to which of the following illegal acts:
on RA 7164, an RN can administer IV injections a. Forgery of documents
provided that he/she: b. Incompetence
a. Has practiced nursing for at least a year c. Malrepresentation
b. Has completed IV prescribed training d. Falsification of documents
protocol
c. Is supervised by a licensed physician 4. You received a summon from the Board of
d. Is a BSN graduate Nursing for complaints of unethical conduct.
You failed to appear. You are liable for:
3. A requirement to practice IV therapy is a a. Sanction
training certificate of completion certified by b. Contempt
the: c. Defiance of authority
a. Association of Nursing Service d. Incompetence
Administrators of the Philippines
b. Council of Continuing Professional Education 5. You failed to renew your nurse registration
for Nurses for more than five (5) years. You can be:
c. Philippine Nurses Association a. Summoned by the Professional Regulation
d. Board of Nursing Commission
b. Charged with unprofessional conduct
4. The attending doctor of your patient c. Delisted from nurse registry
questioned why you started the IV infusion. d. Liable for malpractice
Your best response is:
a. “There was a physician around when I did Situation: RA 7164 declares that it is the
it.” policy of the state to guarantee the delivery of
b. “This is my IV Nurse ID card.” basic health services through an adequate
c. “The Nursing Practice Act of 1991 allow RN’s nursing personnel throughout the country.
to practice IV.” 1. In relation to quality nursing education, the
d. “You ordered it.” board of Nursing ensures that the nursing
schools
5. When you initiate IV infusions, which of the a. Meet standards of nursing practice
following should you watch out for: b. Graduate only competent students
a. Comfort of the patient c. Should be accredited
b. Untoward signs of the infusion d. Comply with school requirements
c. When to start the next infusion
d. Out-of-vein signs 2. To guarantee quality nursing education, an
applicant to the licensure examination should:
a. Belong to the upper 40% in high school 4. The following nurse’s notes should be
graduates charted after an IV injection, except:
b. File all requirement for licensure a. The doctor who prescribed
c. Possess qualities for safe nursing practice b. Number of IV drops/minutes
d. Present evidence of competence c. Time, IV, drugs and dose
d. Any complications noted
3. Nursing licensure tests require official
transcript of records from the education 5. The established IV protocol requires
department, except: certification of proficiency issued by:
a. Graduates from foreign schools a. Chief Nurse of Hospital
b. Graduates of Level III accredited schools b. Association of Nursing Service
c. Applicants for reexamination Administrators
d. When record of related learning experience c. IV trainers
is complete d. Board of Nursing

4. Nursing schools should provide learning C. RELATED LAWS AFFECTING THE


experiences that contribute to the development PRACTICE OF NURSING
of the:
a. DOH health programs 22. The Magna Carta of Public Health Workers
b. Community student laboratory is:
c. Country a. RA 2382
d. School Program b. RA 7277
c. RA 3573
5. This is a specific provision for nursing d. RA 7305
schools:
a. Provision for community learning experience Situation: The legal responsibilities of the
b. Evidence that the school follows educational nurse in any setting must be given importance
standard to prevent litigation.
c. A 3-year college of nursing development 1. Which among the following is related to the
plan local government of delivery of basic services
d. Community based curriculum including health?
a. Executive Order Number 119 Section 3
Situation: The Standard Committee of Hospital b. Ministry Circular Number 2
E recommended that all Intensive Care Nurse c. RA 7160
be competent in IV injections. d. RA 7164
1. RA 7164 provides that IV injections can be
administered by nurses under the following 2. The Dangerous Drug Act known as RA 6425
conditions: states that nobody must do any of the
a. After completion of the BSN program following in relation to prohibited drugs?
b. After doing at least 20 IV injections a. Report suspected user to authority
c. On completion of a special IV training b. Sell, administer and prescribe
program c. Use yellow prescription for doctors
d. Under supervision of the physician d. Sell, administer and deliver

2. Precautions must be strictly observed when 3. Which age group is required to be


giving IV injections. Why is this so? immunized as per presidential decree no. 996?
a. Any IV injection is dangerous a. Children with special needs
b. IV entails skill is insertion b. Children below 8 years
c. IV is a delegated function of the doctor c. Out of school youth
d. Action of IV injection is invasive d. School children

3. As required by RA 7164, an IV training 4. Which legislation aims to promote and


protocol must be established by: improve the social and economic well being of
a. Philippine Nurses Association the workers in government services?
b. Medical protocol a. RA 7164
c. Association of Nursing Adminstration in the b. DOH policies and guidelines
Philippines c. Alma ata
d. Board of Nursing d. Magna Carta
5. The presidential decree that requires all 1. Among the overall goals of most hospitals
health workers to register all births within 30 which is an exception:
days? a. Take care of the sick
a. PD 615 b. Care of people with special needs
b. PD 651 c. Provides preventive services
c. PD 561 d. Serves health needs of people
d. PD 156
2. What is the purpose of an organizational
structure?
a. Provide a mechanism for work distribution
b. Explain lines of authority
VI. MANAGEMENT OF ENVIRONMENT AND c. Reflects mission and goals of the unit
RESOURCES d. Provides a written plan of organization
A. THEORIES AND PRINCIPLES OF
MANAGEMENT 3. Which of the following documents would be
Situation: As a nurse-in-charge, you noted that most helpful to a new staff nurse:
a lot of confusion occur in your unit due to a. Hospital physical plant
some disorganization. You decided to improve b. Job description
work performance. c. Policies and procedure
1. A deliberate course of action that results in d. Organizational chart
alteration of something different in a nursing
unit is referred to as: 4. In your new job, the least important is:
a. Planned change a. Organizational chart
b. Improve systems b. Policies and procedures
c. Re-education c. Physical tour of the hospital
d. Nursing process d. Job description

2. When resistance threatens needed change, 5. The most important benefit of the
what important measure would you use to orientation program is:
overcome the resistance: a. Know employment working conditions
a. Send a memorandum for change b. Builds employee’s identification with
b. Use a problem-solving strategy organization
c. Gain early support from staff c. Learning about the work setting
d. Conduct staff re-education d. Discuss your nurse’s roles

3. This is a factor most influencing acceptance


of staff in change: B. NURSING ADMINISTRATION AND
a. Externally imposed MANAGEMENT
b. Share its benefit Situation: You considered Mrs. Santos, chief
c. Staff participates in decision nurse of our hospital, a traditional and
d. Planned ahead bureaucratic leader.
1. The following statements about bureaucratic
4. The first step in the change process is approach to management are true, except:
facilitating change. This is called: a. Rules and regulations are emphasized
a. Refreezing b. The Chief nurse is comptroller
b. Unfreezing c. Decision giving is participative
c. Moving d. Specialization of task is formal
d. Status quo
2. What is the most basic principle in the
5. These are important qualities of a change human relations approach to management?
agent, except: a. When things go well for the worker, the
a. Leadership skills organization profits
b. Sociability b. When the patient needs are met, nursing
c. Credibility missions are accomplished
d. Management skills c. Strength, speed and skill are accentuated
d. Decision-making is a mediator
Situation: Orientation involves the introduction
of new employees to nursing service.
3. As a staff nurse under a traditional chief d. Sorry Doctor, but I am also accountable for
nurse, notwithstanding, you are most expected my patient.”
to:
a. Maintain professional attitudes 4. After the performance appraisal conducted
b. Observe hospital rules and regulations in the unit, your head nurse cites a long list of
c. Adhere strictly to your job description behaviors needing improvement. Which of the
d. Maintain competence following statement is most applicable to
primary nursing?
4. When you receive different directions from a. Promote sharing of ward resources
the doctor and senior nurse, this situation is b. Make a nurse accountable and reasonable to
referred to as: a group of patient
a. Role difference c. Implies ward-manager responsibilities
b. Role confusion d. Establishes a good nurse-doctor relationship
c. Role compliance
d. Role conflict 5. You suggested to your supervisor the use of
primary nursing care method in the unit. Which
5. The role of nurses has expanded. of the following statement is most applicable to
Acceptance of your role depends on all primary nursing?
conditions, except: to primary nursing?
a. State regulations a. Promote sharing of ward resources
b. Acceptance of clients / patients b. Make a nurse accountable and reasonable to
c. Acceptance of physicians a group of patient
d. Acceptance of nurses c. Implies ward-manager responsibilities
d. Establishes a good nurse-doctor relationship
Situation: Your head nurse called that she
cannot report that morning because of an
emergency that happened in the family. You C. THEORIES, PRINCIPLES AND STYLES OF
were designated as acting head nurse by the LEADERSHIP
supervisor. Situation: Nurses encounter various types of
1. You do now welcome the assignment but situation that calls for management decision-
under the circumstances what is the best thing making
to do? 1. Under varying conditions, the best approach
a. Take an emergency leave to management is:
b. Recall the staff nurses who are off duty a. Management by situation
c. Utilize the team nursing method of b. Management by policy
assignment c. Management by goals
d. Explain to the patients the situation d. Management by objectives
regarding understaffing
2. Management of patient care with varying
2. You had no choice but to delegate some problems:
nursing tasks to the ward attendant. In a. Entails problem-solving
assigning the tasks, which of the following b. Involves therapeutic use of self
should you consider most? c. Requires adaptation to circumstances
a. Performing the head nurse role according to d. Depends on one’s style of leadership
standards
b. Making sure that the attendant is capable of 3. Nursing management goals are best
doing the task achieved by which of the following:
c. Supervise her if necessary a. Manager and staff nurses working together
d. Let her make a written report in case of b. Discussion of work with staff
error c. Selection of competent staff
d. Knowing work’s schedule
3. You noted that a medication dose prescribed
for a patient is unusually large. You verified 4. An effective manager is:
this with the doctor who got angry. Which of a. Situation-oriented
the following is the best response? b. Work-oriented
a. “Doctor, I know my pharmacology.” c. Goal-oriented
b. Doctor, I refuse to give medication unless d. People-oriented
clarified.”
c. Doctor, I am just verifying the order.”
5. This is a major characteristic of permissive situation”. How can management by situation
leadership: be best accomplished?
a. Uses guidelines for group activity a. By personally selecting the nurse to be
b. Believes all staff are work-oriented assigned under her
c. Collaborates with staff in nursing care plans b. By discussing work situation regularly with
d. Allows staff how to get their work done the immediate supervisor
c. By obtaining the work schedule one week in
Situation: Your co-staff has been promoted to advance
the position of senior nurse. d. By bringing the management forces in her,
1. Which is a responsibility of first line the individual capabilities of the team leader
managers? and the situation as closely aligned as possible
a. Oversees operations of nursing units
b. Coordinate with various units 3. Which of the following is LEAST required for
c. Focuses on actual nursing services an effective leadership and management?
d. Represents organization in the community a. Knowledge of basic leadership and
management theory
2. As a manager, your co-nurse first b. A well prepared supervisor and a good
responsibility is directed toward unit: planner
a. Patient classification c. Self-understanding
b. Work load d. Systematic use of self
c. Resource allocation
d. Organizational structure 4. Which of the following LEAST describe
leadership style?
3. The most important category to consider in a. How a manner works with followers
unit management is: b. It is a form of behavior
a. Patient, nurses, administration c. The way in which something is said or done
b. People, facilities, money d. A role or title
c. Realization of goals
d. Doctors, nurses, patients 5. Which of the following does NOT describe
the goals of a nurse manager?
4. You were assigned as team leaders for the a. To select a style of leadership that looks
care of 10 patients in the pediatric unit. Your best for her and practices it exclusively
initial activity will be to: b. To use a style of leadership that best suits
a. Determine abilities of staff the task to be accomplished
b. Give direction for care c. To use a style of leadership that creates the
c. Organize the work load best out of disruptions and conflict
d. Identify patient’s needs d. To use a style of leadership that promotes
high level work performance
5. When you examine individual and group
performance, you are performing a function of: Situation: After three years of working as a
a. Directing staff nurse in a medical ward, Miss Fernando
b. Planning was promoted as a head nurse of a new unit
c. Organizing on the basis of the leadership skills she had
d. Controlling demonstrated with her co-nurses.
1. As a good leader, Miss Fernando has most
Situation: Ms. Lopez is assigned as team likely demonstrated that she can:
leader in charge of 20 patients in a medical- a. Influence her co-workers to work for the
surgical unit achievement of the goals of nursing service
1. As a team leader for the day, her initial b. Always follow instructions as mandated by
activity will be which of the following? the hospital administration
a. Evaluate the abilities of the staff c. Expect pressure upon her co-workers to
b. Organize the work load achieve objectives
c. Give direction to the 3 nurse assigned under d. Be subservient to nursing authorities in the
her hospital
d. Obtain patient assignment
2. Miss Fernando and her current staff in the
2. When assigned as team leader, Ms. Lopez ward agreed together on their goals and
thought that she would carry out her function targets. She is managing by:
utilizing the principles, “Management by a. Contingency
b. Values 4. In simple terms, planning involves deciding
c. Exception in advance the following, except:
d. Objectives a. What and how to do it
b. When to do it
3. Miss Fernando was tasked to decide on the c. Where to report
number of staff nurses she would need to d. Who is to do it
provide quality nursing care in the unit. The
specific tasks would require which of the 5. In the nursing unit, the main aim of
following skills? planning is:
a. Communication skills a. Achieve set nursing goals
b. Managerial skills b. Describing organizational structure
c. Cognitive skills c. Deciding staffing of a unit
d. Evaluative skills d. Setting criteria for quality nursing

4. Miss Fernando is assigned two new team D. CONCEPTS AND PRINCIPLES OF


leaders for one month. As a team leader, the ORGANIZATION
initial activity would be which of the following? Situation: To facilitate effective administration
a. Document all nursing care rendered by the and management of nursing service of any
staff health care facility, a nurse must be
b. Evaluate the performance of the staff knowledgeable of the institution as an
c. Organize the workload organization.
d. Get the patient assignment
1. To guide the nurse in the professional
5. After one month, Miss Fernando was relationship with other department she must
evaluated by supervisors and was found to be aware of the organizational structure of her
have obtained 80% attainment of her goals institutions. The purpose of an organizational
and targets. When asked about her structures to:
management style, she answered, “None in a. Have a written plan of organization to share
particular but deal with the situation as it with the governing bodies
arises”. This implies that she is leading by: b. Explain to each member the line of authority
a. Objectives c. Inform all staff members of the philosophy
b. Laissez faire and goals of the institution
c. Contingency d. Provide the mechanism through which tasks
d. Consultation are distributed among members for goal
realization
Situation: The role of the leader is both
challenging and rewarding. 2. One of the things that Nurse A noticed when
1. Which of the following terms does not relate she entered the office of the Chief Nurse is the
to leadership? hospital’s organizational chart. Formal
a. Goals authority in the organizational chart is
b. Influence indicated by a:
c. Motivation a. Broken line
d. Authority b. Box
c. Straight line
2. When a head nurse relates to people d. Rectangle
according to the roles, she is referred as a/an?
a. Authority 3. Ms. Nelia Hizon has recently been
b. Leader designated as adviser on nursing matters to
c. Manager the secretary of the Department of Health. Ms.
d. Decision-maker Hizon’s relationship to the secretary is
indicated in the organizational chart is
3. This is generally an accepted concept of indicated by a:
management, except: a. Rectangle
a. Getting things done through people b. Straight line
b. Influencing people to do the right things c. Box
c. Directing others to achieve set goals d. Broken line
d. Telling people what to do
4. Unity of command in an organization
denotes which of the following?
a. There should be one goal at a time to
prevent conflict E. PATIENT CARE CLASSIFICATION
b. Staff nurses work with other nurses to Situation: Miss Gregorio is a head nurse in the
accomplish one goal ward where there are fourth year students
c. Subordinates like staff nurses are allowable affiliating for their RLE in management.
to only one person
d. Staff nurses reports to two supervisors in 1. The clinical instructor of the students usually
one department assigned two patients for each. The students
are responsible for the 24 hours nursing care
5. The hospital where you are working is plan of each of the patient assigned to them.
celebrating its Golden Jubilee Foundation. This method of nursing care delivery is called:
Three of the nurse supervisors were assigned a. Team approach
by the overall chairman to head different b. Case management
committees. The principle of management c. Functional method
used here is: d. Primary nursing
a. Delegation of responsibility
b. Channel of communication 2. Regardless of the method of nursing care
c. Unity of command delivery, the head nurse is still accountable for
d. Command responsibility all the nursing care of the patients in the ward.
This is an application of the principle of:
Situation: Your head nurse participated in a 2- a. Scalar Principle
week seminar. You were designated to be b. Unity of Command
officer-in-charge of your nursing unit. c. Proper channeling of communication
1. In unit planning, which of the following d. Command Responsibility
questions initiates the process?
a. How to schedule activities? 3. Every morning, the head nurse confers with
b. Who will do the planning? the staff for 10 minutes to organize the work
c. What is to be accomplished? for the day. Which of the following sequence of
d. Where to get unit supplies? structuring task is correct?
I. assign tasks to the team
2. This is an example of data that should be II. present to the team the needs of
analyzed when identifying nursing needs the patients for the day
a. Number of nursing staff III. match the task with the abilities of
b. Medical diagnosis of patients each staff members
c. Ward supplies and materials IV. ask for suggestion from the staff
d. Patient classification a. III, II, I, IV
b. IV, II, III, I
3. To accomplish the nursing unit work, you c. II, IV, III, I
should first get information about: d. I, II, III, IV
a. Objectives of the unit
b. Nursing service organizational chart 4. One day, one of the members of the staff,
c. Nursing budget committed a medication error. Per hospital
d. Mission of the hospital policy, Miss Gregorio was to prepare an
incident report. In making an incident report
4. This is an example of a patient care unit the head nurse should ensure that her report:
objective: I. includes all pertinent facts regarding
a. To ensure adequate unit supplies/materials the incident
b. To use the Kardex System of charting II. names all involved practices in the
c. To determine the number of nursing staff incident
needed III. includes her appraisal of the
d. To provide safe nursing care incident
IV. is properly signed by all parties
5. Which of the following tools can be most concerned
helpful in the evaluation of staff performance? a. I, III and IV
a. Performance assessment b. I and II
b. Performance standards c. I and III
c. Performance audit d. I, II and IV
d. Performance appraisal
5. It is the policy of the Nursing Service 3. Make her write an incident report
Department that every six months, staff nurses 4. Verify from other nurses why it was
are evaluated. One nurse felt that she was not charted
unfairly evaluated by Miss Gregorio. The right
action to take is: a. 1, 2 and 3
a. Share the feelings with her peer group. b. 2, 3 and 4
b. Keep quiet and respect the head nurse’s c. 4, 1 and 2
decision. d. 4, 2 and 3
c. Clarify from the head nurse that basis for
her appraisal. 5. The morning supervisor was visiting the
d. Complains to the Chief Nurse of the hospital ward when the doctor of a patient complained
about staff nurses who failed to give the “stat”
Situation: The Nursing Service Director of a medication. Applying the principles of ‘line of
leading hospital in Manila wants all her patients authority”, you expected the supervisor to do
to get the care they rightfully deserve. With which of the following first?
the permission of the hospitals board, she a. Talk to the headnurse regarding the doctor’s
asked one of the best head nurses to organize complaints
a unit in such a way that patients are b. Ask the staff nurse to explain the incident
categorized according to the severity of illness. c. Report the incident to the chief nurse
She wants this unit to be the role model of d. Reassure the patient that next time the staff
quality patient care. The health personnel were nurse will not miss the medication
assigned in a manner that they meet the needs
of the patient accordingly.
1. Which of the following concepts can be used F. NURSING CARE SYSTEMS
in classifying the patients in the said unit?
a. Traditional
b. Combined traditional and modern
c. Progressive patient care G. DELEGATION AND ACCOUNTABILITY
d. Modern Situation: As a registered nurse you will
assume accountability for your nursing actions.
2. Which of the following is the BEST reason 1. Which of the following statements best
for classifying patients according to the describe accountability?
severity of illness from the nursing viewpoint? a. Your employer has command responsibility
a. Nursing care of clients is better planned over you
b. Professionals and non-professionals are b. You are liable to court suits
delineated c. You are responsible to your employer
c. Staffing requirements are met d. You yourself is answerable for your actions
d. Overtime work is avoided 2. How do you further develop your nursing
practice skills? By:
3. In utilizing her knowledge of management, a. Giving direct care to clients/patients
the headnurse as the leader of the unit is b. Correctly carrying out your assignments
expected to first? c. Attending performance appraisal meetings
a. Formulate with her members policies to be d. Regular membership to a specialty nursing
followed in the ward groups
b. Evaluate the care rendered to clients
c. Plan with members the nursing care of 3. The primary skills you have to demonstrate
patients when carrying for your client is:
d. Delegate tasks to be done by her members a. Perform correctly nursing procedures given
b. Establishing good nurse-patient relationship
4. As charge nurse of one of the teams, you c. Utilizing the nursing process
are responsible to check the charts of the d. Consider needs of your patients
patients assigned to your team. You noted that
a STAT medication that you know was given 4. In nursing practice, you bear the
was not recorded. You are expected to do the responsibility for:
following in order. a. Maintaining hospital’s good image
1.Call the attention of the medication b. Safeguard of your patients
team c. Living an ethical and moral life
2.Let her explain reason why it was not d. Your professional growth
charted
5. When do you say that a profession has b. What is your latest nursing experience?
autonomy: c. Is it alright with you to work in a
a. When it regulates itself and set standards understaffed unit?
for its members d. What is nursing?
b. When it has code of ethics
c. When it regulates itself 25. You are a new staff nurse in St. Mary’s
d. When it is independent Hospital and you observe that the medical unit
is understaffed. What will you do?
Situation: You have five patients in Ward B. A a. Observe for 1-2 weeks how nursing care is
student nurse is also assigned to your patients. going on
b. Ask the hospital director to hire additional
1. Who is directly responsible for the safe care nurse
of your patients? c. Decide to transfer to another hospital
a. The student assigned to the patients d. Pull out another nurse from another unit
b. You as the staff nurse
c. The headnurse Situation: You observed that most of the staff
d. The clinical instructor leave late after their duty hours. You looked
into staffing needs of the unit to propose
2. Which of the following is true? changes.
a. Students are accountable to their clinical 1. Staffing refers to:
instructor a. Describing job responsibilities
b. Students are liable for errors in practice b. Filling work force requirement
c. Students cannot be held liable for their c. Recruitment and selection of nurse
errors in practice d. Establishing qualifications for employment
d. Students are accountable to the head nurse
2. Which is the more valuable resource of an
3. One of your students was requested by a institution?
doctor to assist in an abortion procedure. Your a. Personnel
best action to do is to: b. Materials
a. Leave the student to decide for herself c. Budget
b. Tell him “no” as she is still a student d. Structure
c. Refer him to the head nurse
d. Tell him that abortion is not legal in the 3. The most important factor to consider in
Philippines staffing is:
a. Nurse-patient ratio
4. One student sought your advice regarding b. Nursing time needs
the patient who confidentially told her he killed c. Patient unit census
someone. Which is the best to note? d. Chief nurse decision
a. Keep information to yourself.
b. You may disclose information after patient’s 4. These are major factors on staffing plans,
death except:
c. If summoned in court, you disclose a. Type of illness sources
information b. Patients needing direct care
d. This is privileged communication hence c. Staff assignment
cannot be disclosed anytime d. Competence of staff

5. If your student has made an error in 5. These statements on loading are true:
medication, what would you first do? a. Loading must achieve organizational goals
a. Report to headnurse at all cost
b. Keep to yourself and the student b. Satisfaction and motivation are different
c. Call student attention at once c. No single motivation sufficiently explain
d. Check any harm to the patient human behavior
d. There are no average person just unique
24. You are one of the nurses assigned in the ones
nursing service department and responsible to
interview nurse applicants. In interviewing an
applicant, what is the appropriate question to VII. RECORDS MANAGEMENT
ask? a. Anecdotal report
a. Are you willing to go on 12 hours duty? b. Incident Report
c. Memorandum Situation: As a nurse, you are aware that
d. Hospital manual proper documentation in the patient’s chart is
e. Documentation a legal responsibility.
f. Endorsement and end-of-shift 1. A nurse gives a wrong medication to a
report client. Another nurse employed by the hospital,
g. Referral as a risk manager, will expect to receive which
Situation: Nurses use a variety of ways to of the following communications?
communicate information about clients. The a. Incident report
more nurses know about a client, the better b. Oral report from the nurse
prepared they are to render high quality care. c. Copy of the medication Kardex
1. Which of the following are general guidelines d. Order changed signed by the physician
that must be followed ensure quality
documentation and reporting? 2. Performing a procedure on a client in the
1. Factual and accurate absence of informed consent can lead to which
2. Complete and current of the following charges?
3. Organized and confidential a. Fraud
4. Computerized and coded b. Harassment
a. 2, 3 and 4 c. Assault and battery
b. 1, 2 and 3 d. Breach of confidentiality
c. 1, 2 and 4
d. 1, 3 and 4
5. A nurse is witnessing consent from a client
2. Which of the following is appropriate before a cardiac catheterization. Which of the
purpose/use of health care records when a following factors is a component of informed
nurse review, client’s charts to assist her in consent?
planning patient care during her shift? a. Freedom from coercion
a. Communication b. Durable power of attorney
b. Education c. Private insurance coverage
c. Auditing d. Disclosure of previous answers given by the
d. Assessment client

3. Medical record that is organized into 6. The rule of the RN includes which of the
separate sections for data from doctors and following principles?
nurses is known by which of the following? a. The RN must directly supervise all delegated
a. SOAPIE record tasks
b. Source record b. After task is delegated, it is no longer the
c. Modified problem-oriented record RN’s responsibility
d. Problem-oriented record c. The RN is responsible for delegated tasks to
conduct personnel
4. You have just finished your morning duty d. Follow up with a delegated task is necessary
shift. Which of the following is NOT part of the only if the assistive personnel is trustworthy
verbal report during your endorsement to the
afternoon shift? Situation: As a headnurse in the medical ward
a. “Mrs. Castro has been uncooperative and for two years, one of the things that you have
overacting the whole morning?” observed that contributes to the quality of
b. “Mr. Cruz is for barium enema in AM.” nursing care in your unit is the observance of
c. Mr. Santos’ pulse is fluctuating. It was accurate documentation and reporting by your
70/min at 9 AM and 100/min at 11:30 AM.” staff.
d. “Mr. Lim’s pain in the leg is relieved by a hot
compress.” 1. Which of the following nursing activities
doesn’t need to be recorded immediately?
5. The main basis of endorsement while doing a. Providing a back rub at bed time
nursing rounds during a change of shift is b. Carrying out the order for blood transfusion
primarily which one of the following? c. Applying a pressure dressing for bleeding
a. Information in the Kardex d. Transporting a client to the x-ray
b. Prescribed medication and therapy department for upper GI series
c. The client’s health needs
d. Routine nursing care 2. Your unit has been constantly recognized for
outstanding endorsement procedure in
between shift and has served as a model to c. Progress notes
other wards. The major purpose of d. Data sheet
endorsement report is to:
a. Inform the physician of a client’s progress 3. Utilizing the SOAP charting, the nurse would
b. Communicate to the other members of the record her nursing diagnosis under which of
team the care rendered to the clients the following?
c. Provide an opportunity for nurses to share a. P-plan of care
common concerns b. S- subjective
d. Provide continuity of care c. O-objective
d. A –assessment
3. The basis for the endorsement report is the
communication to the next shift of the: 4. Which of the following statements about
a. Prescribed medical regimen charting and documentation generally is
b. Client’s health needs correct?
c. Routine nursing orders a. The client record is confidential and cannot
d. Information on the Kardex be used in court
b. All information documented in the incident
4. Which of the following information may not report should be duplicated in the client’s
be included in the verbal endorsement report? medical record
a. “Mr. Tan will be seen by the oncologists in c. Documentation and reporting are less
the morning for the schedule biopsy”. important nursing activities than providing
b. Mr. Baylon’s pulse rate was 85/min at 8:30 client safety and care
AM and 105/min at 1:00 PM d. Mrs. Santos has the right to review her
c. Mr. Diaz has been started on Penadur 500 medical record
mgs every 4 hours 5. Which of the following statements
d. Mr. San Juan has been uncooperative and concerning confidentiality is most accurate?
overacting during the day. a. Nurses are the only one responsible for
protecting their client’s records from
5. When a physician gives a telephone order, unauthorized reader
the only way for the order to be legal is: b. Nurses may only use a client’s record for
a. the physician must give his name over the activities directly related to the client’s health
phone care management
b. the nurse must record the order in the c. Nurses are legally and ethically obliged to
treatment sheet keep information about a client’s confidential
c. the physician must sign the order within the d. A legal suit, without the client’s consent
prescribed time
d. the supervisor must accept the phone order Situation: You often do not finish your nursing
work on time for shift endorsement.
Situation: Mrs. Santos, 80 years old, 1. This is an indicator of effective use of your
underwent surgery for hip dislocation. She was time?
sent back to the ward after a stable condition. a. Keeping a time schedule
During the night, she became confused while b. Using a logbook
attempting to crawl out of bed over the side c. Checking Kardex cards
rails that caused her fall on the floor. d. Reading ward notices
1. Since the event was uncommon and is not
consistent with routine activities in any health 2. What evidence will indicate your effective
care unit, the night supervisor would require use of time?
the staff nurse to write which of the following? a. Doctor’s bills have been issued
a. Affidavit attesting to her innocence b. Charting completed on time
b. Diary to record the event concerning Mrs. c. All serious patients have been cared
Santos d. Patient’s bills have been forwarded to billing
c. Situation report office

2. Having a POMR Record System, the nurse 3. You know that in practice, effective use of
would record her observation about Mrs. time is good management. Which of the
Santos in which of the following part of the following measures would be most helpful in
chart? keeping track of the time?
a. Flow sheet a. Keep a log for the day
b. Nurse’s notes b. Plan your work activities
c. Do a time analysis d. Information on the Kardex
d. Request help in your work
4. Which of the following information may not
4. You observed that much of your time is be included in the verbal endorsement report?
wasted on answering telephone calls. Which of a. “Mr. Tan will be seen by the oncologists in
the following would be a realistic alternative? the morning for the schedule biopsy”.
a. Hung up the phone during peak hours b. “Mr. Baylon’s pulse rate was 85/min at 8:30
b. Train a watcher to answer phone calls AM and 105/min at 1:00 PM”.
c. Recommend a ward clerk for employment c. Mr. Diaz has been started on Penadur 500
d. Assign an attendant to attend to all phone mgs every 4 hours”.
calls d. Mr. San Juan has been uncooperative and
overacting during the day”.
5. Which of the following would you first do?
a. Organize activities by priority 5. When a physician gives a telephone order,
b. Ask an attendant to help you the only way for the order to be legal is:
c. Attend first to patients without watchers a. the physician must give his name over the
d. Complain about your patient load phone
b. the nurse must record the order in the
treatment sheet
c. the physician must sign the order within the
VIII. QUALITY IMPROVEMENT prescribed time
a. Standards of nursing practice d. the supervisor must accept the phone order
b. Nursing audit
c. Accreditation/certification in Situation: As a staff nurse, Miss Cortes should
nursing practice remember that patient’s records are
d. Quality assurance documents that protect her from legal suits in
the future.
Situation: As a headnurse in the medical ward
for two years, one of the things that you have 1. Because of this fact, important observations
observed that contributes to the quality of and interventions are to be communicated
nursing care in your unit is the observance of through the:
accurate documentation and reporting by your a. Medication record
staff. b. Progress notes
c. Flow chart
1. Which of the following nursing activities d. Discharge summary
doesn’t need to be recorded immediately?
a. Providing a back rub at bed time 2. Legally, nursing care that is not
b. Carrying out the order for blood transfusion documented:
c. Applying a pressure dressing for bleeding a. is care provided even if it is not recorded
d. Transporting a client to the x-ray b. may be recorded soon after she remembers
department for upper GI series c. may be recorded during her next round of
duty
2. Your unit has been constantly recognized for d. is considered care that is not provided
outstanding endorsement procedure in
between shift and has served as a model to the 3. Legally, patients’ charts are:
other wards. The major purpose of a. owned by the government since it is a legal
endorsement report is to: document
a. Inform the physician of a client’s progress b. owned by the doctor in change and should
b. Communicate to the other members of the be kept from the administration for whatever
team the care rendered to the clients reason
c. Provide an opportunity for nurses to share c. owned by the hospital and should not be
common concerns given to anyone who request other than the
d. Provide continuity of care doctor-in-charge
d. owned by the patient and should be given
3. The basis for the endorsement report is the by the nurse to her client if requested
communication to the next shift of the:
a. Prescribed medical regimen 4. As a form of information to all those involve
b. Client’s health needs in the care of the patient and to ensure that
c. Routine nursing orders the patient gets all the care prescribed and
planned for her, it is necessary for Miss Cortes a. Descriptive
to document all care that: b. Experimental
a. she provided as well as those she has not c. Exploratory
provided d. Historical
b. the watchers provided
c. she has not provided 3. One study that she read lately in the library
d. she has provided was “The relationship between the health
teaching given to a postpartum primipara
5. The following is true for charting that is mothers having hospital confinement to the
done by nursing students and countersigned by incidence of infection in the newborn”. This
the clinical instructor: study is an example:
a. The staff nurse is purely accountable to the a. Experimental
hospital for the nursing care rendered by the b. Descriptive
nursing students c. Historical
b. The nursing students is legally accountable d. Quasi-experimental
for all information she has written in the
progress notes 4. The title “Development of Instructional
c. The clinical instructor attests that she has Stimulators in Medical and Allied Medical
personal knowledge of the information charted Education” is an example of:
by the student a. Quasi-experimental
d. The instructor cannot be liable because the b. Historical
situation is part of the learning experience of c. Experimental
the students d. Descriptive

5. One study that she read in a foreign journal


IX. RESEARCH was “Improving Physicians and Nurses Clinical
a. Problem identification Breast Examination”. A randomized control
b. Ethics and science of research trial is an example of:
c. The scientific approach a. Quasi-experimental
d. Research process b. Historical
e. Research designs and c. Experimental
methodology d. Descriptive
1. Qualitative
2. Quantitative Situation: After five years of working as a staff
f. Utilization and dissemination of nurse in the medical ward, you were asked by
research findings the chief nurse to be part of the team that will
undertake a hospital wide research on costing
Situation: Ms. Torres is a new faculty of an of nursing services. You recall that one of the
accredited school. One of the expectations in first lessons in your research class is the
that university is for faculty to do research or method by which knowledge is acquired.
publish a scientific paper at least once a year.
In order to appropriately decide what kind of 1. Having worked in the emergency ward for
research Miss Torres is going to conduct by six months, you are convinced that crisis
next year, she must know the different intervention is one of those services for which
research methodology. a nurse can charge professional fee. This
1. One title that she read in a local journal was knowledge is based on:
“Unintended Pregnancies in the Philippines: a. Scientific method
Facts and Issues”. This study can be classified b. Experience
as: c. Consulting expert
a. Quasi-experimental d. Trial and error
b. Historical
c. Experimental 2. The research team learned that some of the
d. Descriptive nursing services of few hospitals are already
approved for costing and are to be submitted
2. Miss Torres remembers one study submitted to Philippine Health for approval. The team
by a student was: “The Intelligence Quotient invited the proponents of the study,
and Milk-Feeding of Selected School-Aged Pupil “Determination of Cost of Nursing Services in
of School X. This kind of research could be an Hospital” for information. The knowledge
example of: obtained from this specific activity is through:
a. Experience c. Ex post facto
b. Consulting expert d. Quasi-experimental
c. Trial and error
d. Tradition 4. If you decide to study the relationship of
eating breakfast and performance in class,
3. On of the problems encountered by the which would be the independent variable:
other members of the research team concerns a. Student’s characteristics
bed sores. In one unit, it was learned that they b. Meal patterns
tried using the bark of banana to line the skin c. Performance in class
where possible pressure sore is suspected. d. Eating breakfast
Eventually they tried water pads to lessen
pressure on the affected part. The knowledge 5. The most appropriate statement of your
obtained in this situation is through: hypothesis is:
a. Trial and error a. There is a positive correlation between
b. Scientific method eating habits and class performance
c. Tradition b. There is a relationship between eating
d. Experience breakfast and performance in class
4. One member of the research team used to c. Performance in class is more likely to be
be assigned in the Reproductive Health Clinic affected when eating breakfast.
of the OPD. The nurses in that clinic were d. Performance in class is associated with
trained on infection control and for a long time eating breakfast
have not been using disinfectant solution for
the sterile container after use and have new Situation: An understanding of the usefulness
sterile pick-up forceps and sterile dry container of scientific findings is more and more essential
everyday. The knowledge of keeping the pick- for quality nursing practice.
up forcep sterile in this manner has been
acquired through: 1. You decided to do a review of literature. The
a. Trial and error most important reason for doing so is to:
b. Scientific method a. Improve your library reading skills
c. Consulting expert b. Get ideas on interpretation of findings
d. Tradition c. Formulate a conceptual framework for the
study
Situation: A surgery of first year students in d. Decide on a sampling method
your school showed that about 40% do not eat
breakfast before coming to school. 2. The sampling method where each member
of the study population has an equal chance to
1. You would like to find out the possible be selected as a subject is called:
causes of this behavior. Which is an a. Purposive sampling
appropriate title of your research? b. Selective sampling
a. Health behavior of first year nursing c. Random sampling
students related to regular breakfast d. Convenient sampling
b. A survey of the eating habits of first year
nursing students 3. The study population where you will select
c. Health habits of first year nursing students your study subjects is referred to as the:
related to nutrition a. Research subjects
d. Health promotion practices of first year b. Study group
nursing students c. Population
d. Universe
2. This is most practical data collection
instrument to use: 4. Which part of the study relates to validity
a. Interview and reliability criteria:
b. Checklist a. Hypothesis
c. Likert’s scale b. Methodology
d. Questionniare c. Variables
d. Instrument
3. The most appropriate research design for
the study is: 5. Facts generally accepted as true or correct
a. Descriptive are referred to as:
b. Retrospective a. Premises
b. Phenomena b. Breastfeed infants
c. Delimitations c. Breastfeeding practice
d. Assumptions d. Education on breastfeeding

Situation: Backache after duty hours has 2. The dependent variable of the study is:
become common among staff nurse assigned a. Education on breastfeeding
in the stroke unit. A group of nurses decided to b. Primipara mothers
make a retrospective study. c. Breastfeeding practice
d. Breastfeed infants
1. A retrospective study would involve nurse
who suffered from backache: 3. A review of literature was done. The most
a. Anytime before or after the start of the important reasons for doing this is:
study a. To learn the research design used
b. During the period of the study b. To provide a theoretical framework for the
c. Previous to the study study
d. No particular period c. To decide on a sampling frame
d. To get ideas on result findings
2. The purpose of the study is to determine the
relationship between lifting technique of nurses 4. The sampling method where each member
and backache. The independent variable is: of the study population has an equal chance to
a. Staff nurses be selected is:
b. Backache a. Convenience sampling
c. Back injury b. Selective sampling
d. Lifting technique c. Purposive sampling
d. Random sampling
3. Fifty nurses consented to be subjects of the
study, 25 nurses with backache and 25 nurses 5. Results of any research finding will be
without backache. Which group is the control? valuable to nurses only when:
a. Group without backache a. Disseminated to nurses concern
b. Group without lifting activity b. Put aside for future use
c. No group c. Published
d. Group with backache d. Filed in library

4. You obtained the written consent of the Situation: When the total population appears
study subjects. Which of the following to be very large, the researcher may decide to
information will be given prior to the consent? get a representative sample and yet make the
a. Assurance to withdraw from the study research findings still valid and acceptable.
anytime
b. Be participant until study is completed 1. This type of sampling uses the technique
c. Right to be informed of finding whereby the population is undivided into areas
d. Right to receive incentives or section and then taking random sample
from each section.
5. Due to some study constraints, the group a. Systematic sampling
decided to a prospective study. This means the b. Stratified sampling
study will be conducted: c. Cluster sampling
a. At a specified future time d. Purposive sampling
b. At a period when subjects are ready
c. When both subject and the lifting activity 2. In this type of sampling, data are collected
are ready from anyone available as those who are
d. When there are patients who need lifting present in the coffee shop or those who are
present in the emergency room at one time or
another.
Situation: A study proposed by a community a. Systematic sampling
nurse has this hypothesis: “Primipara mothers b. Incidental sampling
who received education on breastfeeding are c. Simple random sampling
more likely to breastfeed their infants.” d. Cross-sectional sampling

1. The independent variable of the study is:


a. Primipara mothers
3. In this type of sampling, selection of the
units in the sample is done by some sort of 3. When a nurse prepares a research report to
chance. a captive group, which of the following is NOT
a. Sequential sampling included?
b. Simple random sampling a. Summary of other research studies with the
c. Stratified sampling same result
d. Cluster sampling b. The researcher’s interpretation of the results
of the study
4. A new public health nurse would like to c. A summary of literature used to identify the
collect data on the common problems research problem
encountered by diabetic patients in the d. A description of methods used to conduct
community. In this survey, she selects only the the study
diabetics residing in different barangays of the
community she is assigned. This is an example 4. Which of the following characteristics of a
of what type of sampling? clinical nursing problem is NOT a potential for
a. Purposive sampling research?
b. Cluster sampling a. It has the potential to change how nursing
c. Systematic sampling care is delivered in the unit
d. Stratified sampling b. It is value-laden and involves feelings and
perceptions
5. One nurse is interested to get the effects of c. It reflects something that could be improved
having foods with fortified vitamins A and D in in clinical practice
the development of children from preschool to d. It is a problem that occurs frequently in a
school age level. The type of sampling when a particular client group
given group of subjects are studied for an
extended period of time is called: 5. Which of the following is true in an
a. Cross-sectional experimental study?
b. Longitudinal a. Clients most likely to perform the best are
c. Purposive assigned to the experimental group
d. Cross-cultural b. Subjects are randomly assigned to both the
experimental and controlled groups
c. Conditions affecting the subjects are left
Situation: It is said that the half-life of uncontrolled to generalization of findings
knowledge in the health care filed is 5 years. d. The control group receives the therapy
Because of this, it is important that nurses being studied
directly or indirectly participate in research
activities in order to add knowledge in the Situation: In your study of the health status,
profession. including cause and effect of diseases in your
community, you will apply your knowledge in
1. Which of the following characteristics do statistics and research.
NOT describe scientific investigation? 1. The initial result of the census you made
a. Empirical data are gathered directly or revealed that in 7 families the number of
indirectly through human senses children per family were 2,5,4,3,3,1,3. the
b. It is planned and conducted in a systematic mean number of children per family is:
and orderly manner a. Data is insufficient
c. External factors which may influence the b. 4
relationship among advance phenomena are c. 2
not necessarily controlled d. 3
d. They are conducted to test or develop
theories and study knowledge 2. You will conduct an experimental study on
the causes of neonatal tetanus. The Lasic
2. In which of the following type of research difference between this study and an
does a researcher control the independent observational study is that in the experimental
variable but subjects cannot be randomly investigation, the:
assigned to treatment conditions? a. Investigators determine who is and who is
a. Descriptive not exposed to the suspected causal factor
b. Quasi-experimental b. Study is retrospective
c. Exploratory c. Study and control group may differ in size
d. Experimental
3. In selecting subjects for study you will use c. Community data
randomization. This procedure will ensure that: d. Notes from lecture
a. All these conditions
b. Selection will occur by chance 3. Read very well this statement, Responsible
c. Study and control group have the same parenthood is a 50-50% sharing of
characteristics responsibilities between the husband and wife.
d. Placebo effects are eliminated What part of the research cannon does this
belong?
4. Which of the following measures is used a. Research hypothesis
frequently as a numerator to calculate the b. Problem statement
incidence rate of a disease? c. Tool paradigm
a. Number of old and new cases observed that d. Conceptual framework
occur in a period of time
b. Population at risk during a given period of 4. Responsible parenthood through survey
time research is best analyzed by what design?
c. Number of population at risk a. Correlation
d. Number of new cases observed that occur in b. Qualitative
a period of time c. Triangulation
d. Quantitative
5. Which of the following rates best describe
this statement, “Children in Barrio B between 5. The data that will gather in the study about
the ages of 1 to 5 years old have an average of responsible parenthood should give answer to
3 diarrhea episodes per year. your:
a. Secondary attack rate a. Hypothesis
b. Mortality rate b. Conceptual framework
c. Morbidity rate c. Research problem
4. Case fatality rate d. Instrumentation

6. What type of data collection method in a 6. What is the way of life that is stable,
research on family planning by couples can be enduring, changes over time and is passed on
used by the nurse? from one generation to the next?
a. Questionnaire a. Culture
b. Survey b. Traditions
c. Interview c. Mores
d. Research design d. Habits

5. If research is to be made on eating Situation: One of the terminal competencies


preferences of pre-schoolers, what design of desired of graduates of the BSN curriculum is
study will you use? the development of positive attitudes towards
a. Analytical research.
b. Descriptive 1. The main contribution of research towards
c. Experimental the improvement of the nursing profession is
d. Case control which of the following?
a. Facilitate the development program
Situation: Nurse Vilma is interested to conduct b. Assist the faculty in developing the nursing
a mini-research on responsible parenthood curriculum
1. How should Nurse Vilma start the process of c. Assist the faculty administrator in planning
research? nursing staff requirements
a. Make a survey in the neighborhood d. Provide a theoretical/scientific basis for
b. Identify what should be the focus of the nursing care
study
c. Observe family relationships at home 2. If you are to budget the time allotment for a
d. Watch family interactions in telenovelas and particular study, about 75-80% of this would
family shows be allotted to:
a. Collating and analyzing
2. Where else will Nurse Vilma start the b. Writing the whole data
process of research? c. Survey or interview or actual observation in
a. Conversation with classmates and friends a laboratory or clinical setting to gather data
b. Literature review
d. Identifying research problem, planning on a. It improves clinical competence
the materials and research design to be used b. It develops analytical skills
and the subject to be used in the study c. It contributes to new knowledge
d. It demonstrates professionalism
3. Which of the following statements is NOT
included in describing the limitations? 5. The most important criteria of a
a. Instrument to be used in analyzing data researchable problem is its:
b. Limitation of the research in terms of time a. Potential to improve nursing
and resources b. Relevance to nursing practice
c. Age, sex, religion of the subjects or c. Practicability and clarity
respondents d. Simplicity in practice
d. Knowledge of the researcher
Situation: In clinical nursing research, the
4. Because of the time constraints and limited subjects are generally clients or patients.
funds, delimitations of research study is 1. Doing a random sampling of your study
acceptable. Which one is NOT a consideration subjects ensure that:
in determining a research study? a. Subjects are arranged for organization
a. Novelty of the problem b. Anonymity of subjects are preserved
b. Age and sex of the respondents c. Subjects are assigned to specific groups
c. Geographical location of level students d. Every subject has a chance to be selected
d. Religion and educational attainment
2. When selecting your study sample, which of
5. Which of the following criterion is LEAST the following activities should you do first?
considered in determining whether or not a a. Specify the sampling frame
problem is researchable? b. Identify the target population
a. Cost of the investigation c. Specify the criteria for selection of subjects
b. Nationality of the researcher d. Identify the available population
c. Availability of a particular research
instrument 3. You decided to use convenience sampling.
d. Time factor This is n example of convenience sampling:
a. Selection with consent
Situation: For professional development, it is a b. Simple selection
policy of the nursing service to encourage c. Availability of subjects
nurses to participate in the improvement of d. First come first selected basis
nursing practice.
1. Research contributes to the knowledge base 4. When a subject is selected, your first ethical
of professional nursing. Which of the following consideration is to:
statements is TRUE? a. Obtain consent of subject to participate
a. Research seeks the unknown b. Ensure confidentiality of responses
b. Research solves a problem c. Inform subject when study will start
c. Research studies the world of reality d. Assure anonymity
d. Research relies on observable evidence
5. During the actual collection of data, a
2. When you participate in nursing research, subject refused to participate despite previous
your main activity is to: consent. Which is the appropriate action to
a. Develop problem-solving skills take?
b. Explain a nursing phenomenon a. Proceed as previous consent has been
c. Solve a nursing problem obtained
d. Understand man as a human being b. Respect wish of subject
c. Ask reason for refusal
3. This is a type of research that attempts to d. Convince subject to participate
solve practical nursing problems:
a. Clinical problems research Situation: Research contributes to
b. Basic research improvement of nursing practice. It is essential
c. Applied research that professional nurses understand scientific
d. Quasi-experimental research method.
1. Research in nursing is mainly aimed to:
4. The most important value of research to a. Contribute to professionalism in nursing
practicing nurses is: b. Ensure safe nursing care
c. Solve a nursing problem 5. You decided to include all barangays in the
d. Improve nursing practice municipality and chose a sampling method that
would get a representative sample from each
2. This is a potential good source of barangay. The appropriate method would be:
researchable problems in the clinical area: a. Cluster sampling
a. Patient characteristics b. Stratified sampling
b. Nursing diagnosis c. Systematic sampling
c. Patient’s classification d. Random sampling
d. Modalities of nursing
Situation: A group of nurses decided to
3. The ethical consent in research requires describe the influence of religion on the
which of the following conditions: attitude of staff nurses in the care of dying
a. Consent of the attending physician patients.
b. Written without consent 1. In your review of literature, under what
c. Free and informed appropriate topic would you focus:
d. Must be written a. Care of the terminally ill patients
b. Attitude and religion
4. A review of related literature provides: c. Concept on death and dying
a. Characteristics of subjects d. The dying patient
b. A rational for the research
c. Synthesis of previous studies 2. The variables in your study are the
d. The research methodology following, except:
a. Religion
5. When participating in studies using drugs, b. Setting of the study
the nurse must be aware of: c. Care of the dying
a. Details of research protocol d. Attitude of staff nurses on dying
b. Names of subjects
c. The principal investigator 3. Which of the following research design
d. The investigational drugs would be most appropriate for the study?
a. Descriptive
Situation: Sampling is an essential part of the b. Ex post facto
research process. c. Case study
1. A sample is drawn from the study: d. Quasi-experimental
a. Population
b. Universe 4. This is an example of a continuous variable?
c. Study subjects a. Religion
d. Representative group b. Staff nurses
c. Patients
2. The most important characteristics of a d. Age
sample is its:
a. Population subsets 5. If you chose to concentrate in one
b. Randomization community alone, which research design would
c. Representativeness be appropriate?
d. Appropriate number a. Descriptive
b. Exploratory
3. Random sampling assures that each subject c. Survey
has: d. Case study
a. Been selected based on criteria
b. Been systematically selected Situation: In your study, you examined the
c. Characteristics that match other samples age characteristics of the community using
d. An equal chance of selection descriptive statistics.
1. Descriptive statistics are useful measures in:
4. Which sampling method allows the use of a. Calculating standard deviation
any group of research subject: b. Summarizing data
a. Quota c. Determining frequency of data
b. Purposive d. Estimating central tendency
c. Snowball
d. Convenience 2. The age which occur most frequently is the:
a. Median
b. Mode d. Assure her of your competence
c. Proportion
d. Mean 4. In assessing non-verbal communication,
which of the following observations would be
3. When the mean age of your subjects is 54, most helpful?
which statement is correct? a. Posture and facial expressions
a. 50% of subjects are within normal range b. Physiological signs of anxiety
b. Ages of subjects are skewed c. Reaction to hospitalization
c. Ages of subjects are normal d. Responses to pain
d. 50% of subjects are below 54 years old
Situation: In nursing practice, assisting a
4. When you divide the frequency of each age patient with his problem is a growth facilitating
by the total size of your sample and multiply process both for the patient and the nurse:
the result by 100, the value obtained is: 1. Which of the following nurse behaviors will
a. Percentile be most valuable in establishing a therapeutic
b. Proportion relationship?
c. Ratio a. Attention to desires of the patient
d. Percentage b. Listens to patient’s concerns
c. Gives prompt services
5. Fifty-seven (57) subjects in your study d. Available when needed
ranged in age 21 to 40 years. The median age
of your sample is: 2. This is an example of a therapeutic
a. 31 relationship:
b. 30.1 a. Helping an anxious mother through delivery
c. 30 b. Encouraging a paralytic patient to ambulate
d. 31.1 c. Assisting a diabetic patient to inject himself
d. Providing services solely based on medical
regimen

3. Interacting with your patient involves:


X. COMMUNICATION a. Establishing goals with patient
a. Dynamics of communication b. Provisions of quality care
b. Nurse-client relationship c. Sharing experiences on illness
c. Professional-professional d. Goal-directed verbal and non-verbal
relationship interaction
d. Therapeutic use of self
e. Use of information technology 4. When interacting with your patient, the
Situation: You were assigned to Mrs. most helpful nurse behavior is:
Tagumpay, a difficult patient. a. Explain to patient your services
1. When a client consults you for the first time, b. State your expectation from patient
the best question to start with is: c. A relaxed attending attitude
a. “Why did you come today?” d. Initiate the interaction always
b. “Are you ill?”
c. “What can I do for you?” 5. When a patient cannot see his problem
d. “Do you have any health problem?” clearly, what will be your major task?
a. Assess his level of knowledge
2. Which of the following is not a characteristic b. Ask assistance from co-nurse
genuineness? c. Help clarify his problem
a. Consistent d. identify possible problem cause
b. Defensive
c. Open mindedness Situation: Miss Matias found out that Mang
d. Spontaneous Carding, newly admitted patient, has terminal
cancer and that his nurse has not yet informed
3. In establishing a helping relationship with him of the diagnosis.
Mrs. Tagumpay, which of the following should
be a prime consideration? 1. Initially, Miss Matias should:
a. Ensure open communication a. Tell the doctor that Mang Carding has hinted
b. Show an attitude of concern that he feels he has cancer
c. Develop some degree of trust
b. Be available to listen when the patient a. Effective communication is a two-way
decides to discuss his illness process
c. Call in the head nurse to prepare Mang b. A variety of feelings can be expressed more
Carding about his impending death non-verbally
d. Inform Mang Carding of the diagnosis c. Listening is a part of communication
d. Verbal communications are often more
2. On the second day, the wife of Mang carding effective
seemed to show signs of grieving. The stages
of grieving identified by Elizabeth Kubler-Ross 3. Which of the following measures is specific
are: for impaired communication?
a. Numbness, anger, resolution and a. Refer to a speech therapist
reorganization b. Awareness of speech etiology
b. Denial, anger, identification, depression and c. Provide speech exercises
acceptance d. Develop active listening skills
c. Denial, anger, bargaining, depression and
resolution 4. The overall goal for a client with impaired
d. Anger, loneliness, depression and communication is:
acceptance a. Reduce client’s communication problem
b. Assist client in communication
3. Which of the following therapeutic measures c. Correct faulty perceptions
is not useful in working with grieving families? d. Make honest statements to client
a. Discussing painful periods shared with the
loved ones 5. The best way to communicate information
b. Providing emotional support about a patient is:
c. Encouraging detailed description of sorrow a. Dialogues
d. Discussing plans for the future b. SOAP recording
c. Process recording
4. Which of the following activities would be d. Kardex system
most helpful to relatives working through
grief? 1. Reconciliation to prevent breakdown of the
a. Group meeting with other grieving families sacredness of marriage maybe made possible
b. A course on death and dying by:
c. Volunteer work with hospice care of a a. True forgiveness and migrating abroad
terminal patient b. Prevailing love and understanding
d. Psychotherapy c. Good communication relationship
d. Calling the attention of both parent of the
5. The nurse, when dealing with relatives of couple
dying patients, must be sensitive to their
emotional reactions. A family member who Situation: Stella, 35 years old, learned form
blames herself for the condition of the dying her doctor that she is positive with Human
patient indicates that she: Immunodeficiency Virus (HIV). She is nervous
a. has neurotic tendencies and you want to help her.
b. is oversensitive
c. ahs psychotic tendencies 1. You will prepare a plan of care for Stella. To
d. is potentially a high suicide risk formulate this plan, the first thing that you will
do is to:
Situation: Nurses interact with a variety of a. Discuss with Stella facts about HIV and
individuals and groups, sick and well. Good AIDS
communication skills enhance their caring b. Learn about the specific health needs of
function. Stella
1. This is an example of a non-verbal c. Specify the objectives of your plan of care
communication. d. Prepare a list of agencies where you can
a. Posture and gait possibly refer Stella
b. Body language
c. Gestures 2. You scheduled her for counseling. You can
d. Body image counsel well if you can:
a. Show Stella that you want to help her, give
2. These statements on effective her accurate information and help her make
communication are correct, except:
her own decision about her being positive to
HIV 3. Which of the following is the most important
b. Show that you want to help Stella factor in providing individualized nursing care
c. Give Stella accurate information about HIV to clients in specific cultural groups?
and AIDS a. Communication
d. Help Stella make her own decision about her b. Biologic variations
present condition c. Environmental control
d. Social organization
3. Which of the following is a good way to
greet a client who comes to the health center: 4. Before a nurse can effectively care for a
a. Good morning. Please sit down. The doctor client from an ethnic group other than her
is not yet in. own, it is best for her to do which of the
b. Please sit down. I guess you want some following?
information about HIV. a. Integrate transcultural communication in her
c. Good morning, please sit down. How can I care of patients
help you this morning? b. Determine her own cultural beliefs and
d. Good morning, please sit down. How can I values
help some leaflets about HIV/AIDS. c. Study the different cultural beliefs of the
different ethnic groups in the country
4. In counseling, you can help Stella if you will d. Work with the cultural minority and
let her show her feelings. This can be achieved disadvantaged groups
by:
a. Helping her talk about her feelings 5. In dealing with patients in the ICU, the
b. Performing all these tasks nurse tries to talk with them as close as 1 ½
c. Giving her full attention feet distance or a little more to make the
d. Listening actively and questioning effectively patients experience a close relationship
between them. Which of the following zones
5. Which of these is an effective question? make up the personal space created by such a
a. What have you heard about HIV/AIDS? close interaction?
b. Do you already know something about a. Social distance
HIV/AIDS? b. Personal distance
c. Did your partner know something about the c. Public distance
result of your examination? d. Intimate distance
d. Why did you miss your scheduled check-up?
Situation: Aling Maring, 32 years old from
Situation; Sunshine is a headnurse in the ICU Barrio Maasin was admitted for the first time to
of a big hospital in the city. There are 5 the hospital for diagnostic studies.
patients, one, a Chinese Taipan and the other,
a labor attache. One of the other patients is a 1. What information would be most helpful is
veteran from the Ilocos region and another assessing communication problems of Aling
town mayor from the south who has a stroke. Maring?
The fifth patient cannot speak any dialect a. Educational level
except Bicol. b. Communication style
1. To be effective in interacting with the c. Nonverbal behavior
patient coming from different cultures, which d. Culture-bound health practice
one of the following competencies would be
most appropriate for the nurse to develop? 2. In developing a helping relationship with
a. Language teachnology your patient, it is best that:
b. Information technology a. The family be told about the plan of care
c. Transcultural communication b. Nursing plan of care be prepared by the
d. Interpersonal communication nurse
c. Nursing actions be shared with watcher
2. The nurse facilitates competencies in d. Nursing goals be set with patient
communication appropriate to the above
situation by identifying which of the following? 3. This is a common barrier to meaningful
a. Areas of diversities nurse-patient relationship?
b. Global demands and pressures a. Negative attitude
c. Individual demands of patients b. Failure to emphasize
d. Areas of commonalities c. Close mindedness
d. Ineffective communication c. Anticipate future problems and how patient
will handle them
4. These are communication techniques that d. Terminate the nurse-patient relationship
contribute to a therapeutic relationship,
except: Situation: As a professional nurse practitioner,
a. Anticipate nursing needs of patient it is expected that you are aware of issues and
b. Active listening to what patient says concerns affecting the health care delivery
c. Encourage expression of feelings system in the country.
d. Make yourself available always
1. Traditionally, the most common services of
5. This is an indication that Aling Maring can the health care delivery system include:
effectively communicate. a. prevention of diseases and promotion of
a. Comply with treatment health
b. Follow nurse’s instruction b. hospice and care of the terminally ill
c. Verbalizes her needs c. diagnosis and treatment of diseases
d. Chats with other patients d. rehabilitation and gerontological care

XI. COLLABORATION AND TEAMWORK 2. Ideally, rehabilitation services begin:


a. Networking a. upon admission of clients in the health care
b. Inter-agency partnership system
c. Teamwork strategies b. upon discharge of clients from the health
d. Nursing and partnership with care system
other professions and agencies c. after the client’s physical condition stabilizes
Situation: In health maintenance, one of our d. soon after the client’s request for
nursing goals is to provide continuity of care. rehabilitation service
1. The aim of health maintenance is
a. Referring patient to another health facility 3. In the Philippines, most, if not all, day care
b. Part of your health monitoring function centers can be managed and/or organized
c. Smooth transfer and follow-up care of the nurses because they provide:
patient a. ambulatory care to clients of all ages and
d. To enable patient to remain healthy. are usually situated in malls
b. emergency psychiatric care and counseling
2. Continuity of care is best provided through: to clients experiencing extreme stress
a. Home visits c. care and supervision of older clients usually
b. A referral system those with emotional disturbance
c. Regular clinic follow-up d. care and supervision of pre-school children
d. Barangay health center usually located in the barangay

3. For continuity of care to be successful, 4. The Department of Health is faced with


which role would be most helpful to the nurse: health issues and concerns that need
a. Co-worker immediate attention for the greater benefit of
b. Coordinator the general population. Which of the following
c. Collaborator issues involving health services can be directly
d. Communicator addressed by nurses either individually or
collectively?
4. The most helpful approach to maintain the a. Increasing cost of hospitalization
health gained by a patient during b. Unemployment and underemployment
hospitalization is the: c. Access to health care facility
a. Train the home care member of the family d. Quality health care services
b. Make a regular clinic visit
c. Follow-up home service 5. The legal permission obtained from client
d. Make a discharge plan before an invasive procedure, involvement in
research or administration of experimental
5. Your teaching sessions with your patient is medication and intervention:
nearing its end. What id your most important a. affidavit
goal in the remaining sessions? b. contract
a. Reinforce importance of home health c. informed consent
regimen d. power of attorney
b. Promote patient self-confidence

You might also like